• Shuffle
    Toggle On
    Toggle Off
  • Alphabetize
    Toggle On
    Toggle Off
  • Front First
    Toggle On
    Toggle Off
  • Both Sides
    Toggle On
    Toggle Off
  • Read
    Toggle On
    Toggle Off
Reading...
Front

Card Range To Study

through

image

Play button

image

Play button

image

Progress

1/257

Click to flip

Use LEFT and RIGHT arrow keys to navigate between flashcards;

Use UP and DOWN arrow keys to flip the card;

H to show hint;

A reads text to speech;

257 Cards in this Set

  • Front
  • Back
A 17-year-old patient has periodontitis involving the anterior teeth with sparse plaque. Which of the following is the probable primary pathogen?
A. Actinobacillus actinomycetemcomitans
B. Bactemides forsythus
C. Fusobacterium nucleatum
D. Porphyromonas gingivalis
E. Prevotella intermedia
ACTINOBACILLUS ACTINOMYCETEMCOMITANS

The correct answer is choice A. Adult chronic periodontitis is usually associated with calculus, plaque, inflammation and anaerobic organisms within the periodontal pocket. These organisms include Bacteroides, Fusobacteria, Porphyromonas, and Prevotella. In this case, however, the young age of the patient, and the absence of plaque and calculus indicate a different disease. Juvenile periodontitis has a site predilection for incisors and first molars. It is usually very fast in progression, and not associated with high plaque and calculus levels. Its main cause is the organism Actinobacillus actinomycetemcomitans, also mercifully known as "aa". So the correct answer to question is choice A.
A 17-year-old patient has periodontitis involving the anterior teeth with sparse plaque. Which of the following is the probable primary pathogen?
A. Actinobacillus actinomycetemcomitans
B. Bactemides forsythus
C. Fusobacterium nucleatum
D. Porphyromonas gingivalis
E. Prevotella intermedia
The correct answer is choice A.Actinobacillus actinomycetemcomitans
Adult chronic periodontitis is usually associated with calculus, plaque, inflammation and anaerobic organisms within the periodontal pocket. These organisms include Bacteroides, Fusobacteria, Porphyromonas, and Prevotella. In this case, however, the young age of the patient, and the absence of plaque and calculus indicate a different disease. Juvenile periodontitis has a site predilection for incisors and first molars. It is usually very fast in progression, and not associated with high plaque and calculus levels. Its main cause is the organism Actinobacillus actinomycetemcomitans, also mercifully known as "aa". So the correct answer to question is choice
A 65-year-old man who experiences urinary retention or difficulty in voiding his bladder most likely has
1. carcinoma of the prostate.
2. benign prostatic hyperplasia.
3. malignant neoplasm involving the urethra.
4. bladder metastasis of bronchogenic carcinoma
The correct choice is choice 2, benign prostatic hyperplasia. Benign prostatic hyperplasia or BPH is so common that it is considered by some to be a normal consequence of aging. Half of men over the age of 50 have BPH and almost all men over the age of 70 have BPH. BPH is characterized by the formation of nodules in the prostatic tissue surrounding the prostatic urethra. When these nodules enlarge, urinary obstruction often follows. Most obstruction is partial obstruction. It causes voiding difficulty. The individual cannot fully empty his bladder and he must make frequent trips to the restroom. Complete obstruction may also occur. The individual rapidly develops hydronephrosis and acute renal failure. This is a life-threatening condition as the renal failure become less and less reversible as time elapses. Carcinoma of the prostate, choice 1, rarely causes urinary retention. The carcinoma arises in the lateral lobes of the prostate. It must reach very large proportions to compromise the prostatic urethra which is located centrally. Malignant neoplasms of the urethra, choice 3, may cause urinary retention. These lesions are extremely rare, however. Bronchogenic carcinoma. choice 4, may metastasize to the bladder. However, metastatic lesions and primary tumors of the bladder usually cause hematuria rather than urinary retention. Once again, the correct choice to question is choice 2
A benign neoplasm of the myometrium of the uterus is a
1. myeloma.
2. fibroma.
3. leiomyoma.
4. myoblastoma.
5. rhabdomyoma
Leiomyoma, or fibroid tumors, are benign smooth muscle neoplasms of the uterus and are the most common tumor in women. They rarely transform into malignant tumors. Myelomas, choice 1, are plasma cell dyscrasias with multiple plasma cell tumors scattered throughout the body. Choice 2, fibromas, are tumors of connective tissue often found in ovaries, but not myometrium of the uterus. Choice 4, myoblastomas, are benign circumscribed lesions of soft tissue often found in the tongue but not of muscular origin. They possibly arise from Schwann cells. Choice 5, rhabdomyomas, are benign cardiac muscle tumors usually seen in children. So the correct answer to question is choice 3.
A can of disinfectant spray states that it kills HIV virus. This statement:
A. Is important because of the difficulty in killing this virus
B. Shows that the disinfectant will kill most other pathogens
C. Illustrates the use of a "benchmark" organism
D. Is not a good indication of disinfectant effectiveness
The correct answer is choice D. Not a good indicator. HIV virus is notoriously easy to kill on most environmental surfaces. Many disinfectants, and other common physical and chemical exposures will kill the virus fairly easily. Therefore, ability to kill HIV is NOT an important criterion in choosing disinfectants. Mycobacterium tuberculosis is chosen as the "benchmark organism" (the standard against which the disinfectant is compared). It is very resistant to surface disinfectants, partially due to its waxy cell wall. Another highly resistant pathogen on environmental surfaces is Hepatitis A virus.
A common oral manifestation of Addison's disease is
1. melanosis.
2. xerostomia.
3. glossodynia.
4. "cotton-wool" bone.
5. loss of lamina dura
The correct choice is answer #1, melanosis. Addison's disease is the term used for adrenal insufficiency of any etiology. Adrenal insufficiency may be primary due to autoimmune phenomena, tuberculosis infiltration, metastatic tumor or amyloidosis, or it may be secondary due to abnormalities of the hypothalamus or pituitary, or exogenous administration of steroids. In primary adrenal insufficiency, feedback inhibition of adrenal hormones on the hypothalamus and pituitary is lost. Consequently, the production of adrenocorticotropic hormone (ACTH) is increased. Linked to ACTH production is the production of melanotropin and this hormone causes pigmentation of the skin and mucous membranes in a phenomenon known as melanosis. Xerostomia, choice 2, means dry mouth. It can be seen in a variety of diseases, but is most common in Sjogren's disease and other rheumatologic illnesses. Glossodynia, choice 3. means painful tongue. This can result from a local burn or trauma or from metastatic tumor infiltration of the lingual nerve. The term "cotton wool bone", choice 4, describes the radiographic appearance of bone in Paget's disease. Paget's disease is a metabolic disorder of bone characterized by an increased rate of bone resorption coupled with new bone synthesis. The lamina dura, choice 5, is a thick layer of tissue lining the dental alveolar bone. This tissue is lost in periodontal disease, but is uninvolved in Addison's disease. Once again, the correct choice to question is choice 1.
A complication of peptic ulcer disease that accounts for the majority of deaths is
1. bleeding.
2. perforation.
3. surgical complication.
4. malignant transformation.
5. obstruction from edema or scarring
The correct answer is choice 2. Perferation. Peptic ulcers are chronic isolated ulcers in areas of the esophagus, stomach or duodenum bathed by pepsin and HCL, and are the result of lowered mucosal resistance. The major complications of peptic ulcers include hemorrhage, perforation, obstruction and pain. Malignant transformation, choice 4, is rare in stomach ulcers and unknown in duodenal ulcers, and so is incorrect. Bleeding, choice 1, is a common problem with ulcers, but does not account for the majority of deaths, which is what the question is looking for. Perforation, in which the ulcer grows until it is completely open through the outer wall of the stomach and into the peritoneal cavity, is a serious medical problem and may cause death, and it is our correct answer. Choice 3 is incorrect. Most ulcers are not surgically treated and those that are do not commonly result in death through surgical complications, so it is not the most common cause of death. Likewise, in choice 5, obstruction may occur but does not lead to death as often as does perforation. So the correct answer to question is choice 2.
A deficiency in which of the following cell types is most associated with the development of oral candidiasis?
A. Neutrophils
B. T-lymphocytes
C. B-lymphocytes
D. Plasma cells
E. Macrophages
The correct answer is choice B. T lymphocyes. This question indirectly links immunodeficiency, such as that found in HIV disease, and the development of candidiasis. The major target cells of HIV virus are the T-lymphocytes, specifically the T4 (CD4) type. As the T-lymphocyte count drops, numerous opportunistic infections occur, including the one most familiar to dentists, oral candidiasis. All of the cells listed are leukocytes and are involved in protective and immune responses, However, the T-lymphocyte population is most heavily damaged and most involved in keeping normal levels of Candida from growing out of control. Remember that B-lymphocytes are most involved in humoral immunity, and T-lymphocytes in cell mediated immunity. Some B-lymphocytes become plasma cells, which secrete antibodies. Macrophages and neutrophils are phagocytic, among other functions
A fungus that causes systemic disease, most commonly of the lungs, and is characterized by its production of tubercolate chlamydospores in culture is
A. Microsporum canis.
B. Mycoplasma hominis.
C. Leptospira pomona.
D. Actinomyces israelii.
E. Histoplasma capsulatum
The correct answer is choice E. Histoplasmosis capsulatum transmission is mediated by airborne inhalation of spores that get deposited in alveoli and spread through lymphatics to the regional lymph nodes. Clinical manifestations include acute and chronic pulmonary infections that very rarely progress to a disseminated histoplasmosis. Choice A - Microsporum canis is incorrect. Choice B - Mycoplasma hominis is a sexually transmitted agent and is a major source of infection in postpartum women. Choice C - Leptospira pomona is caused by ingestion or contact through broken skin or mucous membranes with water contminated by urine from infected reservoirs (dogs, sheep, goats cattle, horses, & rats). Choice D - Actinomyces israelii are part of the normal oral flora and are usually pathogenic only after oral trauma. Therefore the correct answer to question is E.
A grayish pseudomembrane is found in an infection caused by a species of:
A. Streptococcus
B. Corynebacteria
C. Staphylococcus
D. Pseudomonas
The correct answer is choice B. Corynebacteria include the pathogen C. diphtheriae, which causes diphtheria. In particular, the bacteria excrete an exotoxin that causes the disease. Diphtheria is spread by droplets, and the peptide exotoxin is absorbed by epithelia, and the necrotic epithelium, fibrin, and blood cells form the familiar "pseudomembrane." The pseudomembrane covers the pharynx and tonsils, and the toxin produced there may do distant damage to liver, kidneys and heart, etc
A lesion on which of the following nerves would not affect the gustatory pathway?
A. V
B. VII
C. VIII
D. IX
E. X
Cranial nerve VIII (choice C) or the vestibulocochlear nerve is the only cranial nerve listed that is not associated with the gustatory pathway. Damage to this nerve during a surgical procedure would not affect any aspect of the gustatory pathway. The vestibulocochlear nerve is primarily associated with balance, equilibrium and hearing. Cranial nerve V or the trigeminal nerve (choice A) provides information concerning the general texture of food as well as the taste-related sensations of peppery or hot. The taste buds are monitored by cranial nerves VII (Facial) (choice B), IX (Glossopharyngeal) (choice D), and X (Vagus) (choice E). The sensory afferents of these nerves synapse within the nucleus solitarius of the medulla oblongata and the axons of the postsynaptic neurons enter the medial lemniscus, which join axons carrying somatic sensory information. This type of sensory information is related to touch, pressure and propioception
A mother states that when she feeds or bathes her newborn, the infant becomes cyanotic; however, the baby appears normal the rest of the time. What is the most likely cause of the infants signs and symptoms?
A. Anemia
B. Asthma
C. Patent ductus arteriosus
D. Septicemia
E. Upper respiratory infection
The correct answer is C. Congenital circulatory problems are normally associated with an abnormal formation of the heart or problems with the interconnection between the heart and the major blood vessels. For example, the incomplete closure of the ductus arteriosus results in the bypassing of the lungs and the recirculation of blood into the systemic circuit. Since normal oxygenation of the blood does not continually occur, the infant can become cyanotic during of stress. When the baby is eating or bathing, blood is shunted to the stomach or skin respectively. When these stressful events are NOT occurring, there is sufficient blood flow to the lungs to prevent cyanosis. Although there are several kinds of anemia (choice A), the classic signs and symptoms of this condition are chronic fatigue and tachypnea on exertion. Asthma (choice B) is characterized by chronic (or episodic) wheezing, dyspnea and cough. Septicemia (choice D) is defined as a severe systemic infection. If the child had septicemia, she would not appear normal most of the time; the infant would appear very lethargic, hyperthermic and "septic." If the infant had an upper respiratory infection (choice E), the child would most likely appear weak, lethargic and would be coughing.
A neoplasm composed of all three germ layers is referred to as a(n):
A. adenoma
B. osteoma
C. teratoma
D. sarcoma
E. carcinoma
The correct answer is choice C. Simple neoplasms arise from one cell type or germ layer. Mixed neoplasms arise from two, and teratomas ("monster growths") arise from all three and often have a bizarre appearance. Cancers arising from epithelium or endoderm are usually called carcinomas (choice E), whereas those originating from mesoderm are called sarcomas (choice D). An adenoma (choice A) is a benign epithelial tumor, and an osteoma (choice B) is a benign bone tumor
A patient with dentinogenesis imperfecta has a history of multiple fractures. Examination reveals blue sclera. The most probable diagnosis is
1. achondroplasia.
2. Marfan's syndrome.
3. osteogenesis imperfecta.
4. Albers-Schonberg disease
Osteogenesis imperfecta is a hereditary disease of collagen metabolism. It is characterized by thin deformed bones, blue sclera, fractured ear ossicles, malformed discolored teeth, and mitral valve prolapse. Choice 1 is incorrect. Achondroplasia refers to a hereditary disease of impaired cartilage formation, poor long bone formation and dwarfism. Choice 2 is also incorrect. Marfan's syndrome is a hereditary connective tissue disease, possibly due to defective elastin in ground substance. Clinically, it may involve aortic aneurysm, mitral valve prolapse, or subluxed lens. Choice 4. Albers-Schonberg disease is another term for osteopetrosis, a rare hereditary disease characterized by abnormally dense bone due to defective resorption of immature bone. So the correct answer to question is choice 3
A patient with facial erythema, fever, Raynaud's phenomenon, joint pain and photophobia probably has
1. scleroderma.
2. Paget's disease.
3. Sjögren's syndrome.
4. erythema multiforme.
5. systemic lupus erythematosus
The correct choice is choice 4, systemic lupus erythematosus. Systemic lupus erythematosus or SLE is an autoimmune disease primarily affecting women. Antibodies to a host of nuclear and cytoplasmic antigens develop as in other autoimmune diseases. The most specific marker of SLE, however, is antibody directed against double-stranded DNA. The anti-nuclear and anti-cytoplasmic antibodies are responsible for many manifestations of the disease. They deposit as antibody antigen complexes in the kidneys, joints, and blood vessels, and activate complement. Tissue destruction then ensues. In addition to the symptoms mentioned in the question, an individual with SLE may suffer from hematuria, proteinuria, nephrosis, pericarditis, pleuritis, lymphadenopathy, leukopenia, thrombocytopenia, and decreased cognition. The disease tends to be chronic with intermittent exacerbations and remissions. Exacerbations require steroid therapy to minimize tissue destruction. Scleroderma, choice 1, is also an autoimmune disease. Many anti-nuclear and anti-cytoplasmic antibodies can be isolated in affected individuals, the most specific of which are antibodies against centromeres and nonhistone nuclear proteins. This disorder is characterized by extensive fibrosis involving the skin and viscera. It begins with thickening of the hands and arthritis. As it progresses, the esophagus thickens, causing dysphagia. The disease may also involve the lungs, leading to pulmonary fibrosis; tke lower GI tract, and the kidneys. Paget's disease, choice 2, is a very common, usually asymptomatic, disease of bone characterized by bone destruction, with excessive bone reformation. When symptoms do occur, they usually include musculoskeletal pain, pathologic fracture, and skeletal deformity. The etiology of Paget's disease is not known. Sjogren's disease, choice 3, is another autoimmune phenomenon. The most specific marker of this disease is antibody called SSB E which is directed against an extractable nuclear antigen. Clinically, Sjogren's disease is characterized by the sicca syndrome which includes dry mouth and dry eyes caused by lymphocytic infiltration of the salivary and-lacrimal glands. Sjogren's disease is commonly associated with rheumatoid arthritis or SLE, but by itself is not a disseminating disease. Erythema multiforme, choice 4, may be associated with SLE, but more commonly it represents a hypersensitivity reaction to drugs or infectious agents. It is characterized by a maculopapular rash which develops into a bullous process. Erythema multiforme is uncommon and self-limited. Once again, the correct choice to question is choice 5.
A patient, with which of the following disease states, is at the highest risk for development of an oral infection following a tooth extraction?
A. Congestive heart failure
B. Depression
C. Diabetes mellitus
D. Hypertension
E. Hyperthyroidism
The correct answer is C. Diabetes mellitus is a disease characterized by polyuria, polydipsia, ketonemia, and/or ketonuria. Complications of this disease are caused by prolonged periods of blood glucose levels greater than 126 mg/dL. Patients with diabetes mellitus are at an increased risk for post-operative infections and impaired wound healing secondary to the sustained periods of hyperglycemia as well as the circulation problems commonly seen in diabetic patients.
Therefore, of the diseases listed, patients with diabetes mellitus are at the highest risk for infectious complication and impaired wound healing following a surgical procedure. Congestive heart failure (choice A) is a condition characterized by an abnormality of cardiac function. In this condition, there is a failure of the heart to pump blood throughout the body at a rate sufficient to meet the requirements of the metabolizing tissues and/or the heart is only able to meet these requirements when the end-diastolic volume is abnormally elevated. This disease is not associated with an increased development of oral infections. Depression (choice B) is a psychiatric disorder where patients have prolonged feelings of hopelessness, sadness, worthlessness, and changes in appetite and sleep behavior. Often, these patients relate feelings of anxiety or guilt and have difficulty thinking or concentrating; suicidal ideation may also be present. Although patients with psychiatric disorders are more prone to inflict self-injury, their ability to heal following a surgical procedure is not impaired. Hypertension (choice D) is a disease associated with high blood pressure. Over 50 million Americans have this disease and the majority of them experience no impairment of wound healing following a surgical procedure. Hyperthyroidism (choice E) is an endocrine disorder characterized by sweating, weight change, nervousness, irritability and fatigue; this hypermetabolic disorder is generally NOT associated with impaired wound healing. However, a patient with hypothyroidism will commonly experience some degree of impairment of wound healing following a surgical procedure.
A person with anti-A and anti-B isoagglutinins in the serum belongs to blood group
1. A
2. B
3. AB
4. O
The correct choice is choice 4, 0. Isoagglutinins are antibodies which cause clumping of cells. These antibodies are not normally present in all individuals. They are only present in those individuals who lack the specific antigens with which the antibodies react. Blood is typed on the basis of the antigens present on the red cell membrane. Type A blood cells, for example, have A antigen on their surface. An individual with type A blood has antibody to B antigens. Individuals with type AB blood have both A and B antigens. They have no anti-A or anti-B antibodies in their serum. If they did, they would agglutinate their own red cells. Type 0 blood cells have no antigens of the A or B class on their surface. Individuals with type 0 blood have anti-A and anti-B antibody in their serum. Consequently, type 0 individuals may only receive type 0 blood during a transfusion. The lack of A and B antigens on the surface of type 0 red blood cells enables type 0 individuals to donate blood to individuals of any blood type. Type 0 individuals are consequently called universal donors, while type AB individuals are termed universal recipients. Once again, the correct choice to question is choice 4.
A positive skin test in an individual who has had tuberculosis is an example of
1. atopy.
2. autoimmunity.
3. hypersensitivity.
4. passive immunity
The correct choice is choice 3, hypersensitivity. When tuberculin is injected into an individual who has had TB in the past, delayed type hypersensitivity results. In this reaction, memory T-lymphocytes recognize the tuberculin, which is a protein lipopolysaccharide extract of the Mycobacterium, and these T-lymphocytes secrete lymphokines. Lymphokines are chemicals which attract macrophages and other mononuclear inflammatory cells to the site. Within 24 hours, fibrin is deposited, leading to induration; vascular permeability is increased, leading to edema; and vasodilatation causes erythema. Atopy, choice 1, is an acute reaction. Here, sensitized mast cells are coated with IgE molecules. When these molecules are cross-linked by the introduction of antigen, degranulation of mast cells results. Histamine is released which causes bronchospasm and bronchial edema, as are various chemotactic agents which bring eosinophils and neutrophils to the site. Common antigens causing atopy include ragweed and drugs. Autoimmunity, choice 2, is the phenomenon responsible for diseases such as systemic lupus erythematosus or myasthenia gravis. In these diseases, the body produces immunoglobulins against host tissue. In lupus, antibodies against nuclear and ribosomal proteins are formed. In myasthenia gravis, antibodies against acetylcholine receptors are produced. Passive immunity, choice 4, refers to the process whereby immunoglobulins from one host are transferred to another host in whom they confer protection. Passive immunity was the primary immunization strategy utilized by the medical community prior to the advent of vaccines. Passive immunity continues to play a fundamental role in the protection of newborns. IgG is transferred from the mother to the fetus through the placenta and from the mother to the newborn through breastfeeding. The infant depends on these immunoglobulins to fend off infection during the first six months of life, while the infant's immune system is maturing. Once again, the correct choice to question is 3.
A summer illness that produces vesicular lesions of the uvula, anterior pillars and the posterior pharynx is
1. influenza.
2. herpesvirus.
3. ECHO virus.
4. coxsackievirus
The correct answer is choice 4. Coxsackie viruses are enteroviruses that produce a variety of illnesses in humans. Herpangina is caused by Coxsackie virus and is characterized by fever, sore throat and pharyngitis with discrete vesicles on the anterior pillars of the fauces, palate, uvula, tonsils and tongue. The disease is more common in summer and is usually self-limiting. Choice 1 is incorrect. Influenza will not produce the vesicular lesions described and is not more common in summer. Choice 2 is incorrect. Herpes virus may cause skin infections, aphthous stomatitis, eczema, conjunctivitis, encephalitis, cold sores, and general herpes. However, none of these conditions fit the description given for the illness in this question. Choice 3 is incorrect. Echovirus is an enterovirus which may cause meningitis, febrile illness with rash, or diarrheal disease. It is more common in the summer as is the disease caused by Coxsackie virus. However, it does not cause an illness with the symptoms described in the question. So the correct answer to question is choice 4.
A virus causing recurrent herpetic lesions is never completely eliminated but assumes a state of
1. latency.
2. symbiosis.
3. mutualism
4. commensalism
5. none of the above
Latency. Varicella, or chicken pox, and zoster, or shingles, are both caused by the same herpes virus. The virus may infect an individual, cause varicella, and then become latent in neurons of sensory ganglia. At a later time, it may reactivate, often when the patient is under physical or emotional stress or is immunocompromised. This form of the disease will be called zoster. Choices 2, 3 and 4 are ecological terms. Symbiosis refers to a close food relationship or other working relationship between species. Among the symbiotic relationships are mutualism, choice 3, a symbiotic relationship where both species benefit; and commensalism, choice 4, a symbiotic relationship where one species benefits while the other is not affected either positively or negatively. None of the choices 2, 3 or 4 refers to recurrent herpetic lesions caused by latent herpes virus. So the correct answer to question is choice 1
A woman just received a kidney transplant and pathological effect on the body would be is currently taking a medication to prevent organ rejection. What is the most likely mechanism of action for this medication?
A. Increases blood flow to the kidney
B. Increases the number of antibodies in the blood
C. Inhibits the blood from clotting
D. Prevention of inflammation in the kidney
E. Reversible inhibition of helper T lymphocytes
The correct answer is E. Reverisible inhibition of helper T lymphocyes. When an individual receives an organ transplant the body will most likely perceive this organ as being foreign (as an antigen). Since antigenic "substances" in body will be attacked by the immune system, certain portions of the immune system must be inhibited to prevent organ rejection. Cyclosporine is an immunosuppressant agent primarily used to prevent organ rejection in patients who have received a kidney transplant. The mechanism of action for this immunosuppressant agent is related to its ability to reversibly inhibit the action of helper T lymphocytes. By reversibly inhibiting the helper T lymphocytes, the entire immune system will not be able to elicit an immune response against the "foreign" organ. Adequate blood flow to the kidneys is necessary; however, increasing blood flow to the kidneys (choice A) will not prevent an immune response. Increasing the number of antibodies in the blood (choice B) will promote organ rejection. Inhibiting the blood from clotting (choice C) and preventing inflammation in the kidney (choice D) are necessary; however, these factors will not prevent an immune response.
Addison's Disease, a disorder of adrenal insufficiency, can lead to the development of oral lesions in any area of the mouth; however, these oral lesions are most commonly found on the
A. Buccal mucosa
B. Dorsum of the tongue
C. Gingival margin
D. Soft palate
E. This disease is not associated with the development of oral lesions
The correct answer is A. Buccal mucosa Addison's disease is a condition caused primarily by inadequate glucocorticoid production. This disorder is primarily caused by a destruction of the zona fasciculata by an autoimmune response of the body. This disorder primarily produces oral lesions on the buccal mucosa, although these lesions can appear anywhere in the mouth. The lesions are typically blotches or spots of bluish-black to dark brown pigmentation. Furthermore, there is a diffuse pigmentation of the skin around the lesions. In addition to these oral lesions, individuals will normally experience a dramatic decrease in blood glucose levels several hours after a meal. In other words, individuals with this disorder may experience hypoglycemia. In this disorder, there usually are elevated ACTH levels. Furthermore, one would typically expect to see hyponatremia and excessive water loss. One of the classic characteristics of Addison's disease is that patients often present with chronic weakness secondary to a continual weight loss
Adenocarcinomas of the large intestine are most common in which segment?
1. Ascending colon
2. Transverse colon
3. Descending colon
4. Rectosigmoid colon
The correct answer is choice 4. Rectosigmoid Colon. Adenocarcinomas of the colon or large intestine constitute approximately 98% of all colonic cancers. They are environmentally linked and are associated with dietary factors including possibly low fiber diets or excess meat and fat intake. They are also associated with villous adenomas such as ulcerative colitis, Crohn's disease, Gardiner's syndrome and familial polyposis. They can spread by extension or metastasis. Approximately 75% occur in the rectum or sigmoid colon. So the correct answer to question is choice 4.
Administration of tetanus toxoid provides what type of immunity?
A. Innate
B. Natural active
C. Natural passive
D. Artificial active
E. Artificial passive
The correct answer is choice D. Artificial active
. Tetanus toxoid is a form of tetanus toxin damaged by heat or formalin, so that it is not dangerous, but is still immunogenic. It can be administered to the patient, who then develops anti-toxin antibodies. Since the patient develops his/her own antibodies, it is active. Since the antibodies are produced against a manufactured substance,(such as a toxoid or a vaccine) and not the organism itself, it is artificial. In passive immunity, antibodies are formed in another person or organism, and transferred to the affected individual. An example of natural, passive, is antibodies transferred from mother to child across the placenta. An example of artificial passive is tetanus antitoxin, antibodies formed by other individuals, transferred to a person following tetanus exposure. Note that teteanus toxoid and tetanus antitoxin are two different substances, the first causing antibody production, and the second giving passive immunity from disease. So the correct answer to question is choice D
Aflatoxin is produced by
A. Mucor.
B. Candida.
C. Tricophyton
D. Penicillium
E. Aspergillus
The correct answer is choice E. Aflatoxin is produced by Aspergillus. Aspegillosis is a disease arising from several species of ubiquitous molds. Organisms are normal inhabitants of the soil, and spores are readily disseminated into the air. Choice A - Mucor does not produce Aflatoxin and is most often caused by the organisms Rhizopus and Mucor. These molds are ubiquitous on decaying vegetable matter in soil. Choice B - Candida does not produce Aflatoxin , and is a normal inhabitant of mucocutaneous body surfaces, soil, hospital environments and some foods. Choice C - Tricophyton does not produce Aflatoxin and is a mycotic infection of any keratinous structure of the skin and its appendages. Choice D - Penicillium does not produce Aflatoxin and is the source of penicillin. Therefore the correct answer to question is E
After a hamstring injury, an individual has difficulty flexing and medially rotating her thigh. What muscles of the hamstring group did this individual most likely injure?
A. Biceps femoris
B. Rectus femoris
C. Satorius
D. Semimembranosus
E. Vastus intermedius
The correct answer is D. Semimembranosus. This pathophysiology "type" question is basically asking "What muscle is responsible for flexing and medially rotating the thigh?" The semimembranosus muscle has this function. Note that the semitendinosus muscle also has this same action. The biceps femoris (choice A) flexes, extends and adducts the thigh. The rectus femoris (choice B) extends the leg and flexes the thigh. The satorius (choice C) flexes the leg as well as flexes and laterally rotates the thigh. The vastus intermedius (choice E) is responsible for extending the leg.
All of the following are associated with Cushing syndrome EXCEPT:
A. abnormal fat deposition
B. "moon face"
C. hypotension
D. diabetes
E. muscle wasting
The correct answer is choice C. Hypotension. Cushing syndrome symptoms are based primarily on the effects of excess cortisol and sometimes on associated excess aldosterone. Abnormal deposition of fat (choice A) causes an unusual physical appearance, including both "moon face" (choice B) and "buffalo hump." Also, the patient often becomes obese. Diabetes (choice D) results from the glucose-raising effects of cortisol in about 20-30% of cases. Also common are weakness, fatigue, and susceptibility to infection (choice E). Hypertension (not hypotension) is also common and is believed to be related to aldosterone increases, which often accompany the cortisol increases
All of the following are associated with primary aldosteronism EXCEPT:
A. hypernatremia
B. hyperkalemia
C. hypertension
D. polyuria
E. polydipsia
The correct answer is choice B.hyperkalemia Much of the answer can be reasoned from the standard effect of aldosterone, which causes the nephron to resorb more sodium and excrete a watery hypotonic urine. Excess sodium retention causes hypernatremia (excess sodium levels) (choice A) in tissue. The excretion of large volumes of water is polyuria (choice D), and to make up for the fluid loss, large amounts of water are taken in (polydipsia) (choice E). Note that in normal kidney physiology, retention of sodium is accompanied by loss of potassium. The excess loss of potassium results in a condition known as hypokalemia (low potassium), not hyperkalemia (high potassium).
All of the following are characteristic of the stools seen in patients with Giardia infections EXCEPT
A. Blood is present
B. Mucus is not present
C. The diarrhea is malodorous
D. They are greasy and tend to float
E. The diarrhea caused can be acute or chronic in nature
The correct answer is A. Blood is present
. Giardiasis is a protozoal infection of the upper small intestine caused by Giardia lamblia. Persons of all ages can be affected by this organism; however, the incidence is highest in children. A large percentage of individuals infected with this organism are asymptomatic. However, those who are symptomatic for diarrhea typically present with the following characteristic stools: The stools are typically free of blood and mucus (choice B). The stool is often very malodorous (choice C), greasy, tends to float (choice D) and is often bulky in nature. Depending on the severity of the infection, the diarrhea caused can be acute or chronic in nature (choice E)
All of the following are true concerning "fibroid tumors" of the uterus EXCEPT:
A. malignant transformation is rare
B. they may require removal due to bleeding tendency
C. they are also known as leiomyosarcomas
D. they are composed of smooth muscle
E. they are the most common tumor in women
The correct answer is choice C.. they are also known as leiomyosarcomas FALSE!
Fibroids are benign smooth muscle tumors (choice D) and are the most common neoplasm in women (choice E). They rarely undergo malignant transformatron (choice A) but are often removed surgically due to effects on fertility and increased bleeding (choice B). They are actually leiomyomas (benign) and not leiomyosarcomas (rare malignant neoplasms). Symptoms may include uterine bleeding, abnormal menstrual bleeding, infertility, pain, or urinary symptoms
All of the following are true concerning carcinoma of the prostate EXCEPT:
A. it is the most common cancer in American males
B. it is usually an adenocarcinoma
C. symptoms may mimic those of BPH
D. it can be detected by digital rectal exam
E. it can be detected through specific antigen
The correct answer is choice A. Prostate cancer is the second most common cancer in men (lung cancer is first). It is usually an adenocarcinoma (choice B). Like benign prostatic hypertrophy (BPH), it often shows first as difficulty in urination, has a fairly high incidence in men over 50, and incidence increases with age (choice C). Survival was formerly significantly less than now due to late discovery with symptoms or rectal exam (choice D). The use of PSA tests (prostate specific antigen) (choice E) for screening of men over 50 has resulted in more early detection and treatment. The use of radioactive seeds has joined surgical removal and beam radiation as a standard treatment.
All of the following are true concerning mesothelioma EXCEPT:
A. it is related to asbestos exposure
B. asbestos and smoking are synergistic causes
C. it has an extremely long latent period
D. it is not a bronchogenic tumor
E. it has a poor prognosis
The correct answer is choice B. Surprisingly, smoking does not act synergistically with asbestos in causing mesothelioma. These two factors are synergistic in causing bronchogenic tumors (choice D). Mesothelioma is not a bronchogenic tumor, but instead is a tumor of the pleura of the lung. The classic case of mesothelioma involves asbestos exposure (choice A), usually occupational, followed by an extremely long latent period of 25-40 years (choice C) before symptoms of the tumor develop. Mesothelioma generally has a very poor prognosis (choice E).
All of the following are typical inhabitants of damaged tissue in Vincent's stomatitis EXCEPT:
A. Peptostreptococcus
B. Streptococcus
C. Bacteroides
D. Fusobacteria
E. Borrelia
The correct answer is choice B. Streptococci are Gram-positive facultative anaerobes. They are common inhabitants of both the soft tissues and hard tissues of the mouth. They are not found, however, to a great degree in lesions of Vincent's stomatitis (acute necrotizing ulcerative gingivitis (ANUG, trenchmouth). The flora found in this disease are primarily anaerobic and include spirochetes, Borelia (choice A), Bacteroides (choice C), Fusobacteria (choice D), and Peptostreptococcus (choice E
Allergens that are usually responsible for contact hypersensitivity are
1. lipids.
2. haptens
3. proteins
4. carbohydrates
The correct choice is choice 2, haptens. Contact hypersensitivity occurs when an individual who has been previously exposed to an allergen comes in contact with the allergen. During the prior exposure, mast cells became coated with IgE molecules. On repeat exposure, the allergen cross-links the IgE molecules and mast cell degranulation ensues. This causes edema, inflammation and erythema at the site. Eosinophils are attracted which serve to attenuate the inflammatory response. The allergens which cause contact hypersensitivity can be lipid (choice l), protein (choice 3), or carbohydrate (choice 4). Regardless of their composition, they are referred to as haptens. Haptens are molecules which cannot induce antibody production unless they are complexed to a large protein. Haptens are capable of cross-linking antibodies on their own, however, as long as those antibodies have been previously produced. Thus, haptens can initiate the hypersensitivity response. Once again, the correct choice to question is choice 2
Amyloidosis is characterized by all of the following EXCEPT:
A. cells notable for basophilic stain reaction
B. proteinuria
C. glomerular damage
D. hepatomegaly
E. cardiac arrhythmias
The correct answer is choice A. cells notable for basophilic stain reaction False!
Amyloidosis involves deposition of highly specialized proteins in various organs. The proteins include an immunoglobulin-like AL (amyloid light chain) and a unique nonimmunologic AA (amyloid-associated protein), as well as a glycoprotein P component. Deposition in the kidney causes glomerular damage (choice C) and proteinuria (choice B). Deposition in the spleen and liver cause hepatosplenomegaly (choice D). Damage to the heart may involve cardiomyopathy, arrhythmia, or heart failure (choice E). Amyloid proteins are notable for a strong reaction to the eosin part of H and E stain, making the amyloid deposits highly eosinophilic (not basophilic
An autosomal dominant trait showing 50 percent penetrance will be phenotypically expressed in what percent of the offspring?
A. 0
B. 25
C. 33
D. 50
E. 75
The correct answer is choice B. 25% Penetrance refers to the extent to which a given gene expresses itself. To illustrate, imagine a dominant gene for green hair, G. Both genotypes GG and Gg are expected to phenotypically have green hair. However, due to environmental factors, or due to the effect of other related genes, green hair may not always show phenotypically, even though the individual posseses the dominant G gene. If it actually shows in the appearance of 50% of the individuals who have the appropriate genotype, it is said to have 50% penetrance. The rest of the problem is math. If, for some reason, male parents in a large group express this green trait 50% of the time and female parents also express this green trait 50% of the time, we expect the offspring to express it 50% times 50% or 25% of the time. So the correct answer to question is choice B
An embolus originating as a thrombus in the femoral vein usually occludes a blood vessel in the
1. lung.
2. brain.
3. liver.
4. kidney
The correct answer is choice 1.LUNG The key here is to trace the path of a thrombus through the vascular system. If the thrombus or clot forms in the femoral vein, it will eventually continue to travel back in the venous system until it reaches the vena cava. It is thus entering larger and larger veins. It will then pass through the right atrium, right ventricle and into the pulmonary artery. The pulmonary artery will branch into smaller arteries and eventually capillaries in the lung. Somewhere along the way the thrombus will get caught in one of the small arteries or arterioles of the lung. So choice 1 is correct. Choices 2, 3 and 4 are all incorrect because the blood will pass through the lung and back again to the left side of the heart before it could reach the brain, liver or kidney. So again, the correct answer to question is choice 1
An increased serum acid phosphatase level is clinically significant and aids in the diagnosis of which of the following conditions?
1. Paget's disease of bone
2. Primary hyperparathyroidism
3. Secondary hyperparathyroidism
4. Breast carcinoma with bone metastasis
5. Prostatic carcinoma with bone metastasis
All five conditions listed involve bone pathology. However, elevated acid phosphatase will help distinguish prostatic metastatic carcinoma from the others. Acid phosphatase is produced by prostatic epithelium and is elevated in later, but not early, prostatic carcinoma. Choice 1 is incorrect. Paget's disease or osteitis deformans involves bone resorption and replacement with poorly mineralized tissue. In this disease, alkaline phosphatase, not acid phosphatase, is highly elevated. Choices 2 and 3 are incorrect. Primary hyperparathyroidism involves adenoma, hyperplasia or carcinoma of the parathyroid gland and is accompanied by elevated serum calcium and elevated parathyroid hormone. Secondary parathyroidism is often caused by chronic renal failure and vitamin D deficiency. It may be accompanied by lower serum calcium. Breast carcinoma with bone metastasis is not accompanied by elevated acid phosphatase. So the correct answer to question is choice 5
An individual that presents with keratoconjunctivitis, parotid gland enlargement and a positive Rh factor is most likely to be diagnosed with
A. Polyarteritis nodosa
B. Reiter's syndrome
C. Rheumatoid arthritis
D. Sjögren's syndrome
E. Systemic Lupus Erythematosus
The correct answer is D. Sjögren's syndrome is an autoimmune disorder that is characterized by a chronic dysfunction of exocrine glands. Individuals with this disorder often present with keratoconjunctivitis, parotid gland enlargement, dry mouth leading to a difficulty speaking and swallowing, loss of taste, and pleuritis. A positive Rheumatoid factor (Rh factor) is seen in over 70% of all patients with this disorder. Polyarteritis nodosa (choice A) is a disorder characterized by focal or segmental lesions of blood vessels. The pathological hallmark of this disease is acute necrotizing inflammation of the arterial media with extensive inflammatory cell infiltration of all layers of the vessels and surrounding tissue. Patients also present with fever, hypertension, abdominal pain, and elevated sedimentation rate. Reiter's syndrome (choice B) is the leading cause of arthritis in young adults, especially men. This disorder is commonly associated with a recent onset of arthritis, conjunctivitis and urethritis. Another common finding in this condition is keratoderma blennorrhagicum, which typically presents as an erythematous pustular, scaly and plaque-like set of lesions on the palms of the hands and soles of the feet. Rheumatoid arthritis (choice C) is a progressive disorder of unknown etiology. The characteristic feature of this disorder is the persistent inflammatory synovitis, usually involving the peripheral joints in a symmetric manner. Although a positive Rh factor is seen many individuals with the disorder, keratoconjunctivitis and parotid gland enlargement are not commonly seen. Systemic lupus erythematosus (choice E) is associated with arthralgias, fatigue, malaise, fever, and weight loss as well as the malar "butterfly" rash. This is an erythematous rash raised over the cheeks and bridge of the nose. Various renal and nervous system complications are also associated with this disorder.
An individual with Addison's disease is most likely to experience which of the following?
A. Decreased ACTH levels
B. Hypernatremia
C. Hypoglycemia
D. Water retention
E. Weight gain
The correct answer is C. Hypoglycemia. The symptoms of this disorder are primarily due to the destruction of the zona fasciculata by an autoimmune response of the body. In Addison's disease, individuals will normally experience a dramatic decrease in blood glucose levels several hours after a meal. In other words, individuals with this disorder may experience hypoglycemia. In this disorder, there are normally elevated ACTH levels (choice A). Furthermore, one would typically expect to see hyponatremia (choice B) and excessive water loss (choice D). One of the classic characteristics of Addison's disease is that patients often present with chronic weakness secondary to a continual weight loss (choice E).
An individual with mitral regurgitation will most likely suffer from which of the following?
A. Decreased heart rate
B. Fluid congestion in the lungs
C. Increased cardiac output from the ventricles
D. Peripheral edema
E. Ventricular arrhythmias
The correct answer is B. Fluid congestion in the lungs
. Mitral regurgitation is a clinical condition that occurs when the mitral valve leaflets are not able to close normally resulting in the "left ventricular" blood being ejected into the left atrium as well as through the aortic valve with each ventricular contraction. The net effect is an increased volume load on the left ventricle and an increased left atrial pressure. When the pressure in the left atrium increases, pulmonary edema will begin. As the amount of fluid in the lungs increases, the patient will begin to experience increasing dyspnea and fatigue. In mitral regurgitation, the heart rate (choice A) will either remain constant or slightly elevated in an effort to compensate for the extra "load" on the left ventricle. Since the net effect of mitral regurgitation is an increased volume load on the left ventricle and an increased left atrial pressure, cardiac output from the ventricles decreases (choice C). Peripheral edema (choice D) typically occurs when there is some degree of "right sided" heart problem. When the mitral regurgitation is chronic in nature, the patient is more likely to develop atrial arrhythmias, not ventricular arrhythmias (choice E).
An oral lesion that may appear as an ulcer, a nodule or a vegetative process and is often mistaken for squamous cell carcinoma is a manifestation of
1. candidiasis.
2. trichinosis.
3. sporotrichosis.
4. histoplasmosis
The correct answer is choice 4.
Histoplasmosis is a fungal disease caused by Histoplasma capsulatum. Infection occurs via the respiratory system and is often asymptomatic. In severe disseminated cases, ulcers of the nose, mouth, tongue and intestine can occur. The ulcers contain focal areas of necrosis in small granulomas. These ulcers may be confused with ulcerous areas of carcinomas. Choice 1 is incorrect. The lesions in candidiasis or thrush, another fungal infection, are whitish plaque-like areas. Choice 2 is incorrect. Trichinosis, a disease caused by flatworms, does not usually produce oral lesions. Choice 3 is incorrect. Sporotrichosis is a fungal disease usually traumatically introduced into the skin. The lesion is usually localized in the area of introduction and does not spread to the oral tissues. So the correct answer to question is choice 4
An Rh-negative mother delivered a normal first child. Her second child developed symptoms of erythroblastosis fetalis. Which of the following can be concluded?
1. The father is Rh-negative.
2. The first baby is Rh-negative.
3. The first baby is a girl; the second is a boy.
4. The mother has very high levels of serum complement and anti-Rh IgE.
5. None of the above
The correct answer is choice 5. NONE OF THE ABOVE. Erythroblastosis can occur when Rh negative mothers are carrying Rh positive fetuses; in particular, second Rh positive fetuses. The situation can only occur when the fetus is Rh positive due to the fact that the father is Rh positive. So choice 1 is incorrect. Choice 2 is also incorrect. Both children must be Rh positive as the first Rh positive baby sensitizes the mother's immune system to form anti-Rh antibodies and the second child causes a full and rapid antibody response. Choice 3 is incorrect as the sex of either the first or second child is irrelevant to this condition. Choice 4 is incorrect. Anti-Rh immunoglobulin would not be IgE because IgE is involved primarily in allergic reactions of skin and other tissues and binds to mast cells and basophils. It would not form as an anti-Rh antibody. So the correct answer to question is choice 5
Anemia found in iron deficiency is of which type?
A. Macrocytic
B. Microcytic
C. Normocytic
D. Hyperchromic
The correct answer is choice B. MICROCYTIC. Macrocytic (choice A) refers to larger than normal cells, normocytic (choice C) to normal size cells, and microcytic to undersized cells. Hyperchromic (choice D) is more colored than normal, normochromic is normal color, and hypochromic is less colored. The anemia in iron deficiency causes an inability to produce hemoglobin, and the resultant erythrocytes are smaller and less red than normal (microcytic, hypochromic). In pernicious anemia, they are macrocytic and are accompanied by hypersegmented neutrophils. In blood loss, cells are initially normocytic and normochromic but become macrocytic as the marrow falls behind in RBC replacement
Anthrax is caused by bacteria of the genus:
A. Neisseria
B. Clostridium
C. Bacillus
D. Corynebacteria
E. Mycobacteria
The correct answer is choice C. Anthrax is caused by Bacillus anthracis, a Gram-positive, spore-forming rod. B. anthracis is the major pathogen of the genus. Other Bacillus species include B. cereus and B. subtilis, which are soil organisms. B. cereus may sometimes grow on food and give off an exotoxin, causing a form of food poisoning. B. stearothermophilus should be familiar as the "spore test" or biologic monitor for autoclaves.
Antiseptics differ from disinfectants in that antiseptics
1. sterilize.
2. kill pathogens.
3. are bacteriostatic only.
4. are applied to viable tissues
The correct choice is choice 4. Antiseptics are chemical agents which are applied to tissues to inhibit the growth of pathogenic bacteria or to kill pathogenic bacteria. Examples of antiseptics include betadine, alcohol and various soaps. Disinfectants are also chemical agents, but they are generally more caustic than antiseptics. They kill cells indiscriminstely and may cause severe damage to viable tissues. Examples of disinfectants include alkylating agents, oxidants and sulfhydryl reagents. Choice 1 is incorrect because antiseptics and disinfectants do not sterilize tissues or surfaces. Sterilization occurs when all microorganisms are destroyed. Antiseptics and disinfectants reduce the infectivity of matter, but nonpathogenic organisms may remain viable. Choice 2 is incorrect because antiseptics are less bacteriocidal than disinfectants. In fact, disinfectants are defined as those agents which are bacteriocidal at low concentrations. The bacteriocidal properties of antiseptics make choice 3 incorrect. Antiseptics are bacteriostatic, but they are not exclusively bacteriostatic. They can be bacteriocidal as well. Once again, the correct choice for question is choice 4.
As a result of dental prophylaxis, microorganisms around teeth enter the bloodstream. This condition is an example of
1. pyemia.
2. toxemia.
3. bacteremia.
4. septicemia.
5. focal infection
The correct choice is choice 3, bacteremia. Bacteremia literally means bacteria in the blood stream. It can be caused by a variety of conditions. Bacteremia occurs, for example, when pneumococcal pneumonia leads to pneumococcal meningitis. Here, some bacteria from the lung enter the blood stream, causing bacteremia. The organisms are canied by the blood to the meninges where they cause a severe infection. Bacteremia also occurs following dental procedures, including routine cleaning. Patients with valvular heart disease are encouraged to take prophylactic antibiotics prior to dental procedures to protect their valves from the bacteremia induced by the dentist. In septicemia, choice 4, organisms can also be cultured from the blood stream. Septicemia differs from bacteremia in that the former is associated with hypotension, increased cardiac output and decreased peripheral resistance. Septicemia is a life-threatening condition and requires IV antibiotics immediately. Bacteremia rarely requires treatment. Pyemia, choice 1, literally means pus in the blood. Pyemia is usually associated with septicemia or bacteremia. It rarely occurs in sterile blood. Toxemia, choice 2, occurs when a poison circulates in the blood. Examples of toxemia include botulism in which botulism toxins circulate or the toxic shock syndrome in which toxins produced by Staphylococci circulate. Focal infection, choice 5, is a condition wherein bacteria can be isolated from only one site. Once bacteria enter the blood stream, the infection becomes diffuse. Once again, the correct choice to question is choice
Bacille Calmette Guerin (BCG) is best described as a(n):
A. antibiotic
B. vaccine
C. antitoxin
D. tuberculin test
E. passive immunity
The correct answer is choice B. VACCINE BCG is also known as Bacille Calmette Guerin, and is an attenuated-related mycobacterium. It has been given to many tuberculin-negative individuals around the world and results in a variable resistance to tuberculosis (TB). People vaccinated with BCG develop antibodies that are sometimes effective against TB. Therefore, BCG is best described as a vaccine. Since the patient forms his/her own antibodies, it is active, not passive, immunity (choice E). It is not an antibiotic (choice A), as it does not kill bacteria. It is not an antitoxin (choice C), as it does not counteract a toxin formed by mycobacteria. It is not the tuberculin test (choice C), which is accomplished using PPD (purified protein derivative) derived from mycobacterial cells
Bacteremias in patients with heart valve abnormalities may result in
1. scarlet fever.
2. rheumatic fever.
3. Ludwig's angina.
4. infective endocarditis.
5. streptococcal pharyngitis
The correct answer is choice 4. Infective endocarditis is a bacterial infection of the lining tissues of the heart including the heart valves. It is caused by Streptococci or Pneumococci. Valves with irregularities of structure or function are more susceptible to being colonized by bacteria which sometimes enter the bloodstream. A large group of bacteria traveling in the bloodstream is known as a bacterernia and a hacterernia can be caused by dental treatment including prophylaxis. scaling and root planning. This fact leads to the necessity for antibiotic prophylaxis for dental patients with heart valve abnormalities. Choice 1 is incorrect. Scarlet fever is an infection caused by group A beta hemolytic Streptococci. Choice 2 is also incorrect. Rheumatic fever is a disease also caused by hcrnolytic Streptococci. It can lead to heart valve damage which later can make a patient more susceptible to endocarditis. Choice 3 is incorrect. Ludwig's angina is a massive inflammation and cellulitis especially of the mandibular and pharyngeal areas which can lead to blockage of the air passages. It is a possible sequela of Streptococcal infection, choice 5. Streptococcal pharyngitis is sore throat caused by Streptococci. So the correct answer to question is choice 4.
Bacteria with high lipid content in their cell walls include all of the following EXCEPT:
A. corynebacteria
B. mycobacteria
C. nocardia
D. bacteroides
The correct answer is choice D. Bacteroides are a genus of Gram-negative, anaerobic bacteria, including B. fragilis, B. melaninogenicus, and B. gingivalis. B. gingivalis and B. melaninogenicus are common findings in diseased periodontal pockets. Bacteroides is not known for an especially high lipid content of the cell wall. The other bacteria listed all contain a high level of lipids in their walls. Mycobacterium (choice B) contains many waxes, fats, and mycolic acids, and this fatty wall prevents easy entrance of antibiotics into the cells. Nocardia (choice C) are closely related to Mycobacteria and share the same mycolic acid based cell wall structure. The two genera are known as the acid-fast bacteria. Corynebacteria (choice A) include C. diphtheriae, the causative agent of diphtheria. They are Gram-positive aerobes. Their cell wall is notable for having a high lipid and wax content, although they are not acid fast
Bacterial capsules usually consist of
1. chitins.
2. celluloses.
3. polypeptides.
4. polysaccharides
The correct answer is choice 4. polysaccharides Bacterial capsules are layers surrounding the cell wall. They are usually composed of polysaccharides, choice 4, and are rarely composed of polypeptides, choice 3. So in terms of the most usual structure, the correct answer is choice 4. The bacterial cell wall varies in structure in various types of bacteria but may contain polypeptides, lipoproteins, polysaccharides and other materials. Choice 1 is incorrect. Chitins are found in cell walls of fungi and in exoskeletons of insects. Choice 2 is incorrect. Celluloses are primarily found in plant cell walls and some related polysaccharides as mentioned are found in bacterial cell walls. So the correct answer to question is choice 4.
Bacterial endotoxins may play a role in development of periodontal disease due to
1. high amounts of endotoxin in plaque.
2. the ability of endotoxins to incite an inflammatory response.
3. the fact that endotoxins are similar to ground substance.
4. the presence of endotoxins in both gram-positive and gram-negative bacteria.
5. none of the above
The correct choice is choice 2, the ability of endotoxins to incite an inflammatory response. Endotoxins are lipid moieties found in the outer membrane of gram-negative bacteria. They are not found in gram-positive organisms; therefore, choice 4 is incorrect. Endotoxins may incite local inflammatory responses, but they are generally of greater pathologic significance when circulating in the blood stream. They cause systemic effects of vasodilatation and vascular leakage which ultimately lead to shock. They also increase clotting and may induce disseminated intravascular coagulation. Their role in periodontal disease stems from their ability to activate complement and attract neutrophils and macrophages. Endotoxin is not present in high amounts in plaque as stated in choice 1. Plaque is a complex of primarily gram-positive organisms and dextran. As stated, gram-positive organisms lack endotoxin, though some gram-positive organisms are capable of causing a clinical syndrome which looks very much like endotoxic shock. Ground substance, mentioned in choice 4, is not similar to endotoxin. Ground substance is the intracellular cement made up of hyaluronic acid and other mucopolysaccharides. Endotoxin is composed of lipopolysaccharide. Once again, the correct choice to question is choice 2
Bacterial flagella are:
A. similar in structure to eukaryotic flagella
B. composed of a 9+2 filament arrangement
C. composed of microtubules
D. composed entirely of protein
The correct answer is choice D. composed entirely of protein. Several differences exist between prokaryotic cells, including bacteria and eukaryotic cells. Among them is the structure of the flagellum, which is the major organ of locomotion for many bacterial species. The standard eukaryotic (advanced cell, NOT bacterial cell) flagellum consists of a microtubular structure with nine pairs of tubulin microtubules surrounding an inner core of two single microtubules. This is the familiar "9+2" arrangement. The bacterial (prokaryotic) cell does not share this structure, having only a protein-based flagellum without the 9+2 structure. Large differences also exist between eukaryotic and prokaryotic nuclei, mitochondria, and ribosomes
Bence-Jones proteins are tumor markers for which of the following types of cancer?
A. Hepatocellular
B. Myeloma
C. Ovarian
D. Pancreatic
E. Prostate
The correct answer is B.Myeloma. Tumor markers can be used to aid in the screening, diagnosis and monitoring of an individuals response to the treatment of the cancer. Furthermore, tumor markers can aid in the "staging" of an individuals cancer. Bence-Jones proteins can be used to diagnose and help in the prognostic indications for multiple myeloma. Hepatocellular (choice A) carcinoma can be monitored with the detection of alpha-fetoprotein. Ovarian cancer (choice C) is monitored by the presence of CA 125; a high level of this tumor marker is closely related to a poor prognosis. Pancreatic (choice D), colorectal and gastric carcinoma can be followed by the presence of CA 19-9. Prostate cancer (choice E) is monitored by the presence of PSA (prostate-specific antigen
Beta-thalassemia results in:
A. microcytic cells with excess beta hemoglobin
B. microcytic cells with insufficient beta hemoglobin
C. macrocytic cells with excess alpha hemoglobin
D. macrocytic cells with insufficient alpha hemoglobin
The correct answer is choice B.microcytic cells with insufficient beta hemoglobin
In the thalassemias, insufficient amounts of one hemoglobin chain are produced. In beta thalassemia, insufficient beta chain is produced, resulting in a microcytic cell filled with excess alpha chain. The alpha chains can aggregate, become insoluble, and lead to hemolysis and anemia. Patients homozygous for this disease or who are transfusion-dependent are known as B-thal major
Biological monitors, used for autoclave testing, are composed of:
A. Viral spores
B. Mycobacterium tuberculosis
C. Live bacteria
D. Bacterial spores
E. A combination of viruses, fungi and bacteria
The correct answer is choice D. BACTERIAL Spores, such as those of Bacillus stearothermophilus, are especially heat tolerant. If they are killed by some sterilizing process, then you can be sure that all other microorganisms are killed as well. These spores are sometimes referred to as "benchmark" organisms for sterilization. Because of this property, we do not need to test our autoclave with a combination of different organisms. Note that there are no viral spores. Note also that Mycobacterium tuberculosis is the "benchmark" organism for surface disinfectants
Blood in the sputum is characteristic of each of the following EXCEPT one. Which one is this EXCEPTION?
A. Emphysema
B. Tuberculosis
C. Lobar pneumonia
D. Pulmonary embolism
E. Bronchogenic carcinoma
The correct answer is choice A. Emphysema In each case we will try to figure a source for blood in the sputum. In the case of tuberculosis, there is an active infection by Mycobacterium tuberculosis with accompanying fever, anorexia, necrosis of lung tissue and destruction of blood vessels in lung parenchyma. Similarly, in lobar pneumonia, an infection is present which can erode surrounding blood vessels. In a pulmonary embolism, a thrombus, most likely from a leg vein, becomes lodged in the lung. If the embolus becomes infected (septic embolus), then the situation is similar to that just mentioned. A bronchogenic carcinoma can cause bleeding as it expands into nearby vascular tissue. Emphysema is usually not accompanied by bleeding. There is collapse of alveolar walls with loss of diffusional surface area, cough, dyspnea and barreled chest. So the correct answer to question is choice A
Both heat and alcohol damage bacteria primarily by:
A. denaturing protein
B. alkylating DNA
C. oxidizing sulfhydryl groups
D. dissolving fatty cell membranes
The correct answer is choice AA. denaturing protein
. All of the processes listed can be used to kill or inactivate microorganisms. Alcohol and heat (dry or steam) primarily act as protein denaturants. Dry heat can kill by desiccation ethylene oxide and glutaraldehyde alkylate nucleic acids. Peroxides, chlorine, iodides, and mercury based compounds act on sulfhydryl groups. Detergents can damage fatty parts of cell membranes
By definition, which of the following antibacterials specifically retards the growth of organisms eventually leading to the death of the organism?
A. Acyclovir
B. Cefixime
C. Clarithromycin
D. Levofloxacin
E. Penicillin
The correct answer is C.Clarithromycin This question is essentially asking, "Which of the following antibiotics are bacteriostatic?". As a general rule, all antibiotics can be classified as either a bacteriostatic or bactericidal agent. Bacteriostatic antibiotics inhibit or retard the growth of bacteria, which will eventually lead to the death of the organisms. Bactericidal antibiotics directly cause the death of bacteria. Antibiotics are generally placed in "classes" according to their mechanism of action. For example, clarithromycin (choice C), a macrolide antibiotic, is a bacteriostatic agent that binds to the 50S ribosomal subunit of susceptible organisms and inhibits RNA-dependent protein synthesis. Acyclovir (choice A) is an antiviral agent commonly used in the treatment of herpes virus infections. Cefixime (choice B) is a bactericidal cephalosporin antibiotic that interferes with bacterial cell wall synthesis. Levofloxacin (choice B) is a bactericidal Fluroquinolone antibiotic that inhibits bacterial DNA gyrase leading to the death of the organism. Penicillin (choice E) is a bactericidal penicillin antibiotic that interferes with bacterial cell wall synthesis
Cervicofacial actinomycosis is usually due to
1. poor aseptic technique during oral surgery.
2. contamination of a trauma or surgical site with spores.
3. contamination of a trauma or surgical site with endogenous organisms.
4. spread of the organisms from a dermal or thoracic infection
The correct answer is Choice 3.contamination of a trauma or surgical site with endogenous organisms. Actinomycosis is a chronic suppurative disease that forms draining sinus tracts and is caused by Actinomyces israelii and related filamentous bacteria. They are part of the normal flora of the oral cavity and why they begin to invade tissue is not clear. However, it is believed that trauma, such as tooth extraction, necrotizing bacterial infection or aspiration, may cause clinical actinomycosis. Typical lesions may drain to the outside through sinuses and contain so-called sulphur granules of the central mycelium and filaments. So choice 3 is correct as it involves trauma and endogenous organisms or normal flora. Choice 2 is incorrect as Actinomyces does not form spores. Choice 1 is incorrect as the source of Actinomyces is the endogenous flora and not bacteria that might be present from outside sources and spread through aseptic technique. Choice 4 is incorrect. Although infections may often drain to the face, the source of the bacteria is the oral cavity and not the skin of the face or thorax. So the correct answer to question is choice 3.
Chronic passive congestion of the lung is characterized by
1. hyaline membranes.
2. chronic bronchitis.
3. giant cell arteritis.
4. interstitial infiltration of PMN's.
5. edema of alveolar walls and "heart failure" cells
The correct answer is choice 5. edema of alveolar walls and "heart failure" cells
Pulmonary congestion involves elevated hydrostatic or lowered colloidal blood pressure, increased capillary permeability, decreased lymphocytic drainage, edema, and increased sodium retention. It is often associated with heart failure or shock. Hemosiderin-filled macrophages are often found in alveolar spaces and are the so-called heart failure cells. Choice 1, hyaline membranes, are often associated with viral or mycoplasmal pneumonia, but not chronic passive congestion. Choice 2, chronic bronchitis, is a chronic obstructive disease and may lead to pulmonary emphysema and other pulmonary problems, but not chronic passive congestion. Choice 3, giant cell arteritis, is a granulomatous inflammation of small- and medium-sized vessels, especially cranial vessels. Choice 4 is incorrect. Interstitial infiltration of PMNs occurs often with bacterial pneumonia, but not with chronic passive congestion. So the correct answer to question is choice 5
Classic hemophilia is due to a deficiency of normal Factor VIII which in turn is due to a genetic deficiency that is
1. partially dominant.
2. autosomal dominant.
3. autosomal recessive.
4. sex-linked dominant.
5. sex-linked recessive
The correct answer is choice 5. sex-linked recessive Hemophilia A or classic hemophilia is a deficiency of Factor VIII-C, leading to a secondary hemostasis deficit. Factor VIII-C is a clotting protein and, in classic hemophilia, it can either not be produced from its precursor or, a when produced, it is defective. Hemophilia A is inherited as a sex-linked recessive trait; that is, it is carried on the X chromosome and transmitted primarily from carrier heterozygous mothers to sons. So the correct answer to question is choice 5
Corynebacterium diptheriae is often associated with tonsillopharyngeal infections; this bacteria can be best classified as a(n)
A. anaerobic bacteria
B. gram negative rod
C. gram positive cocci
D. gram positive rod
E. virus
Corynebacterium diptheriae is an aerobic, nonmotile, non-sporulating, irregularly staining gram-positive rod. Infections of the respiratory tract are most often tonsillopharyngeal and may also be laryngeal and nasal. In tonsillopharyngeal infections, this bacteria may initially lead to erythema. However, spots of gray or white exudate are common. Furthermore, these "spots" often extend and coalesce within a day to form confluent sharply demarcated pseudomembranous lesions.
Crohn disease most commonly involves the:
A. duodenum and jejunum
B. jejunum and ileum
C. ileum and colon
D. colon and rectum
The correct answer is choice C. ILEUM AND COLON. Crohn disease is a recurrent granulomatous inflammatory disease of the bowel, most commonly affecting the terminal ileum and colon. The etiology is not clear, and infection, autoimmune, nutritional, and hormonal theories have all been considered. It has a remitting/relapsing course and is associated with a higher than normal risk of small intestine adenocarcinoma. It is more common in people of Jewish ancestry and usually begins in early adulthood. Pathologically, skip lesions are seen-areas of involvement alternating with segments of normal tissue
Detergents kill bacteria by interfering with the function of the cell
1. wall.
2. capsule.
3. membrane.
4. ribosome.
5. chromosome
The correct answer is choice 3.membrane This is a basic cell organelle structure and property question. Detergents have the property of emulsifying and dissolving fats and lipids. Therefore, they will kill bacteria by dissolving any organelles which contain significant amounts of lipid. So cell membrane, choice 3, is correct. The cell membrane is primarily lipid and phospholipid, with a smaller amount of protein. Choice 1 is incorrect. Bacterial cell walls are composed of lipopolysaccharide, peptidoglycan or lipoprotein, but not lipid. Choice 2 is incorrect. Bacterial capsules are composed of extracellular polymers and it is almost always a polysaccharide type substance. Choice 4 is incorrect. Cell ribosomes are composed of RNA. Choice 5 is incorrect. Chromosomes are composed of DNA. So the correct answer to question is choice 3.
Development of pseudomembranous colitis is a major adverse effect of prolonged therapy with
1. vancomycin.
2. clindamycin.
3. tetracycline.
4. streptomycin.
5. cephalosporin.
The correct choice is choice 2, clindamycin. Pseudomembranous colitis is caused by a toxin secreted by Clostridium difficile. The disease presents as watery diarrhea, but dysentery or bloody diarrhea occurs in a small percentage. On sigmoidoscopy, yellow plaques are found lining the colon. Histologically, a pseudomembrane consisting of acute inflammatory cells and fibrin can be identified. Many antibiotics can cause this disorder, but it is especially common following prolonged clindamycin treatment which upsets the normal colonic flora. Vancomycin, choice 1, is in fact the treatment, not the cause, of pseudomembranous colitis. It must be given orally to be efficacious. The major side effects of vancomycin include renal failure and deafness. Tetracycline, choice 3, has adverse effects including hepatotoxicity and photosensitivity. It rarely causes pseudomembranous colitis. Streptomycin, choice 4, is an aminoglycoside. Like all aminoglycoside, it almost never causes pseudomembranous colitis. It can, however, produce pancytopenia and anaphylaxis. The cephalosporins, choice 5, can cause anaphylaxis in penicillin-sensitive patients. In addition, these drugs may be associated with hemolytic anemias and elevations of liver enzymes. Pseudomembranous colitis is rare with these agents. Once again, the correct choice to question is choice 2
Difficulties in controlling mycobacterial growth with antibiotics are due to all of the following EXCEPT:
A. mycolic acid is present in cell walls
B. mycobacteria are quick growing and dividing
C. mycobacteria are intracellular parasites
D. mycobacteria may be found in Ghon complexes
The correct answer is choice B. Mycobacteria are notoriously difficult to kill with antibiotics. They possess waxy, fatty cell walls containing mycolic acids. These walls prevent easy uptake of antibiotics into the cell. Furthermore, the cells are often clustered in Ghon complexes of live and dead mycobacterial cells, leukocytes, and other inflammatory cells. It is also difficult for the antibiotics to penetrate the Ghon complex or walled-off tubercle. The bacteria also are found inside of the monocytes or giant cells that engulfed them. Antibiotics must pass into these cells to reach the mycobacteria. Mycobacteria also grow and divide extremely slowly, so antibiotics that depend on the growth and metabolism of the bacterial cells will act only very slowly. As a result, the course of therapy for tuberculosis is usually long (6 to 12 months
Disseminated miliary tuberculosis results from spread of the tubercle bacillus by way of
1. the lymphatics.
2. the bloodstream.
3. the air passages.
4. direct extension.
5. none of these
The correct answer is choice 2. THE BLOODSTREAM Miliary tuberculosis refers to a widespread dissemination of the causative agent Mycobacterium tuberculosis to a number of organs and tissues. This is accomplished primarily through the bloodstream and leads to a high mortality rate. Of course, the most commonly involved organ in tuberculosis is the lung. However, other organs including the urinary tract and meninges are often infected. Likewise, tuberculosis can spread through the lymphatics or by direct extension from one organ to an adjacent organ. However, miliary or widespread tuberculosis is caused by spread through the circulatory system. So the correct answer to question 22 is choice 2.
Dry heat destroys organisms primarily by:
A. Lysis
B. Oxidation
C. Precipitation of salts
D. Dissolving of cell membranes
E. Denaturation of proteins
The correct answer is choice E. The two major effects of dry heat on microorganisms are denaturation of proteins and dehydration. Proteins lose their normal configuration and shape after a certain temperature. In the case of enzymes, these proteins are no longer functional and the organism will die. Dehydration removes all water, which is necessary for cellular life. Lysis is a general term for disruption of a cell membrane and leakage of cell contents. Detergents may dissolve the fatty part of membranes, causing lysis. Many disinfectants, such as glutaraldehyde and ethylene oxide, act by alkylation of DNA molecules. Some disinfectants, such as iodine compounds and peroxide act by oxidation of protein sulfhydryl groups.
Each of the following characterizes a chlamydial infection EXCEPT one. Which one is this EXCEPTION?
A. Large numbers of asymptomatic carriers
B. Frequent co-infection with gonorrhea
C. The ability of the organism to survive in the host extracellularly
D. The greater likelihood that younger women will acquire salpingitis
The correct answer is choice C. Chlamydia1 infections are NOT characterized by the ability of the organism to survive in the host extracellularly. Chlamydiae are obligate intracellular parasites that infect birds and mammals. Notice the answer choices all display clinical characteristics of chlamydial infections which are true except choice C which is a morphological feature. This represents an important test-taking strategy particularly dealing with "EXCEPT" questions: One of these is not like the others. When answering EXCEPT questions realize that all of the answer choices will have something in common except for the correct choice which will not be like the others. You need to look for any relationships between the answer choices and find the one that does not fit. In this case choice C is not a clinical charateristic like the other choices, therefore if you did not know the correct answer to this question choice C would be a good guess. Anyway, choice A - large numbers of assymptomatic carriers, choice B - frequent co-infection with gonorrhea, and choice D - the greater likelihood that younger women will acquire salpingiitis, are all characteristics of a chlamydial infection and cannot be the answer. Therefore the correct answer to question is C
Each of the following is a histologic feature of malignant growth EXCEPT one. Which one is this EXCEPTION?
A. Aplasia
B. Anaplasia
C. Pleomorphism
D. Hyperchromatism
E. Abnormal mitosis
The correct answer is choice A. Aplasia refers to a lack of cell growth or proliferation. Neoplasms are generally areas of uncontrolled growth and proliferation. All four other choices are associated with tumor growih. Anaplasia is loss of cell differentiation and tissue organization. Pleomorphism is wide variety of size and shape of nuclei and cells. Hyperchromatism is intense color and staining of nuclear material. Tumors also have abnormal, frequent and rapid mitoses. So the correct answer to question is choice A
Each of the following is a risk factor in atherosclerosis EXCEPT one. Which one is this EXCEPTION?
A. Heredity
B. Alcoholism
C. Hypertension
D. Diabetes mellitus
E. Hyperlipoproteinemia
The correct answer is choice B. ALCOHOLISM Atherosclerosis refers to the narrowing of arteries due to the accumulation of fatty plaque along the arterial walls. A tendency for atherosclerosis can be inherited. In particular, the tendency toward hyperlipidemia (especially low density lipoproteins). This hyperlipidemia (hyperlipoproteinemia) can also be exacerbated by high fat diet, especially saturated fat. Hypertension, or high blood pressure, also is linked to a faster development and greater degree of atherosclerosis, in particular DIASTOLIC pressure. In diabetes, hyaline material may be deposited on aterial walls, reducing blood flow. Other risk factors include increasing age, cigarette smoking, obesity and sedentary lifestyle. Alcoholism is NOT a risk factor. So the correct answer is choice B
Ectopic Cushing syndrome is usually caused by:
A. an adenoma of the adrenal gland
B. a carcinoma of the adrenal gland
C. an adenoma of the anterior pituitary
D. exogenous administration of cortisol
E. secretion by bronchogenic carcinomas and pancreatic neoplasms
The correct answer is choice E. E. secretion by bronchogenic carcinomas and pancreatic neoplasms The answer can be correctly determined merely from the term ectopic, which refers to another, or unusual, area of the body. Tumors (usually adenomas) of the anterior pituitary (affecting ACTH) (choice C) or of the adrenal cortex (affecting cortisol release) are both direct causes of Cushing's syndrome (excess cortisol). The ectopic source of excess cortisol is that coming from an unusual source and place. Some lung and pancreas neoplasms can produce and secrete cortisol, and these tumors cause ectopic Cushing's syndrome. Iatrogenic Cushing syndrome is caused by administration of cortisol (choice D) to a patient, as a pharmaceutical agent (antiinflammatory, etc.)
Esophageal varices are a common complication of
1. portal hypertension.
2. primary hypertension.
3. mesenteric thrombosis.
4. carcinoma of the esophagus
The correct answer is choice 1. portal hypertension.
Esophageal varices are distended veins in the esophagus. Through chronic liver damage such as that in alcoholic cirrhosis, portal circulation may be impeded and alternate channels bypassing the liver may become engorged with blood. Specifically, the portal blood drains by way of coronary veins of the stomach into esophageal veins and then into the azygous system of veins. Esophageal varices can lead to life threatening bleeding if damaged. Choice 2 is incorrect. Primary hypertension refers to high blood pressure of unknown etiology with readings of greater than 140/90. It does not lead to esophageal varices. Choice 3 is incorrect. Mesenteric thrombosis refers to a thrombosis or solidified blood clot forming in a mesenteric vessel. Thrombi may form emboli which are thrombi which travel to other vessels and lodge in those vessels. However, they will not cause esophageal varices. Choice 4 is incorrect. Carcinoma of the esophagus may lead to bleeding within the esophagus but this bleeding would not be hccause of esophageal varices. So the correct answer to question is choice 1.
Ethylene oxide sterilization procedures usually require exposure times of:
A. 20-30 minutes
B. 1-2 hours
C. 2-3 hours
D. 8-10 hours
E. 24 hours
The correct answer is choice D. D. 8-10 hours
Ethylene oxide is used for sterilization of heat sensitive instruments and materials. Its advantage is that it does not melt, damage or corrode instruments, as heat may. Its disadvantages include long cycle time (8-10 hours) and additional time to air out the materials so that they do not contain ethylene oxide residue. 20-30 minutes is a common steam autoclave time. 1-2 hours is a common dry heat oven time. 24 hours was a time recommended for "cold sterilization" by chemicals. Cold sterilization is a misnomer, as it is actually high level disinfection
Factors that predispose susceptible erythrocytes to sickling include all of the following EXCEPT:
A. high glucose
B. low oxygen
C. low pH
D. dehydration
The correct answer is choice A. NOT GLUCOSE Sickle cell disease is caused by one amino acid mutation (valine substitution for glutamine) in the hemoglobin chain. The resulting hemoglobin S chain is susceptible to deforming (sickling). Factors that increase sickling tendency include low oxygen (choice B), higher levels of hemoglobin S (as opposed to hemoglobin A), low pH (choice C), and dehydration (choice D). Glucose level does not affect this tendency
Fat embolism is most often a sequela of
1. fracture.
2. infection.
3. contusion.
4. infarction.
5. deep laceration
The correct choice is choice 1, fracture. Fat embolism occurs in almost all patients who suffer severe trauma with multiple fractures. It commonly presents clinically as a sudden change in mental status and shortness of breath. Renal failure may follow. When single bones are fractured without concomitant severe trauma, fat embolism occurs most often with hip fractures. Femur fracture rnay also cause fat embolism, but more rarely. Various medical conditions are associated with fat embolism due to 10: abnormalities in fat physiology. These conditions include diabetes mellitus, pancreatitis and alcoholism. When significant fat embolism occurs, regardless of the cause, the prttient cames a grave prognosis. Infection (choice 2), contusion (choice 3), infarction (choice 4) and deep laceration (choice 5) are not associated with fat embolism unless these conditions lead to pathologic fracture of a long bone. Once again, the correct choice for question is choice
Following the treatment of an oral infection with clindamycin, a patient develops antibiotic associated colitis. This infection is most commonly caused by
A. Clostridium difficile
B. Closrridium novyi
C. Clostridium perfringens
D. Clostridium ramosum
E. Closrridium septicum
The correct answer is A. Antibiotic induced colitis (Pseudomembranous Colitis) is characterized by severe persistent diarrhea, severe abdominal cramps and is caused by the toxin produced by Clostridia difficile (choice A). This choice is generally seen towards the end of a therapy; however, it may begin up to several weeks following discontinuation of therapy. The treatment of this form of colitis is to discontinue medication, provide fluid and electrolyte replacement, corticosteroids (systemic and/or via enema) as well as vancomycin or metronidazole. Clostridium perfringens (choice C) is the bacteria most commonly associated with the development of clostridial myonecrosis, otherwise known as gas-gangrene. This bacteria is also commonly associated with the development of food poisoning. Furthermore, this bacteria, as well as Clostridium novyi (choice B) and Clostridium septicum (choice E), are known to cause Clostridial tissue infections and bacteremias. Clostridium ramosum (choice D) may be found as part of the normal colonic flora in many individuals.
For the majority of individuals, the initial infection with herpes simplex virus results in
1. encephalitis as a young adult.
2. a dermal rash in childhood.
3. herpes labialis in puberty.
4. a subclinical disease.
5. genital herpes
The correct choice is choice 4, a subclinical disease. Most individuals are infected with the herpes simplex virus by the time they are two years old. In 90% of those infected, the initial disease is asymptomatic or subclinical. In the remaining 10%, gingivostomatitis develops. This is characterized by multiple vesicle formation over the oral mucosa and lips. Occasionally the nasal cavity and cornea are affected as well. Very rarely, the primary infection may include encephalitis or a diffuse skin rash. Encephalitis does occur in young adults as stated in choice 1, but this does not usually result from primary infection. Rather, it usually occurs when latent herpes simplex virus infection is reactivated. Choice 2 is incorrect because, as stated, dermal rash is actually a rare complication of primary infection. It occurs in less than 1% of infected individuals. Herpes labialis, choice 3, is the hallmark of reactivated herpes, not of primary infection. This lesion is more commonly called a fever blister. It is a vesicular eruption on the lip which occurs when latent herpes simplex virus particles in the trigeminal ganglia resurface. Genital herpes, choice 5, is caused by a different herpes simplex virus than the one infecting children. Genital herpes is caused by herpes simplex virus II, whereas young children acquire herpes simplex virus I. Herpes simplex virus II is transmitted by sexual contact and it causes a recurrent, painful vesicular lesion on the genitalia. Infection with this virus is far less pervasive than infection with herpes simplex virus I and, consequently, genital herpes is not the most common manifestation of herpes simplex virus infection. Once again, the correct choice to question is choice 4.
Generally, resistance of a bacterium to killing by heat is influenced by time and by temperature employed in growth of the culture. Another factor is the ability of the organism to
1. form spores.
2. withstand acid pH.
3. produce protopiasts.
4. undergo transformation.
5. grow at elevated temperatures
The correct answer is choice 1. Spore formation is employed by certain bacterial species as protection against harsh conditions, including elevated temperature. The spores known as endospores are formed primarily by genus Bacillus and genus Clostridium. Spores may also be formed under poor nutritional conditions such as depletion of the nitrogen or carbon source. Choice 2 is incorrect. The ability to withstand low pH is not directly connected to the ability to withstand high temperature. Choice 3 is incorrect. Protoplasts are wall-defective microbial forms or, basically, gram-positive bacteria without walls. They are especially a fragile and their formation would not aid in the temperature resistance of the organism. Choice 4 is incorrect. In transformation, recipient bacteria take up DNA fragments released from other donor bacterial cells. Transformation does not normally aid the bacteria to resist being killed by high temperature. Choice 5 is incorrect. Most bacterial organisms grow faster at elevated temperatures up to a given point. This property does not cause them to resist elevated temperature in the way that formation of a protective endospore would. So the correct answer to question is choice 1.
Generally, the antibiotic of choice for prophylactic therapy covering dental procedures in a patient with a heart valve abnormality who is allergic to penicillin is
1. ampicillin.
2. lincomycin.
3. tetracycline.
4. streptomycin.
5. erythromycin
The correct answer is choice 5. erythromycin Antibiotic prophylaxis for dental procedures for patients with heart murmur, rheumatic heart disease or other valvular problems usually consists of two grams of oral penicillin one hour before the procedure and one gram six hours after the procedure. In the penicillin-allergic patient, erythromycin is the drug of choice, with one gram given one hour before the procedure and 500 mg six hours after. It should be noted in the choices that choice 1, ampicillin, would never be given to the penicillin-allergic patient as it is a form of penicillin. Choice 2, lincomycin, is a bacterial protein synthesis inhibitor and is similar in mode of action to erythromycin. However, it is not used as an antibiotic prophylaxis. Likewise, choices 3 and 4, tetracycline and streptomycin, are bacterial protein synthesis inhibitors, but are not used in this way for prophylactic therapy. So the correct answer to question is choice 5.
Gout results from a metabolic defect in which of the following?
A. Fat
B. Purine
C. Pigment
D. Glucose
E. Calcium
The correct answer is choice B. In gout, the normal breakdown of purines results in the formation of uric acid. Purines are generally difficult to degrade, and excretion involves, primarily, making the purine more soluble so that it can be excreted. Uric acid is this more soluble purine ring form. Excess uric acid, in the form of sodium urate crystals, develops in gout, and these crystals collect in the synovial capsules of joints of the lower extremity, especially the big toe. Excess urate can be caused by excess production, insufficient excretion or some inborn error of purine metabolism. So the correct answer to question is choice B.
Hashimoto thyroiditis is associated with all of the following EXCEPT:
A. goiter
B. nervousness and heat sensitivity
C. plasma cell infiltrate
D. increased TSH
E. association with autoimmune diseases
The correct answer is choice B. NOT nervousness and heat sensitivity
. Hashimoto thyroiditis is probably autoimmune-related and is more common in patients with other autoimmune diseases (choice E). Histologically, lymphocytic and plasma cell infiltrates (choice C) are noted. The gland enlarges (goiter) (choice A) but produces little thyroxin due to damage to the secreting cells. and so all symptoms are those of hypothyroidism. Therefore, the patient is often cold and lethargic, rather than hot and nervous. Note that goiter can be produced by many causes, not only iodine deficiency. TSH levels are high (choice D), as the anterior pituitary senses low thyroxin and secretes TSH to stimulate the thyroid gland
Hemophilia B is an X-linked recessive disorder where there is a deficiency of which of the following substances?
A. Factor VIII antigen
B. Factor VIII coagulant
C. Factor IX coagulant
D. Fibrinogen
E. Vitamin K
The correct answer is C. Factor IX coagulant
. Hemophilia B (Christmas disease, factor IX hemophilia) is a hereditary bleeding disorder caused by a deficiency of coagulation factor IX; this disorder is also known as "Christmas disease." Individuals with this disorder will typically have a prolonged PTT. A deficiency of the factor VIII antigen (choice A) is typically seen in von Willebrand's disease. This disease is the most common congenital disorder of homeostasis; it is characterized by a deficient or defective von Willebrand factor (vWF). Although factor VIII antigen levels are decreased, the coagulation factor VIII levels are generally normal. Hemophilia A is an X-linked recessive disorder where factor VIII coagulant levels are decreased (choice B) and factor VIII antigen levels are normal. Hemophilia A is the "classic type of hemophilia" affecting 1 in 10,000 males. The bleeding tendency of this disorder is related to the deficiency of the factor VIlI coagulant. Afibrinogenemia is a rare disorder caused by a severe deficiency of fibrinogen (choice D). In this disorder, both the prothrombin time and partial thromboplastin time are markedly prolonged. Vitamin K (choice E) plays a crucial role in coagulation by acting as a cofactor for the synthesis of the vitamin K dependent clotting factors: 11, VII, IX and X. When vitamin K is deficient, the clotting cascade is interrupted and the prothrombin time will be prolonged
Herpes simplex virus type 1 is clinically most often associated with
1. neural lesions.
2. genital lesions.
3. cervical carcinoma.
4. oral and ocular lesions.
5. none of the above
The correct choice is choice 4, oral and ocular lesions. Herpes simplex I, or HSV-I infection, is one of the most common viral infections. Almost all individuals are infected between 6 and 18 months of life, but only a small proportion become symptomatic. Those who do develop gingiva stomatitis which is characterized by multiple vesicular lesions in and around the mouth. Vesicles may also erupt over the cornea and cause blindness. The recurrent form of HSV-I infection is extremely common. After the primary infection, the virus becomes latent in the dorsal root ganglia. At times of stress, it reactivates and causes the common fever blister. This is a vesicular lesion at the mucocutaneous border of the lip. Neural lesions, choice 1, are rare complications of HSV-I infection, though the advent of AIDS has made HSV-I neurologic disease a significant clinical entity. A neurologic illness caused by HSV-I is encephalitis. This may occur during primary infection or with reactivation. Genital lesions, choice 2, may occur with HSV infection. However, they are usually caused by herpes simplex virus II infection. HSV-II has also been implicated in the pathogenesis of cervical carcinoma along with other viruses, most notably the papilloma virus. HSV-I does not induce cervical carcinoma. Once again, the correct choice to question is choice 4
Hypersensitivity to M. Tuberculosis is manifested by which of the following?
A. Necrosis
B. Exudation
C. Epithelioid cells
D. Langhans' giant cells
E. Spreading of the initial focus
The correct answer is choice A. The immune system normally resists primary infection by M. tuberculosis. Activated macrophages engulf the bacteria and wall them off into nodules called tubercles or granulomas. Calcified tubercles are called Ghon complexes. Langhans cells are fused macrophages, while epithelioid cells are modified macrophages. Both are involved in walling off chronic infections. If primary TB becomes reactivated, it is termed secondary TB. TB which spreads beyond the lungs, usually through the bloodstream, is known as miliary TB. However, secondary TB still present in the lungs, is often contained by the actions of the immune system. The resulting hypersensitivity reactions of the immune system, however, may destroy areas of lung tissue, causing necrosis of the lung parenchyma, visible as large dark areas on X-ray. So the correct answer to question is choice A.
If a gram stain reveals the presence of gram negative diplococci, the infection is most likely caused by which of the following organisms?
A. Campylobacter jejuni
B. Haemophilus influenzae
C. Neisseria meningitidis
D. Staphylococcus aureus
E. Streptococcus pneumoniae
The correct answer is C Neisseria meningitidis
. The gram-stain is often a quick and easy way to assess the "type" of bacterial infection in a given patient. For example, if a gram stain reveals the presence of gram negative diplococci, the infection is most likely caused by the bacteria Neisseria meningitidis. Based on this information, a clinician can prescribe an antibacterial regimen that has excellent coverage against this particular bacteria. Campylobacter jejuni (choice A) can be described as microaerophilic, motile gram-negative rods. These bacteria commonly cause diarrhea. Haemophilus influenzae (choice B), which is commonly seen in adults with pneumonia, can be described as gram-negative, pleomorphic rods. Staphylococcus aureus (choice D) is a bacteria associated with a variety of infections, such as skin infections and upper respiratory infections. These organisms are commonly described as gram-positive cocci in clusters. Streptococci pneumoniae (choice E), as the name suggests, is a common cause of pneumonia. These organisms are gram-positive diplococci
If a patient is diagnosed with mononucleosis, one would expect to see all of the following EXCEPT
A. Atypical lymphocytes on the differential
B. Lymphadenopathy
C. Normal liver function tests
D. Normal renal function tests
E. Splenic enlargement
The correct answer is C.NOT. Normal liver function tests
Mononucleosis is a condition caused by the Epstein-Barr virus. Frequently, individuals with this virus will present with fever, sore throat and severe malaise. In almost all cases of mononucleosis, liver function tests will be elevated. There is normally an increase of atypical lymphocytes on the differential (choice A). Lymphadenopathy (choice B) is also commonly seen in cases of mononucleosis. Although liver function tests are abnormal in patients with this disease, renal function tests are mostly within normal limits (choice D). Splenic enlargement (choice E) is commonly seen in more than half of the cases of mononucleosis
If a woman contracts an infection caused by Treponema pallidum during pregnancy, the child is at an increased risk for the development of which of the following?
A. Genital herpes
B. Gray teeth and brittle bones
C. Kernicterus
D. Koplik's spots
E. Widely spaced, peg shaped incisors
The correct answer is E. Treponema pallidum is the causative organism of syphilis. The transmission of Treponema pallidum from a mother with syphilis to her fetus across the placenta can occur at virtually any stage of pregnancy. Signs and symptoms of late congenital syphilis occur 2 years after birth; the appearance of Hutchinson's teeth affect the upper central incisors resulting in notched, widely spaced, peg-shaped incisors (choice E). Other congenital effects include the appearance of "mulberry" molars or the poorly developed cusps of sixth-year molars. Genital herpes (choice A), inthe mother, can result in the development of mucocutaneous lesions on the lip of theperioral skin in the neonate as well as the development of CNS infections in the neonate. A bacterial infection treated with a tetracycline during pregnancy can result in permanent tooth discoloration (grayish teeth) (choice B), tooth enamel defects, and retarded bone growth in infants and children. Kernicterus (choice C) is a neurological disorder seen in infants born to mothers who receive sulfonamide antibiotic during pregnancy. Koplik's spots (choice D) are seen in individuals who have contracted rubeola; these spots are small, irregular, grayish-white lesions on upper buccal mucosa and are considered pathognomonic for this disease
If an individual has a tumor that is secreting excess amounts of antidiuretic hormone, the pathological effect on the body would be
A. Dehydration
B. Hypertension
C. Kidney failure
D. Vasodilation of the peripheral blood vessels
E. Polycythemia
The correct answer is B. The gist of this question is as follows: What happens when excess antidiuretic hormone is secreted? Antidiuretic hormone (ADH) or vasopressin is a hormone that is typically released when there is a fall in blood pressure or there is a rise in the blood concentration of electrolytes. The primary function of antidiuretic hormone is to decrease the amount of water lost through the kidneys through increased renal reabsorption of water. Since this hormone functions to increase blood pressure, an excessive amount of this hormone in the blood would lead to hypertension or high blood pressure. Dehydration (choice A) would not occur when antidiuretic hormone blood levels increase since the primary function of ADH is to prevent water loss. Kidney failure (choice C) would occur if there was insufficient blood flow to the kidney; increasing the blood volume would increase blood flow to the kidney. Antidiuretic hormone is known to cause vasoconstriction of the peripheral blood vessels; not vasodilation of the peripheral blood vessels (choice D). Polycythemia (choice E) is a condition characterized by an increased number of red cells in the blood
If an individual was diagnosed with emphysema, one would expect to see all the following EXCEPT
A. Cyanosis
B. Decreased diffusing capacity of the lungs
C. Increased anteroposterior diameter
D. Increased total lung capacity
E. Minimal sputum production
The correct answer is A. NOT Cyanosis Emphysema is a chronic condition characterized by shortness of breath and an inability to tolerate physical exertion. The underlying problem of this disease is due to the destruction of alveolar surfaces and inadequate surface area for carbon dioxide and oxygen exchange. In emphysema, there is minimal ventilation-perfusion imbalance because the individual is continually hyperventilating and therefore able to keep blood gases within the normal range. In other words, cyanosis rarely develops. The primary etiology of this disease is related to the decreased diffusing capacity of the lungs (choice B). Patients with emphysema typically develop a "barrel chest"; the individuals anteroposterior diameter increases (choice C). In emphysema, the FEV1 is decreased; both the total lung capacity (TLC) (choice D) and residual volume (RV) are increased. Minimal sputum production (choice E) is commonly seen in patients with emphysema.
If clindamycin is a lincosamide antibiotic used in the treatment of oral infections commonly caused by anaerobic bacteria, it would most likely be used in the treatment of an infection caused by
A. Bacteroides fragilis
B. Corynebacterium diphtheria
C. Escherichia coli
D. Proteus mirabilis
E. Streptococcus mutans
The correct answer is A. Bacteroides fragilis
Clindamycin is an antibiotic primarily used in the treatment of serious anaerobic infections (including serious oral infections) when other less toxic alternatives are not recommended. This question is essentially asking, "Which of the following is an anaerobic bacteria?" Bacteroides fragilis (choice A) is an anaerobic bacteria commonly associated with the development of gingival diseases. Corynebacterium diphtheria (choice B) and Streptococcus mutans (choice E) are both gram positive organisms; Streptococcus mutans is the most common cause of dental caries. Both Escherichia coli (choice C) and Proteus mirabilis (choice D) are gram-negative organisms
Immunosuppressed patients are particularly prone to develop severe oral disease caused by
1. Candida albicans.
2. Actinomyces israelii.
3. Pneumocystis carinii.
4. Streptococcus mutans.
5. Streptococcus salivarius
The correct choice is choice 1, candida albicans. Oral candida albicans infection, also called candidiasis or moniliasis, is a marker of immunosuppression. Candida is a ubiquitous organism and it does not cause infection in the normal host. However, if the immune system is compromised due to AIDS, congenital disease, renal transplantation therapy or leukemia, or if the normal flora ol the oral cavity has been cleared by long-term treatment with broad spectrum antibiotics, oral candidiasis results. This disease present as white curd-like lesions of the tongue and buccal mucosa. In patients with oral candidiasis and risk factors for AIDS, esophageal candidiasis should be suspected. An esophagram should be obtained if the patient has dysphagia. Esophageal candidiasis is a treatable disease and it establishes the diagnosis of AIDS by CDC criteria. Actinomyces israelii, choice 2, causes infections of soft tissue which lead to abscess formation. This disease usually follows oral trauma or aspiration of saliva. It can occur in a normal host and not a marker of immunosuppression. By contrast, Pneumocystis carinii, choice 3, causes disease only in immunocompromised patients. In fact, infection with this organism is sufficient for a diagnosis of AIDS. Pneumocystis, unlike candida, does not cause infection in the mouth. It is a lung pathogen which causes severe pneumonia with a large alveolar to arteriole oxygen gradient. Strep. mutans, choice 4, was already discussed. This organism is responsible for the development of dental caries. It causes disease in the normal as well as the immunocompromised host. Strep. salivarius, choice 5, is an alpha-hemolytic Streptococcus. This organism can cause endocarditis which is a fairly common disease in IV drug abusers and those with valvular heart disease. It is not typically an infection of immunocompromised patients, though these patients may have a particularly poor prognosis if they do develop Strep. endocarditis. Once again, the correct choice to question is choice 1.
Impetigo is a superficial infection of the skin most commonly caused by
A. Group A streptococci
B. Staphylococcus aureus
C. Streptococcus agalactiae
D. Streptococcus mutans
E. Streptococcus pyogenes
The correct answer is A. Group A streptococci
. Impetigo is described as a type of superficial cellulitis primarily caused by Group A Streptococci (choice A) followed by Staphylococcus aureus (choice B). This condition is most commonly seen in hot, humid weather, which can aid in the colonization of the skin. When minor trauma occurs, such as a scratch, the bacteria enter the superficial layer of the skin, which subsequently results in the development of an infection. Since this is a highly communicable disease, it can be spread through close contact. Patients typically present with small, fluid filled (pus) vesicles that readily rupture and form the golden "crusts" characteristic of this disorder; pruritus is common. Streptococcus agalactiae (choice C) is known to cause septicemia in infants as well as bovine mastitis. Dental caries is a destructive disorder of the hard tissues of the teeth primarily caused by Streptococcus mutans (choice D). Streptococcus pyogenes (choice E) causes scarlet fever, epidemic "sore throat", tonsillitis and many other infections in humans.

(extra note- group A strep is strep pyogenes....Staph would be correct)
In a patient with parathyroid adenoma, you would expect to find:
A. high serum calcium
B. neuromuscular excitability
C. tetany
D. cardiac conduction defects
E. hyperphosphatemia
The correct answer is choice A. high serum calcium
. A parathyroid adenoma usually causes excess secretion of parathyroid hormone. This hormone, which stimulates osleoclasts, causes calcium to be removed from tissues, especially bone, and placed back into the blood. A result should be hypercalcemia with subperiosteal resorption, and sometimes kidney stones. All of the other choices listed would be due to hypocalcemia (low serum calcium), including that caused by excess calcitonin, or loss of parathyroid function. The answer choices B, C, D and E review standard results of low blood calcium
In addition to Neisseria meningitidis which of the following is a significant cause of meningitis?
A. Streptococcus faecalis
B. Streptococcus pyogenes
C. Streptococcus pneumoniae
D. Staphylococcus aureus
E. Staphylococcus epidermidis
The correct answer is choice C. Streptococcus pneumoniae is a significant cause of meningitis and is the number one cause of meningitis in the elderly. Choice A - Streptococcus faecalis is part of the normal fecal flora and causes urinary tract infections in hospitalized patients; rare cause of subacute endocarditis. It is not a significant cause of meningitis. Choice B - Streptococcus pyogenes (Group A, B-Hemolytic), is an obligate human parasite spread person-to-person by respiratory secretion via droplets, direct contact with the skin, or fomites. Clinical manifestations are characterized as suppurative or nonsuppurative predominantly involving pharyngitis and various skin infections. It is not a significant cause of meningitis. Choice D - Staphylococcus aureus is a common infectious agent of humans and tends to cause localized or toxin-mediated disease. It should not be considered normal flora, but it does transiently colonize the nasopharynx, skin, and vagina of up to 30% of the population. It is not a significant cause of meningitis. Choice E - Staphylococcus epidermis is most commonly a nosicomial pathogen. Clinical manifestations are typically related to instrumentation and procedures and include endocarditis and urinary tract infections. It is not a significant cause of meningitis. Therefore the correct answer to question is C.
In Goodpasture syndrome, antibodies are directed against the basement membranes of both the:
A. heart and lung
B. liver and kidney
C. kidney and lung
D. kidney and pancreas
E. liver and pancreas
The correct answer is choice C. kidney and lung
Goodpasture syndrome involves both a necrotizing hemorrhagic pneumonitis and a focal glomerulonephritis. Antibodies are formed to common antigens in the basement membranes of both organs. This leads eventually to renal failure, uremia, and pulmonary disease. In immunofluorescent studies, immunoglobulins can be seen lining the basement membranes of both organs
In order for a virus to infect a host cell, it must first absorb to the cell surface. This involves
1. phagocytic activity by the cell.
2. energy output on the part of the cell.
3. insertion of virally specified glycoproteins into the host cell membrane.
4. a specific interaction between a viral surface component and a specific receptor on the cell membrane.
5. all of the above
choice is 4 a specific interaction between a viral surface component and a specific receptor on the cell membrane.
In order for virus particles to infect a cell, they must enter the cell. The first step of entry is to attach to the cell membrane surface. This is the process of adsorption, not absorption as the question states. Anyway, it occurs as the virus adsorbs at a specific cell receptor site. The virus loses one viral polypeptide in the process. Choice 1 is incorrect. The cell does not phagocytize or ingest the virus. Very few human cells have phagocytic ability. Choice 2 is incorrect. No energy use in the form of ATP is needed for adsorption. Choice 3 is incorrect as the virus coat is protein, but not glycoprotein. Choice 4 is correct as the viral protein will attach only to specific membrane receptor sites. So the correct answer to question is choice 4.
Inadequate immobilization atter a bone fracture is likely to result in
1. lipping.
2. a pannus.
3. pseudoarthrosis.
4. Heberden's nodes
The correct choice is choice 3, pseudoarthrosis. Bone healing following fracture occurs in distinct stages. In the first stage, blood at the fracture site coagulates and a fibrin meshwork or procallus is elaborated. This meshwork becomes coated with fibroblasts, cartilage and bone matrix, resulting in fibrocartilaginous callus formation. With time, the fibrocartilaginous callus is resorbed and an osseous callus is laid down. The new bone is then remodelled along stress lines. The healing process requires adequate immobilization to reach completion. If the bone is moved during repair, the osseous callus may fail to form, leaving a fibrocartilaginous callus at the fracture site. This callus is flexible and the bone may appear to have a false joint or pseudoarthrosis. Lipping, choice 1, is the slang term for osteophyte formation. Osteophytes are bony outgrowths that occur along the ridges of bones in degenerative joints. They may fracture and produce joint immobility or pain. A panus, choice 2, is a mass of chronic inflammatory tissue which forms in the joints of patients with rheumatoid arthritis. As the disease progresses, the panus erodes into surrounding bone and may lead to fusion of the joint. Heberden's nodes, choice 4, are bony bumps on the distal phalanges of the hands in patients with osteoarthritis. Once again, the correct choice to question is choice 3.
Increased functional demand on the heart produces increased size of the myocardium by
1. hyperplasia.
2. hypertrophy.
3. calcification.
4. fatty infiltration.
5. increased amounts of fibrous connective tissue.
The correct answer is choice 2. hypertrophy, Increased functional demand on the heart, for example, through exercise will cause enlargement of the heart muscle or myocardium. However, this enlargement does not occur through hyperplasia or increase in cell number. It is believed that the number of cardiac muscle cells or fibers in the heart remains fairly constant after birth and that increases in myocardial size result from hypertrophy or added length or width of the cardiac muscle cells. Choices 3, 4 and 5 are all incorrect. Fatty infiltration, increased fibrous connective tissue and calcification are all pathologic conditions. They are not normal responses of the heart to increased demand which will help the heart meet the increased needs of the body. So the correct answer to question is choice 2.
Insufficient intake of protein, despite sufficient caloric intake, results in the condition:
A. marasmus
B. kwashiorkor
C. beriberi
D. scurvy
E. pernicious anemia
The correct answer is choice B. Kwashiorkor occurs when the body receives sufficient caloric intake (energy value) but insufficient protein content. This can occur if a diet is limited, for example, to one of a few starchy food items. Symptoms include edema, anemia, dermatoses (desquamation and pigment changes), hepatomegaly, and anergy. Marasmus (choice A) refers to insufficient caloric intake and results in cachexia, muscle wasting, loss of subcutaneous fat, and failure to thrive. Beriberi (choice C) occurs through lack of thiamine, a B vitamin. Symptoms include nervous system deterioration and heart dilation. Scurvy (choice D) occurs through lack of vitamin C. Symptoms include poor wound healing and bleeding gingiva. Pernicious anemia (choice E) results from Vitamin B12 (cobalamin) deficiency and leads to a megaloblastic anemia and neurologic damage
Insufficient numbers of circulating polymorphonuclear leukocytes (PMNs) is best described as:
NBDE Part I released questions - Microbiology-Pathology
Insufficient numbers of circulating polymorphonuclear leukocytes (PMNs) is best described as:
A. leukemia
B. neutropenia
C. leukopenia
D. pancytopenia
E. leukocytosis
The correct answer is choice C. Neutropenia (choice B) describes the state of insufficiennt numbers of neutrophils (PMNs), whereas leukopenia is a more general term referring to low levels of leukocytes. Leukemias (choice A) are neoplasms resulting in elevated levels of immature white blood cells in circulation. Pancytopenia (choice D) refers to an overall diminishment of all cell types from the bone marrow, including red blood cells, white blood cells, and platelets. Leukocytosis (choice E) refers to an elevated leukocyte level, such as that following infection (as opposed to that found in leukemia). Causes of neutropenia may include problems with the bone marrow (aplastic anemia or drug suppression), increased destruction of PMNs, or effects of drugs (chloramphenicol or sulfonamides).
Jaundice does NOT result from which of the following?
NBDE Part I released questions - Microbiology-Pathology
Jaundice does NOT result from which of the following?
1. Gallstones
2. Hepatitis A
3. Hemolytic anemia
4. Vitamin K deficiency
5. Carcinoma of the common bile duct
Good Work
The correct choice is choice 4, vitamin K deficiency. Jaundice refers to the yellowish pigmentation of the sclera, skin and mucous membranes in individuals with elevated serum bilirubin levels. Any condition which causes hyperbilirubinemia may cause jaundice. The synthesis of coagulation factors VII, IX and X are vitamin K dependent. Therefore, vitamin K deficiency causes a coagulopathy. It does not lead to jaundice. Vitamin K deficiency is present, however, in many jaundiced individuals. This occurs because conditions which cause jaundice may also cause a vitamin K deficiency. An example of such a condition is liver failure. The liver synthesizes bile salts which are essential for vitamin K absorption from the gut. The liver also clears bilirubin from the blood. In liver failure, both of these functions are compromised and a coagulopathy with jaundice may result. Gallstones, choice 1, and carcinoma of the common bile duct, choice 5, cause jaundice when they obstruct bile flow into the duodenum. Bilirubin is a major component of bile and it cannot be adequately excreted in these conditions. As a consequence, it overflows into the bloodstream. Bilirubin which has been processed by the liver is called conjugated bilirubin. It is assayed as direct bilirubin when liver function tests are requested. It is direct bilirubin that is elevated in gallstones and carcinoma of the bile duct. Indirect bilirubin or unconjugated bilirubin is devated in the bloodstream when bilirubin enters the bloodstream before being processed by the liver. This occurs in hemolytic anemia, choice 3. When red cells are hemolyzed, globin chains are released from hemoglobin molecules. They are converted to bilirubin by a sequence of enzymes and the increased bilirubin load cannot be adequately cleared by the liver. Hepatitis A, choice 2, causes an elevation in both direct and indirect bilirubin. Direct bilirubin is released into the blood when hepatocytes are destroyed. As a consequence of hepatocellular necrosis, the clearance of indirect bilirubin is reduced. Indirect hyperbilirubinemia follows. Both the direct and the indirect hyperbilirubinemias cause jaundice. Indirect hyperbilirubinemia is very dangerous in newborns. Indirect hyperbilirubin deposits in the brain and causes a neurologic condition known as kernicterus. Once again, the correct choice for question is choice 4.
Keloids are composed predominantly of
NBDE Part I released questions - Microbiology-Pathology
Keloids are composed predominantly of
1. adipose tissue.
2. collagen fibers.
3. granulation tissue.
4. caseous necrotic debris.
Good Work
The correct answer is choice 2. Keloids are formed when abnormal increases of collagen occur during connective tissue repair. They can result in bulging tumor-like scars. Choice 1 is incorrect. Adipose tissue is fat storage tissue which would contain high levels of lipid but not collagen. Choice 3 is incorrect. Granulation tissue is undifferentiated repair tissue which forms in wound healing. It does not contain excess collagen and keloids are not formed from granulation tissue. Granulation tissue contains vascular and epithelial cells, fibroblasts, macrophages and mast cells. Choice 4 is incorrect. Caseous necrotic debris refers to debris which is necrotic, being composed of dead and decaying cells. Caseous necrotic debris is cheese-like in appearance, soft and friable. It is common in tuberculosis granulomas, fungal infections and central areas of certain tumors. So the correct answer to question is choice 2.
Lecithinases are produced by
NBDE Part I released questions - Microbiology-Pathology
Lecithinases are produced by
1. Bacillus anthracis.
2. Clostridium perfringens.
3. Salmonella schottmulleri.
4. Mycobacterium tuberculosis.
5. Corynebacterium diphtheriae.
Good Work
The correct answer is choice 2. Clostridia, in general, are anaerobic, gram positive motile rods. They are responsible for the diseases botulism, tetanus, and gas gangrene. Clostridia spores are usually introduced into wounds by soil or feces. In an anaerobic environment caused by decaying or necrotic tissue, the spores will germinate and the resulting cells produce a number of toxins. They can include hemolysins and necrotizing toxins. Clostridium perfringens in particular produces an alpha toxin which is a lecithinase. It damages cell membranes by splitting lecithin. Clostridia can also produce collagenases , DNAase and enterotoxins . So the correct answer to question is choice 2.
Left-sided heart failure or shock may be associated with
NBDE Part I released questions - Microbiology-Pathology
Left-sided heart failure or shock may be associated with
1. pneumoconiosis.
2. bronchiectasis.
3. pulmonary edema.
4. pulmonary emboli.
Good Work
The correct answer is choice 3. Pulmonary edema can be caused by left-sided heart failure. As the left ventricle reduces the volume of blood pumped, a backup of fluid and pressure occurs through the left atrium, pulmonary vein and lungs, with resulting pulmonary edema. Choice 1, pneumoconiosis, is not associated with left-sided failure, but instead is related to dust inhalation, including coal and asbestos, and is a restrictive lung disease. Choice 2, bronchiectasis, refers to abnormal dilation of bronchi and bronchioles caused by chronic necrotizing infection. It is not associated with left-sided failure. Choice 4. pulmonary emboli, are due to small coagulated areas of blood traveling to the lungs usually from the large veins of the legs and pelvis. They travel back to the right side of the heart and then in the pulmonary artery to the lungs. However, they are not associated with left-sided heart failure. So the correct answer to question is choice 3.
Lysozyme can attack bacterial cells by
NBDE Part I released questions - Microbiology-Pathology
Lysozyme can attack bacterial cells by
1. degrading their flagella.
2. inactivating their ribosomes.
3. preventing the transpeptidation reaction.
4. breaking the bond between N-acetyl glucosamine and N-acetyl muramic acid.
Good Work
The correct answer is choice 4. Lysozyme is found in tears, saliva and nasal secretions. It attacks bacterial cell walls composed of peptidoglycans. Peptidoglycans contain polymers of repeating N-acetylglucosamine and N-acetylmuramic acid joined by B 1-4 linkages. Lysozyme breaks these B 1-4 linkages, thus dissolving the cell walls. Lysozyme is particularly effective on grampositive bacteria. Choice 1 is incorrect. Lysozyme has no effect on flagella. Choice 2 is incorrect. Lysozyme does not reach the interior of the bacterial cytoplasm where ribosomes are located. It only acts on the cell wall. Choice 3 is incorrect. Lysozyme has no effect on transpeptidation which involves protein formation. As mentioned previously, it acts only on the peptidoglycans of the cell wall. So the correct answer to question is choice 4.
Malignant epithelial cells have an increased number and wider distribution of which of the following receptors?
NBDE Part I released questions - Microbiology-Pathology
Malignant epithelial cells have an increased number and wider distribution of which of the following receptors?
A. IL-1
B. Laminin
C. Histamine
D. Complement
E. Immunoglobulin
Good Work
The correct answer is choice B. When normal cells become malignant, a number of changes can occur in their receptors. The number and distribution of normal receptors may change, and the tumor cell may express new receptors not previously shown. These are known as tumor-specific antigens. This concept is the basis for potential immunological treatment of tumors. In this case, malignant epithelial cells increase the number of receptors known as laminin receptors, on the cell surface. In general, for tumors caused by chemical carcinogens, when cells of identical genetic background are transformed with the same chemical carcinogen, each tumor formed has its own unique tumor-specific antigens. When cells are transformed by an oncogenic virus, all new tumors express the same tumor-specific antigen. Cells transformed by different viruses exhibit different tumor-specific antigens. So the correct answer to question is choice B.
Methicillin-resistant Staphylococcus aureus (MRSA) organisms are:
NBDE Part I released questions - Microbiology-Pathology
Methicillin-resistant Staphylococcus aureus (MRSA) organisms are:
A. resistant to all antibiotics
B. a form of untreatable TB
C. common causes of nosocomial infections
D. streptococci
E. none of the above
Good Work
The correct answer is choice C. MRSA refers to methicillin-resistant Siaphylococcus aureus. These organisms cause difficult-to-control infections, often in hospital patients and health care workers. Staph organisms that were resistant to most penicillins were formerly killed by methicillin, a penicillinase-resistant drug. However, more strains of Staphylococcus are now resistant to methicillin, and infections by these organisms are difficult to control. Vancomycin is sometimes used. The resistance of MRSA organisms is probably due to their exposure to several antibiotics in the hospital environment, and patients in hospitals are most susceptible. Hospital-based infections are known as nosocomial. A similar phenomenon is found in multiple drug resistant tuberculosis (MDRTB).
Microcytic hypochromic anemia is most often due to
NBDE Part I released questions - Microbiology-Pathology
Microcytic hypochromic anemia is most often due to
1. infection.
2. malabsorption.
3. folate deficiency.
4. chronic blood loss.
5. a genetic abnormality.
Good Work
The correct choice is choice 4, chronic blood loss. Microcytic hypochromic anemia may be due to a variety of causes. The most common cause is iron deficiency. Iron deficiency may result from blood loss secondary to acute or chronic hemorrhage or even physiologic bleeding such as menses. It also occurs when demand increases, a common occurrence in pregnant women. In impoverished countries, iron deficiency results from decreased iron intake. Iron deficiency may also result from malabsorption, choice 2, but this is uncommon except in Crohn's disease. Microcytic hypochromic anemia also occurs when the individual cannot utilize body iron stores. This occurs in chronic inflammatory diseases and in cancer. Genetic abnormalities, choice 5, may cause this type of anemia as well. The prevalence of these genetic abnormalities is very low, however, when compared to the prevalence of chronic blood loss. Infection, choice 1, is not a cause of anemia though it may lead to conditions which cause anemia such as malabsorption or disseminated intravascular coagulopathy. Folate deficiency, choice 3, causes a megaloblastic anemia. It is a common finding in chronic alcoholism and other diseases associated with malnutrition. Once again, the correct choice to question is choice 4.
Most bacterial endotoxins are composed of
NBDE Part I released questions - Microbiology-Pathology
Most bacterial endotoxins are composed of
1. pure lipids.
2. pure proteins.
3. pure carbohydrates.
4. mucoprotein complexes.
5. lipoprotein-polysaccharide complexes.
Good Work
The correct choice is choice 5, lipoprotein-polysaccharide complexes. Endotoxins are composed of lipid, protein and sugar moieties, but only the lipid portion called lipid A is responsible for the biologic effects of this molecule. Endotoxin is found only in gram-negative bacteria. It is a component of the outer membrane which is released into the circulation when the bacterium is killed by the host defense system. Circulating endotoxin has a variety of effects on vascular permeability and clotting mechanisms, and may E ultimately lead to hypotension and disseminated intravascular coagulopathy. Endotoxin differs from the various exotoxins produced by many strains of bacteria in that exotoxins are generally composed of protein alone. Examples of exotoxins include tetanospasmin which is responsible for tetanus and enterotoxin which causes cholera. Mucoproteins, choice 4, are protein polysaccharide complexes. Unlike endotoxin, they lack lipid. Once again, the correct choice to question is choice 5.
Osteomyelitis is most commonly caused by
1. Actinomyces bovis.
2. Borrelia vincentii.
3. Nocardia asteroides.
4. Staphylococcus aureus.
5. Mycobacterium tuberculosis.
Good Work
The correct choice is choice 4, Staphylococcus aureus. Osteomyelitis is an infection of bone which may or may not involve the bone marrow. It usually begins when a blood-borne organism, most commonly Staph aureus, is deposited in bone. Osteomyelitis can also occur following penetrating trauma in which the organism is introduced directly into bone. Osteomyelitis most commonly occurs in children and adolescents and it requires a protracted course of IV antibiotics and occasionally surgical debridement. Any organism can cause osteomyelitis. In addition to Staph. aureus, various strains of Streptococci, Pneumococci and Neisseria commonly cause osteomyelitis. Pseudomonas aeruginosa is a common pathogen among drug abusers. With the advent of AIDS, other organisms have been found frequently in the bone marrow, including Mycobacterium tuberculosis, choice 5, and atypical Mycobacteria including Mycobacterium avium intracellulare. Actinomyces, choice 1, rarely enters bone. More frequently, it causes abscesses in connective tissue which forms sinus tracts in the skin. Borrelia, choice 2, are spirochetes like T. pallidum, the agent causing syphilis. Borrelia has been linked to relapsing fever, skin and mucous membrane ulcers and lung lesions, but is not associated with osteomyelitis. Nocardia, choice 3, is primarily a pathogen in the lung where it causes suppurative fibrotic and necrotic lesions which may be cavitary and diffuse. Once again, the correct choice to question is choice 4.
Organisms that exhibit dimorphism and grow on Sabouraud's medium are
1. fungi
2. bacteria.
3. mycoplasmas.
4. rickettsiae.
5. spirochetes.
Good Work
The correct choice is choice 1, fungi. Dimorphism is a phenomenon demonstrated by some species of fungi. The term literally means "two forms." Fungi which are dimorphic can be found as either a mold or a yeast. Molds are collections of multiple tubular structures called hyphae. They are multicellular conglomerates with hyphae specialized for various functions such as reproduction or nutrient absorption. Yeasts, by contrast, are unicellular. They are quite small and tend to be spherical. They show no specialization of function. Fungi which cause disease in humans are usually dimorphic. They are yeasts when invading the host, but become molds when cultured. Fungi are generally cultured on Sabouraud's agar. This agar supplies the fungi with glucose and peptone as nutrients, and inhibits the growth of bacteria with chloramphenicol and cycloheximide. Fungi are slow growing. Consequently, fungal cultures may take weeks to produce positive results. Bacteria, choice 2, are dimorphic in that they can exist as unicellular organisms or as spores. They are selectively inhibited by Sabouraud's agar, however. Mycoplasma are very small microorganisms which resemble bacteria in their ability to grow on artificial media, though they do not grown on Sabouraud's agar. They show dimorphism insofar as they can be found as spherical bodies or as filaments. Rickettsia, choice 4, are obligate intracellular microorganisms. They are always found as Coccobacilli either singly or in pairs. They do not display dimorphism. Rickettsia are not grown on agilr. They require cell culture for proliferation. Spirochetes, choice 5, do not show dimorphism. They are always observed as long and thin spiralled organisms. The spirochetes which cause disease in humans cannot be cultured in artificial media. They will only grow in live organisms. Once again, the correct choice to question is choice 1.
Opsonization may occur as a component of the humoral immune response to
1. virulent Streptococcus pneumoniae.
2. intracellular microorganisms.
3. infections by viruses.
4. nucleic acids.
5. none of the above.
Good Work
The correct choice is choice 1, virulent Streptococcus pneumoniae. Opsonization refers to the coating of microorganisms by antibody and/or complement to enhance phagocytosis. Because the process usually involves antibody, it is considered pan of the humoral response to infection. Antibody can bind only to extracellular organisms. It does not penetrate the plasma membrane. Therefore, choice 2 is incorrect. Viruses, choice 3, usually cause intracellular infection. Consequently, the humoral arm of the immune system does not play an important role in the defense against viral infection. Nucleic acids, choice 4, are not in and of themselves infectious agents. When foreign nucleic acids enter the bloodstream, they are cleared by humoral immune mechanism as are foreign proteins and carbohydrates. However, opsonization is a term reserved for the ingestion of organisms, not molecules. Opsonization is of primary importance in the host's defense against encapsulated extracellular organisms such as Streptococcus pneumoniae. In fact, individuals who cannot produce adequate antibodies for opsonization often succumb to overwhelming Streptococcal infection. Once again, the correct choice to question is choice 1.
Of the following sexually transmitted diseases, the LEAST common is:
A. syphilis
B. Neisseria gonorrhea
C. lymphogranuloma venereum (LGV)
D. chlamydia
E. non-gonococcal urethritis (NGU)
Good Work
The correct answer is choice C. Gonorrhea caused by Neisseria gonorrhoeae (choice B), Chlamydia (choice D), and nongonococcal urethritis (NGU) (choice E) are all extremely common sexually transmitted diseases (STDs). NGU is caused by either Ureaplasma or Mycoplasma, whereas chlamydia is caused by C. trachomatis. Lymphogranuloma venerum (LGV) is also caused by a strain of C. trachomatis and results in a syndrome presenting with swollen infected lymph nodes in the groin. However, this disease form is very uncommon. Syphilis (choice A), caused by Treponema pallidum, is less common than the first three but more common than LGV.
Nephrolithiasis is most likely to result from which of the following diseases?
1. Hyperpituitarism
2. Cushing's disease
3. Hyperparathyroidism
4. Paget's disease of bone
Good Work
The correct choice is choice 3, hyperparathyroidism. Nephrolithiasis is the medical term for kidney stones. Kidney stones may be composed of a variety of constituents. Most stones are calcium containing and most patients with calcium stones have elevated levels of calcium in their urine and normal serum calcium levels. About 10% of patients, however, will have increased serum calcium and this results most commonly from hyperparathyroidism. Other causes of hypercalcemia include Cushing's disease, choice 2, and Paget's disease, choice 4. Hyperpituitarism, choice 1, may cause hypercalcemia if increased ACTH is a component of the disorder. Kidney stones may also be composed of uric acid. These stones occur in patients with gout and in those with leukemia i receiving chemotherapy. Magnesium stones also mur. These can be quite large. They can fill the entire renal pelvis. Magnesium stones are formed during urinary tract infection with Proteus species. Proteus splits urea to form ammonia which readily crystallizes with magnesium to form a very solid crystal. Cysteine stones are the rarest form of renal stone. They occur in patients with a genetic id impairment in the transport of the amino acid cysteine. Cysteine accumulates in the kidney and precipitates around a nidus. Once again, the correct choice to question is choice 3.
Neoplasms of which of the following organs have been shown to be hormonally dependent?
1. Liver
2. Parotid
3. Prostate
4. Pancreas
Good Work
The correct answer is choice 3.
Benign prostatic hypertrophy is probably hormonally related. It involves the formation of large nodules which obstruct the urethral canal. In addition, prostatic carcinoma is also thought to be hormonally related, with androgens in particular being involved. Not mentioned here, but useful in other questions about prostatic carcinoma, is the fact that the tumor can be detected through elevated acid phosphatase produced by the prostatic epithelium. Choices 1,2 and 4 do not produce neoplasms with hormonally dependent growth. So the correct answer to question is choice 3.
Myasthenia gravis is caused by dysfunction of the
1. motor nerves.
2. smooth muscle.
3. sensory nerves.
4. skeletal muscle.
5. myoneural junction.
Good Work
The correct answer is choice 5. Myasthenia gravis is an autoimmune disease in which IgG is directed against the acetylcholine receptor of the neuromuscular junction. Clinical signs include muscular weakness, especially of muscles supplied by cranial and peripheral nerves. It typically affects young women primarily, and generally will not cause sensory problems. Be careful of answers 1 through 4 which state that the disease affects the muscles or nerves when, in fact, it affects the junction between them. So the correct answer to question is choice 5.
Multiple myeloma signs and symptoms include all of the following EXCEPT:
A. lower than normal blood viscosity
B. increased susceptibility to infection
C. accumulation of Bence-Jones proteins
D. proteinuria
E. "punched out" lesions on x-ray
Good Work
The correct answer is choice A. In multiple myeloma, large numbers of excess abnormal immunoglobulins are formed (monoclonal gammopathy). Proteins deposit in bone, joints, and kidney, leading to pain, proteinuria (choice D), and nephropathy. The inability to produce normal antibodies increases susceptibility to infection (choice B). Bence-Jones proteins (choice C) appear in both the blood and urine. The excess blood protein results in a hyperviscous blood, also leading to circulatory damage. "Punched-out" lesions (choice E) refer to characteristic radiolucencies on x-ray caused by loss of calcified tissue.
Multiple drug resistance is related most closely to
1. viruses.
2. plasmids.
3. transformation.
4. cell chromosomes.
Good Work
The correct choice is 2, plasmids. Plasmids are self-replicating, autonomous segments of DNA located in the cytoplasm of many bacteria. These gene packages code for proteins which have a variety of functions. In Clostridium, for example, the exotoxin responsible for botulism is encoded by a plasmid. Similarly, the exfoliation produced by Staph. aureus is actually a plasmid product. Plasmids provide Pseudomonas species with the capacity to metabolize hydrocarbons, and they make some strains of E. Coli resistant to invasion by bacteriophages. Most relevant to patient care are the plasmids which confer drug resistance. These plasmids are referred to as R factors, where R stands for resistance. R factors encode enzymes which inactivate various antibiotics. The most well known of these is penicillinase. This enzyme hydrolyzes the beta-lactam ring of the penicillin molecule. Other R factors encode enzymes which inactivate aminoglycosides through phosphorylation or acetylation, or inactivate tetracycline by modifying the permeability of the bacterial membrane. Virtually every antibiotic has an associated R factor and it can be both challenging and frustrating to treat a patient infected with a resistant organism. Viruses, choice 1, have not been shown to introduce genetic material into bacteria which can encode resistance factors. Choice 4 is incorrect because plasmids are separate from the chromosomal DNA. Plasmids do not encode proteins responsible for cell growth and development as do chromosomes. Transformation, choice 3, refers to the process whereby an organism becomes virulent through the acquisition of a set of genes which encodes virulence factors. This process plays no role in drug resistance, though resistance factors can be passed from one organism to another as can virulence factors. Repeating the correct choice to question, the correct choice is choice 2.
Most rickettsial diseases produce severe illness in humans because rickettsiae
1. produce potent exotoxins.
2. are destructive for endothelial cells.
3. are particularly destructive to cutaneous tissues.
4. are primarily neurotropic and cause extensive damage in certain CNS centers.
Good Work
The correct answer is choice 2.
Rickettsiae are small bacteria that are obligate intracellular parasites. Except for Q fever, rickettsial diseases are transmitted by arthropods; that is, insects, fleas and ticks. Rickettsiae multiply in the endothelial cells of small blood vessels and produce vasculitis, so choice 2 is correct. Vascular lesions may appear on the skin, choice 3, but Rickettsiae cause equal or greater damage to the heart, brain and other organs. Choice 4 is incorrect because their main target cell is endothelial, not neural, even though they may damage the brain. The brain damage is due to vascular problems caused by the infected endothelium. Choice 1 is incorrect. The Rickettsiae produce endotoxins, not exotoxins. So the correct answer to question is choice 2.
Sjögren's syndrome is characterized by all of the following EXCEPT:
A. xerostomia
B. atrophied parotid glands
C. dry, burning eyes
D. peripheral neuropathy
E. sicca syndrome
Good Work
The correct answer is choice B. Sjogren syndrome is an autoimmune disease characterized by xerostomia (dry mouth) (choice A), dry eyes (choice C), with a burning or dry cracked sensation, peripheral neuropathy (choice D), and vasculitis. The parotid gland, rather than being shrunken as might be expected, is actually larger than normal. This is due to fibrosis of the gland, in which glandular tissue is replaced by fibrous connective tissue with no secretory function. The primary form is known as sicca syndrome (choice E), but other secondary forms exist, in which the disease is associated with another autoimmune diseases.
Shock during infection with gram-negative bacteria is most likely caused by
1. an exotoxin.
2. an endotoxin.
3. ribonuclease.
4. a capsular antigen.
5. a flagellar antigen.
Good Work
The correct answer is choice 2. An endotoxin is a material found in gram negative bacterial cell walls. It consists of LPS or lipopolysaccharide and is highly toxic to animals. Of the lipid and polysaccharide segments of LPS, the lipid part, known as lipid A, is the toxic fragment. Endotoxin will cause fever in individuals after a 60-90 minute latent period after injection. Other symptoms include leukopenia or reduced white cell count, hypoglycemia, hypotension and shock. Impaired blood supply to vital organs, triggering of complement and intravascular coagulation are also possible. So the correct answer to question is choice 2.
Rheumatic fever may leave the heart seriously damaged because of
1. primary infection of the heart valves with Group A streptococci.
2. secondary infection of the heart valves with Group A streptococci.
3. damage to the heart valves thought to be a result of hypersensitivity to hemolytic staphylococci.
4. damage to the heart valves thought to be a result of hypersensitivity to Group A streptococci.
Good Work
The correct choice is choice 4, damage to the heart valves thought to be a result of hypersensitivity to group A streptococci. Rheumatic fever is a disease of young children which has been on the rise in recent years. It is characterized by migratory polyarthritis involving large joints, chorea or involuntary limb movements, carditis, subcutaneous nodules, and a skin rash. It occurs several weeks after group A, beta-hemolytic Strep. pharyngitis. Rheumatic fever may be followed by rheumatic heart disease which involves the mitral and aortic valves. Stenosis of these valves results when fibrous tissue is deposited. The carditis of rheumatic fever which often progresses to rheumatic heart disease is thought to be secondary to antibody cross-reactivity. During the primary pharyngitis, antibodies form against the group A streptococci. These antibodies react against an antigen present in myocardial tissue and inflammation of the heart, or carditis, results. Primary infection of the heart valves with group A streptococci mentioned in choice 1 does not occur in rheumatic fever. When Strep species do cause endocarditis, the pathogenic organisms are usually alpha-hemolytic streptococci. Secondary infection of the heart valves, choice 2, occurs when organisms are camed by the blood stream to the heart from a distant site of infection. In rheumatic fever, the blood and heart valves are sterile. The disease is caused by immunologic mechanisms, not by bacterial invasion. Choice 3 is incorrect because, as mentioned, rheumatic fever follows Strep. infection, not Staph. infection. Once again, the correct choice to question is choice 4.
Respiratory syncytial virus (RSV) infection differs from influenza infections in that
A. RSV can be treated with amantadine.
B. RSV causes disease primarily in infants.
C. influenza virus is difficult to transmit.
D. RSV can be prevented with effective vaccines.
E. secretory IgA is not effective in preventing disease.
Good Work
The correct answer is choice B. RSV differs from Influenza infections in that RSV causes disease primarily in infants. Choice A - is incorrect because Amantadine is effective only in type A influenza infections and acts to decrease symptoms. Choice C - is incorrect because Influenza virus is relatively easy to transmit via inhalation. (RSV is transmitted via aerosolized droplets and fomites). Choice D - is incorrect because RSV cannot be prevented with effective vaccines. (for Influenza vaccines composed of inactivated virus are designed to elicit immunity against the existing serotypes in the population. Vaccines change from year to year based on the particular serologic determinants of the virus). Choice E - is incorrect because IgA is found in the gut, breast milk, lungs and tears. It would therefore be present in both RSV and influenzae. Therefore the correct answer to question is B.
Recurrent herpes labialis occurs in people who
1. are hypersensitive to herpesvirus.
2. have been infected with herpesvirus and who have antibodies against the virus.
3. have been infected with herpesvirus but who fail to produce antibodies against the virus.
4. None of the above
Good Work
The correct choice is choice 2, have been infected with the herpes virus and who have antibodies against the virus. Herpes labialis is the medical term for the common fever blister or cold sore. It occurs when the herpes simplex virus number I is reactivated after a latency period. During the latency period, it was present in a dormant state in the trigeminal ganglion. Upon reactivation, the virus travels down the trigeminal nerve to produce localized disease in the distribution of the nerve, most commonly at sites in and around the oral cavity. Reactivation is a poorly understood process. Immunoglobulins present in the patient's serum which were induced by the virus upon primary infection do not prevent reactivation. They do, however, attenuate the disease which tends to be less severe than the primary infection. Choice 1 is incorrect because individuals who are hypersensitive to the virus would produce increased quantities of IgG upon initial exposure. Recurrent herpes labialis would be exceptionally mild or even subclinical in these people. Choice 3 is incorrect because it has been well documented that individuals with recurrent herpes simplex virus I infection have antibodies to the virus which were induced upon primary infection. Once again, the correct choice to question is choice 2.
Prolonged administration of streptomycin may result in damage to which of the following nerves?
1. Optic
2. Facial
3. Auditory
4. Trigeminal
Good Work
The correct answer is choice 3. Streptomycin is an antibiotic that inhibits bacterial protein synthesis and was first isolated from the mold Streptomyces. It binds to the 30S bacterial ribosomal subunit. It is used commonly against Staph aureus, Mycobacterium tuberculosis, and synergistically with penicillin against organisms such as Enterococci and Streptococci. Side effects have included a narrow toxic to therapeutic ratio, renal toxicity and ototoxicity, meaning damage to the hearing through a damaging effect on the auditory nerve. So the correct answer to question is choice 3.
Progressive massive fibrosis of the lung is characteristic of
1. silicosis.
2. anthracosis.
3. actinomycosis.
4. hemosiderosis.
5. bronchiectasis.
Good Work
The correct answer is choice 1. Silicosis is a condition caused by chronic exposure to silica such as in mining, sand blasting or road construction. It begins as nodular fibrosis but may expand to massive fibrosis. Choice 2 is incorrect. Anthracosis is caused by chronic exposure to coal, polluted air or cigarettes. Black pigment is deposited in lung parenchyma. However, disease is not always present. Choice 3 is incorrect. Actinomycosis is an infectious disease caused by Actinomyces species. Choice 4 is incorrect. Hemosiderosis in the lungs is found in idiopathic pulmonary hemosiderosis, an uncommon condition characterized by cough, weight loss and hemoptysis which is coughing of blood. Choice 5 is incorrect. Bronchiectasis refers to abnormal dilation of bronchi and bronchioles caused by chronic necrotizing infection. So the correct answer to question is choice 1.
Production of bone in scar tissue occurs by
1. dysplasia.
2. anaplasia.
3. neoplasia.
4. metaplasia.
5. fibroplasia.
Good Work
The correct answer is choice 4.
Metaplasia refers to reversible change in tissue type from one to another; in this case, from scar tissue to osteoblastic tissue. Choice 1, dysplasia, refers to early pathologic tissue changes, especially abnormal proliferation of atypical cells. Choice 2, anaplasia, is another stage of cellular disorder and refers to loss of cell differentiation and tissue organization. Choice 3, neoplasia, refers to the growth of masses of abnormal tissue, either malignant or benign. Fibroplasia, choice 5, refers to abnormal formation of fibrous connective tissue. So only choice 4, metaplasia, has to do with a change in type of cell produced from one normal type to another in the same location. So the correct answer to question is choice 4.
Patients with Bruton's agammaglobulinemia principally suffer from infections caused by
1. fungi.
2. viruses.
3. parasites.
4. pyogenic bacteria.
5. tubercle bacilli.
Good Work
The correct answer is choice 4.
Bruton's agammaglobulinemia is an X-linked trait which results in failure of maturation of B cells. Therefore, no immunoglobulins can be produced and bacterial infections are extremely difficult to defend against. As an X-linked trait, it appears almost entirely in males and presents early in childhood, with recurring bacterial infections. The affected person has normal T cell function which should indicate some ability to defend against fungi, viruses and parasites, choices 1,2 and 3, and against the Mycobacteria, including the tubercle bacilli, choice 5. So the correct answer to question is choice 4.
Patients receiving chemotherapy for leukemia are particularly prone to develop oral
1. candidiasis.
2. actinomycosis.
3. blastomycosis.
4. histoplasmosis.
5. coccidioidomycosis.
Good Work
The correct answer is choice 1.
Candidiasis is a fungal infection caused by the yeast Candida albicans. It is a condition often found in the immunosuppressed and in patients taking antibiotics for long periods of time. Candida is a naturally occurring member of the normal oral flora and grows opportunistically. In cases of leukemia, chemotherapy is used to reduce abnormal white blood cell counts. This, combined with the fact that the present immune system in the leukemic patient operates poorly, leads to the increased risk of opportunistic infections by yeast and other organisms. Choice 2 is incorrect. Actinomycosis is an infection by Actinomyces israelii. It is not especially increased in the leukemic patient. Choice 3 is incorrect. Blastomycosis is a fungal disease and it affects the lung and skin primarily and it is not especially found in leukemic patients. Choice 4 is also incorrect. Histoplasmosis is another fungal disease, often asymptomatic. It is not especially found in leukemic patients. Choice 5 is incorrect. Coccidioidomycosis is another fungal disease often affecting the lungs, but not especially common in leukemic patients. Again, the correct answer to qucstion is choice 1.
The alum adjuvant employed, in many vaccines acts biologically in tissues to
1. serve as a local inflammatory stimulus.
2. prolong antigen release to immunocompetent cells in the region.
3. Both (1) and (2) above
4. Neither (1) nor (2) above
Good Work
The correct choice is choice 3, both 1 and 2 above. An alum is a double salt consisting of aluminum sulfate and a sulfate of an alkaline earth element such as sodium or potassium. An adjuvant is a chemical compound which facilitates a specific reaction. Alum adjuvants are used in vaccines to augment the immunogenicity of the vaccine. They increase inflammation and decrease blood flow at the injection site. This has the effect of bringing many lymphocytes and macrophages into a region with a high local concentration of the immunogen. The vasoconsuiction prevents the cells of the immune system and the antigen from diffusing away from the site. Through inflammation and vasoconstriction, the alum increases the likelihood that effective immunization will result. Once again, the correct choice to question is choice 3.
The alteration of Corynebacterium diphtheriae to a virulent strain by the transfer of DNA temperate bacteriophage is
1. mutation.
2. sexduction.
3. conjugation.
4. recombination.
5. lysogenic conversion.
Good Work
The correct answer is choice 5. Lysogenic conversion refers to a process in which phages known as temperate phages infect a bacterial cell and can convert the bacterium from one type to another. Phage conversion or lysogenic conversion is thus defined as the acquisition of new properties as a result of phage infection. It differs from transduction in that the genes controlling the new properties are only found in the phage DNA and are not incorporated in the host cell genome. Phage recombination refers to a bacterium becoming infected with two different types of phage particles and then when lysed, releasing phage particles with combined properties of both phage types. Mutation is a general term for a change in DNA structure. Recombination is a general term for DNA rearrangement. Conjugation refers to mating between two types of bacterial cells with exchange of genetic material. So the correct answer to question is choice 5.
The aerosol produced during operative dental procedures is likely to contain a predominance of
1. yeasts and spirochetes.
2. spirochetes and rod forms.
3. spirochetes and rickettsiae.
4. gram-positive microorganisms.
5. gram-negative rod forms and spirochetes.
Good Work
The correct choice is choice 4, gram-positive microorganisms. The aerosol produced during oral surgery will seflect the flora present in the oral cavity. If the patient is healthy and has not been on antibiotics, the flora will consist mainly of gram-positive organisms. This is true because under normal conditions, the mouth is populated predominantly by Streptococci. Yeasts, spirochetes and gram-negative rods are also present in the oral cavity, but at low concentrations. Consequently. an aerosol produced during oral surgery will contain small concentrations of these microorganisms. Rickettsia, mentioned in choice 3, are not present in the oral cavity under normal circumstances. In illness, the flora of the mouth often changes. AIDS patients and patients on broad spectrum antibiotics often have yeasts of the Candida species in abundant quantities in their mouths. An aerosol from these patients would likely show a predominance of yeasts as opposed to gram-positive cocci. Similarly, patients with syphilitic chancres in their mouths may have many spirochetes in their aerosol. Gram-negative rods may colonize the oral mucosa in hospitalized patients and give rise to gram-negative aerosols in these patients. Aerosolization of rickettsia occurs in systemic rickettsia1 disease, but this is primarily due to lung invasion. Oral lesions in rickettsia are not common. Once again, the correct choice to question is choice 4.
The absence of cellular differentiation is characteristic of
1. anaplasia.
2. metaplasia.
3. hyperplasia.
4. hypertrophy
5. inflammation
Good Work
The correct choice is choice 1, anaplasia. Anaplastic tumors are usually more aggressive than tumors which show cellular differentiation. Anaplastic cells can be thought of embryonic cells. They show a tremendous capacity for replication. Differentiated cells are more complex and specialized. They do not replicate as readily. In fact, many differentiated cells such as neurons do not replicate at all. Metaplasia, choice 2, occurs when tissues of one organ begin to look like tissues of another organ. Metaplasia usually occurs in response to a chemical injury. Barrett's esophagus, for example, occurs when acid from the stomach refluxes into the esophagus, causing the esophageal mucosa to transform into gastric mucosa. Another example of metaplasia occurs in smokers. Cigarette smoke causes bronchial mucosa to transform from the normal ciliated columnar epithelium to stratified squamous epithelium. Hyperplasia, choice 3, occurs when the number of cells in a tissue increases. Normal growth requires hyperplasia such as when the breasts enlarge at puberty, but hyperplasia can also be a pre-malignant condition. Hypertrophy, choice 4, occurs when cells increase in size. Body builders cause hypertrophy of muscle cells by lifting weights. Inflammation, choice 5, is the infiltration of tissue by a combination of neutrophils, lymphocytes, macrophages and tissue fluid. It occurs when tissues becomes infected and in some noninfectious diseases as well. Once again, the correct choice to question is choice 1.
The ability of mammalian organs to regenerate after injury is best demonstrated by the
1. lung.
2. liver.
3. heart.
4. brain.
5. kidney.
Good Work
The correct choice is 2, liver. Cells can be classified into three types according to their rate of replication. Labile cells rapidly proliferate and do so continuously. All epithelial cells are labile cells as are cells of the hematopoietic system. Stable cells replicate very slowly, but they maintain the capacity to replicate quickly in the face of injury. Liver cells are the stable cells which are most adept at replication. A rat liver, for example, can regenerate 75% of its mass in one week. Other stable cells include alveolar cells in the lung, choice 1, and kidney cells, choice 5. Heart and brain cells, choices 3 and 4, belong to the third class of cells, the permanent cells. These cells show virtually no capacity to replicate after injury. Following myocardial infarction, for example, the necrotic tissue is replaced with fibrous tissue, not with healthy myocardial cells. Similarly. patients who suffer strokes rarely show improvement in functioning. When they do, it is usually due to decreased cerebral edema, not to regeneration of neurons. Once again, the correct choice to question is choice 2.
Tetany may result from hypofunction of which of the following glands?
1. Thyroid
2. Hypophysis
3. Parathyroid
4. Adrenal cortex
5. Adrenal medulla
Good Work
The correct choice is choice 3, parathyroid. Parathyroid hormone is an important hormone in calcium metabolism. It acts to elevate serum calcium levels by stimulating osteoclastic activity and bone dissolution and by activating vitamin D which absorbs calcium from the gut and by increasing renal resorption of calcium. Hypofunction of the parathyroid gland leads to decreased parathyroid hormone secretion and, consequently, decreased serum calcium levels. Hypocalcemia is associated with a variety of symptoms, including tetany. Tetany is a syndrome characterized by muscle spasms, cramps and seizures. Hypothyroidism, choice 1, is associated with generalized lethargy and slowing of mental and motor functions. Signs of this disease include coarse skin, dry brittle hair, hoarse voice, and peripheral and periorbital edema. Hypofunction of the hypophysis or pituitary, choice 2, can present in a variety of ways. The pituitary secretes thyroid-stimulating hormone, adrenal corticotropic hormone, growth hormone, follicle-stimulating hormone, luteinizing hormone, prolactin, antidiuretic hormone, and oxytocin. Hypopituitarism can be manifested as deficiency syndromes of any or all of these hormones. These deficiency syndromes include hypothyroidism, adrenal insufficiency, growth retardation, ovarian failure, loss of libido, and diabetes insipidus. Hypofunction of the adrenal cortex, choice 4, is most commonly due to adrenal suppression secondary to the administration of steroids or from infiltration of the adrenals by tumor. It is manifested clinically as weakness, weight loss, hypotension, hyperpigmentation and salt craving. Hypofunction of the adrenal medulla, choice 5, is not associated with a distinct clinical syndrome. So the correct choice to question is choice 3.
Tetanus or lockjaw is a disease caused by which of the following species of bacteria?
A. Bacteroides
B. Clostridium
C. Corynebacteria
D. Mycobacterium
E. Nocardia
Good Work
The correct answer is B. The Clostridium genus of bacteria are sporulating anaerobic bacilli that are known to produce potent exotoxins and toxic enzymes. Tetanus or lockjaw is a worldwide disease that is caused by the exotoxin produced by the bacteria Clostridium tetani; hence the name tetanus. The classic muscular spasm in tetanus involves the face, neck, or other parts of the body. The name lockjaw is derived from the muscular contractions that rigidly close or "lock" the jaws together. Bacteroides (choice A) are gram negative pleomorphic rods commonly found in the natural cavities of animals and humans. Outside of their native haunts, the bacteria cause hemolytic and necrotizing lesions. Corynebacteria (choice C) are gram positive aerobic microbes. The major organism of this genus is Corynebacteria diptheriae, which is known to cause diphtheria. Diphtheria is a degenerative disease affecting the nervous system, heart, kidneys and cortex of the adrenal gland. Mycobacterium (choice D) is the genus commonly associated with the acid-fast bacteria Mycobacterium tuberculosis; the organism known to cause tuberculosis.
Ten days after hospitalization for a large, incapacitating myocardial infarct, a 50-year-old man suddenly develops paralysis of the right side of his body. The best explanation for his brain damage is
1. rupture of a congenital aneurysm of the circle of Willis.
2. brain abscess from necrosis of the myocardium.
3. detachment of a bacterial embolus from the pulmonic valve.
4. detachment of a mural thrombus from the right ventricle.
5. detachment of a mural thrombus from the left ventricle.
Good Work
The correct answer is choice 5. It is not uncommon for myocardial infarction to lead to a thrombus formation on the walls of the heart. This thrombus may break off as an embolus and travel to the brain with subsequent brain damage. This is what probably has occurred in this case. So choice 5 is correct. We choose 5 over 3 and 4 because emboli travelling to the brain must come from the left side of the heart or systemic side. Emboli from the right side, either the right ventricle or pulmonic valve, would travel to the lungs and remain there. Choice 2 is incorrect. Brain abscess is not a normal complication of myocardial infarction. Choice 1 is also incorrect. Although rupture of a congenital aneurysm is possible, it is certainly not the most likely cause in this case. Remember that emboli are common post-MI problems and that they often travel from the left heart to the brain. So the correct answer to question is choice 5.
Suppuration is mainly the result of the combined action of four factors. Which of the following is NOT one of these factors?
1. Necrosis
2. Presence of lymphocytes
3. Collection of neutrophils
4. Accumulation of tissue fluid
5. Autolysis by proteolytic enzymes
Good Work
The correct answer is choice 2. Suppuration involves the formation of pus in an infected area. Pus is formed from tissue fluid, cellular debris, dead and dying neutrophils and dead bacteria. Necrosis refers to decay and decomposition of living material so it is included here. Accumulation of tissue fluid is also included. Neutrophils will kill microorganisms through the use of lysosomal enzymes but may also lyse themselves, which is autolysis. These dead neutrophils build up in the pus. Choice 2, lymphocytes are not involved and choice 2 is our answer. Lymphocytes may produce antibodies from plasma cells which are derived from B lymphocytes or they may be involved in cell mediated cytotoxicity as in T lymphocytes. However, they are not found in pus. So the correct answer to question is choice 2.
Striated muscle, smooth muscle, and cardiac muscle have in common the fact that
1. hyperplasia of these elements is common.
2. hypertrophy is a common response to injury.
3. they have a limited capacity to regenerate.
4. they need a constant high O2 concentration to function.
Good Work
The correct answer is choice 3. Muscle tissue cells are generally thought to not increase in number over time. Most researchers feel that the number of individual muscle cells does not increase after birth but that growth of individual cells or fibers occurs both in length and in width. When injured, the muscle will attempt to repair itself using nearby satellite cells to produce new fibers. However, it is not very efficient and often some scar tissue replaces the injured muscle tissue. So choice 3 is correct. Choice 1 is therefore incorrect as hyperplasia refers to production of many new cells. Choice 2 is incorrect. Hypertrophy or growth in size of individual cells or fibers occurs especially after exercise but not often after injury. Choice 4 is incorrect although muscle function is more efficient under aerobic conditions and although anaerobic respiration cannot occur indefinitely without fatigue. For limited amounts of time a muscle can contract using anaerobic respiration. Lactic acid accumulates in the muscle as an anaerobic waste product and is later oxidized as the so-called oxygen debt is repaid. So the correct answer to question is choice 3.
Sterilization refers to which of the following?
1. Absence of all living forms
2. Inhibition of bacterial growth
3. Removal of pathogenic bacteria only
4. Removal of pathogenic bacteria, viruses, and fungi
Good Work
The correct answer is choice 1. Sterilization refers to the absence of all living organisms including bacteria, fungi, viruses and bacterial spores. Choice 2 is incorrect. Inhibiting bacteria only is not sufficient. Nor is killing pathogenic bacteria only, choice 3. Choice 4 sounds better in that it includes bacteria, fungi and viruses but sterilization will kill all organisms, pathogenic or non pathogenic and will also kill spores as well. In fact, to test completeness of sterilization by autoclave, a spore test is usually performed where bacterial spores which are difficult to kill, are autoclaved and checked to see if they can be cultured after the autoclaving. So the correct answer to question is choice 1.
Sterilization of surgical instruments that are sensitive to heat can best be accomplished by:
A. High level chemical disinfectant (glutaraldehyde)
B. Ethyl alcohol
C. Ethylene glycol
D. Ethylene oxide
E. Autoclave (121 C)
Good Work
The correct answer is choice D. The standard chemical sterilizer uses ethylene oxide as the sterilant. The process occurs at room temperature, and is especially useful for materials which are damaged by heat, such as some rubbers, fabrics or cloths. Glutaraldehyde can be used as a high level disinfectant, but is not considered to be a sterilant because it is not sporicidal, except after extensive periods of time. It is not the standard chemical sterilant. Ethyl alcohol is a low to medium level surface disinfectant. Ethylene glycol is automotive antifreeze. The steam autoclave is the standard sterilizing method, generally used for any material unless there is some contraindication to it. Its high temperature (higher than boiling water) and moisture can damage heat sensitive instruments, dull sharp edges and corrode some metals.
Sterilization of surgical instruments that are sensitive to heat can be accomplished by using
1. phenol.
2. an autoclave.
3. ethyl alcohol.
4. ethylene oxide.
Good Work
The correct answer is choice 4. Sterilization of dental instruments is usually accomplished through either a dry heat autoclave (dry clave), a steam autoclave, or a chemical autoclave (chemiclave). The chemical most often used for chemical sterilization is ethylene oxide. Choice 1, phenol, is incorrect. Phenol based disinfectants may be used for high level disinfection. However, these disinfectants do not sterilize, that is, they do not kill all organisms including spores. Choice 2 is incorrect. Autoclaving is a possible method of sterilization. However, it involves the use of high pressure steam so instruments sensitive to heat cannot be autoclaved. Choice 3, ethyl alcohol is incorrect. Ethyl alcohol cannot be used to sterilize and it is also not a high level disinfectant. So the correct answer to question is choice 4.
Sterilization is best defined as:
A. Inhibition of bacterial growth
B. Killling of all viral forms
C. Removal of pathogenic bacteria
D. Absence of all life forms
E. Death of viral and bacterial organisms
Good Work
The correct answer is choice D. When a material or object is sterilized, no living organisms remain on it. That includes bacteria, viruses, fungi, protozoans and, in particular, bacterial spores. Most of the choices listed mention only a few types of organisms. Note that whether the organisms are pathogenic or not is irrelevant. The benchmark for effective sterilization is the ability to kill bacterial spores. Disinfection is generally not sporicidal, while sterilization is. This is also the basis for the use of bacterial spores (of Bacillus stearothermophilus) to test autoclave effectiveness.
Staphylococcus aureus is classified as a:
A. Gram + aerobe
B. Gram - aerobe
C. Gram + anaerobe
D. Gram - anaerobe
Good Work
The correct answer is choice A. Staphylococcus aureus is a Gram-positive aerobic coccus. It produces coagulase and is hemolytic. The most common lesion associated with S. aureus is the abscess. Abscesses tend to fill with liquefied necrotic tissue and inflammatory cells and point in the direction of least tissue resistance.
Soap acts as a disease prevention chemical primarily by:
A. Bacteriocidal effect on microbes
B. Bacteriostatic effect on microbes
C. Removal of microbes
D. Dehydration of microbes
E. Inhibition of microbial growth
Good Work
The correct answer is choice C. Soap acts as an emulsifying agent, dissolving grease and oil which hold microorganisms on the skin. Due to this property, most microorganisms are removed from the hands during washing, and hand washing is considered to be the single most effective infection control measure in the dental office. Bacteriostatic effect refers to inhibition of bacterial growth, rather than direct killing of bacteria. For example, some antibiotics are bacteriostatic. Bacteriocidal refers to direct killing of bacterial cells. Many disinfectants and antibiotics are bacteriocidal. While soap and some soap additives may have either of these effects, they are not the primary effects of soap. Dehydration of microbes is one method by which dry heat autoclaves kill microorganisms. Denaturation of protein is another. Soap does not kill bacteria this way.
The characteristic finding in Actinomyces infections is:
A. sulfur
B. tangled masses of filaments with "clubs"
C. mycolic acid cell walls
D. atypical bone loss around first molars
Good Work
The correct answer is choice B. Be careful in answering this question. Actinomyces infections, which may follow oral surgical procedures, often are accompanied by draining fistulas and material described as "sulfur granules." These granules are creamy-yellowish and consist of tangled masses of Actinomyces filaments with club-like extensions. They also have necrotic debris associated with them. They are not, however, composed of sulfur (choice A) and have only a sulfur-like appearance. Note that mycolic acid cell walls (choice C) are found mostly in mycobacteria, and that choice D describes juvenile periodontitis, which is caused by Actinobacillus, not Actinomyces.
The characteristic lesion of primary syphilis is the:
NBDE Part I released questions - Microbiology-Pathology
The characteristic lesion of primary syphilis is the:
A. gumma
B. maculopapular rash
C. hard chancre
D. saddle nose
Good Work
The correct answer is choice C. A hard chancre is a raised hard area near the entrance of the spirochete into the body, often on the mouth or genitalia. It is described as a raised ulcer with a clean, hard base. It is the characteristic lesion of primary syphilis. Secondary syphilis is characterized by a red maculopapular rash (choice B), especially over the back and shoulders, and pale papules in the mouth, axilla, and anogenital region. Meningitis, nephritis, or hepatitis may also be present at this stage. Both primary and secondary syphilitic lesions are infectious. The characteristic lesion of tertiary syphilis is the gumma (choice A), a granulomatous lesion often found in skin bones or liver. Symptoms of congenital syphilis include Hutchinson's teeth, mulberry molars, saddle nose (choice D), and interstitial keratitis.
The chemical nature of bacterial endotoxins is a
1. lipid.
2. protein.
3. complex carbohydrate.
4. lipopolysaccharide complex.
5. low molecular weight polypeptide.
Good Work
The correct answer is choice 4. Endotoxins are complex lipopolysaccharides derived from cell walls and liberated when the bacteria lyse. Endotoxins are taken up by reticuloendothelial cells or endothelial cells and are degraded or neutralized. Clinically, they lead to fever, leukopenia, and in serious cases, to hypotension, shock, intravascular coagulation and death. All other answer choices are incorrect as endotoxin is a lipopolysaccharide. It's not a lipid, protein, complex carbohydrate or low molecular weight polypeptide which would also be a protein like answer choice 2. So the correct answer to question is choice 4.
The earliest changes in rheumatoid arthritis occur in the
1. villi.
2. synovia.
3. attached ligaments.
4. articular cartilage.
5. subarticular bone.
Good Work
The correct choice is choice 2, synovia. Rheumatoid arthritis, or RA, is a systemic autoimmune disease involving primarily the joints, lungs, heart and blood vessels. The most common form of the disease is a crippling arthritis involving the proximal interphalangeal joints of the hands and feet. It affects approximately 3% of all women, but less than 1 % of men. In the early stages, the synovia become infiltrated with acute and chronic inflammatory cells. The synovial fluid contains an abundance of neutrophils. As the disease progresses, the synovia thickens and becomes vascularized. The number of chronic inflammatory cells increases dramatically and foci of necrosis can be found. This mass of vascularized inflammatory tissue is called the panus. The panus erodes into the adjacent articular cartilage, choice 4, and subsequently involves the supporting ligaments, choice 3. In the final stages, the panus erodes into bone, choice 5. Fibrous tissue replaces the joints and mobility is lost at that stage. Villi, choice 1, are not present in either the normal or diseased joint. Once again, the correct choice to question is choice 2.
The effectiveness of autoclaving is best determined by
1. culturing bacterial spores.
2. culturing the water reservoir.
3. thermocouples (temperature recording devices).
4. indicators that change color at elevated temperatures.
Good Work
The correct answer is choice 1.
Whether the autoclaving is effective or not depends primarily on whether all bacteria, fungi, virus and spores have been killed. Regardless of the temperature and pressures measured, it would still be an assumption to say that all living organisms are killed unless these organisms are tested for. In particular, spores are cultured because they require the most rigorous autoclaving conditions in order to be killed. If we do not find spores, we can safely assume that there are no bacteria, fungi or viruses as well. Choices 3 and 4 are incorrect as these devices might be malfunctioning, and are only indirect and not direct indicators of living organisms. Choice 2 is incorrect. We should culture an instrument or possible source of spores to see if they have been killed. It is possible that the autoclaving water might be sterile, but an instrument might contain spores if not sufficiently autoclaved. So the correct answer to question is choice 1.
The etiologic basis of Hashimoto's thyroiditis is
1. autoimmune.
2. bacterial infection.
3. hereditary enzyme deficiency.
4. premalignant diffuse hyperplasia.
5. secondary to hyperpituitarism.
Good Work
The correct answer is choice 1.
Hashimoto's thyroiditis is a chronic lymphocytic thyroiditis involving a goitrous enlargement of the gland due to lymphocytic and plasma cell infiltrates. It is the leading cause of hypothyroidism in the U.S. It is believed to have an autoimmune etiology. In this condition, thyroid-stimulating hormone or TSH is increased and, later, decreased T4 production occurs with accompanying malaise and fever. The gland is symmetrically swollen and firm. Choices 2, 3, 4 and 5 are therefore all incorrect as they are not etiologic factors in Hashimoto's thyroiditis. So the correct answer to question is choice 1.
The etiology of megacolon (Hirschsprung disease) is:
A. benign muscle neoplasm
B. malignant muscle neoplasm
C. imperforate anus
D. absence of ganglion cells
E. defective autonomic neurotransmitters
Good Work
The correct answer is choice D. Megacolon is discovered shortly after birth when the newborn exhibits vomiting, nausea, distention, and constipation. The newborn cannot eliminate feces (pass meconium). Microscopically, ganglion cells of Metssner and Auerbach plexi are missing. The muscle of the colon cannot relax between peristaltic contractions. Imperforate anus is another, unrelated congenital bowel condition in which no complete separation of tissues occurs to allow formation of an anal opening.
The first human cancer that has been strongly linked to a virus is
1. Hodgkin's disease.
2. multiple myeloma.
3. Burkitt's lymphoma.
4. acute lymphocytic leukemia.
5. chronic myelogenous leukemia.
Good Work
The correct choice is choice 3, Burkitt's lymphoma. Burkitt's lymphoma is a B-cell tumor which is the most common neoplasm in children in regions of the world where Epstein-Barr Virus (EBV) is endemic: These regions include New Guinea and Central Africa. It is thought that concomitant infection with malaria or another infectious agent in a host who has mononucleosis from EBV renders this host susceptible to developing Burkitt's lymphoma. This is a consequence of the immunosuppressive effect of malaria or some other agent. Immunosuppression prevents the host from clearing the Epstein-Barr virus infection. This DNA virus of the herpes family can then cause clonal expansion of B-cells as it replicates unhindered. Clonal expansion leads to an increased likelihood of producing a malignant strain of B-cells through random mutation. The mutation which most commonly gives rise to Burkitt's lymphoma involves the translocation of genes from chromosome 8 to chromosome 14. Other viruses which have been linked to human cancers include the human immunodeficiency virus which is associated with Kaposi's sarcoma, hepatitis B virus which may cause hepatocellular carcinoma, and herpes simplex I1 which may play an etiologic role in the development of cervical carcinoma. Human tumors of the lymphoid system other than Burkitt's lymphoma, such as Hodgkin's disease, multiple myeloma, acute lymphocytic leukemia, and chronic myelogenous leukemia have not been firmly linked to any specific viruses. In animals, however, many viruses have been discovered which produce lymphomas and leukemias. Examples include the gross leukemia virus which causes leukemia in mice and the feline leukemia virus which causes cancer in millions of domesticated cats each year. Once again, the correct choice to question is answer #3.
The immediate type of hypersensitivity in which histamine does NOT play a major role is
1. asthma.
2. urticaria.
3. anaphylaxis.
4. Arthus reaction.
Good Work
The correct answer is choice 4. Asthma, choice 1, is a type 1 IgE mediated hypersensitivity reaction. It involves histamine and leads to airway resistance, labored breathing and hypoxia. Urticaria, choice 2, is a rash or hives which accompanies IgE mediated hypersensitivity reactions. So histamine is again involved. Choice 3, anaphylaxis, is an acute type 1 hypersensitivity reaction involving IgE and histamine especially from a second exposure to an antigen. Choice 4, Arthus reaction refers to a hypersensitivity reaction caused by large numbers of antibody antigen complexes that fix complement, attract PMNs, and are phagocytized by them. The PMNs release lysosomal enzymes which can damage tissue. Histamine does not play a major role in this Arthus reaction. An example of an Anhus reaction is serum sickness. So the correct answer to question is choice 4.
The indiscriminate use of broad-spectrum antibiotics is contraindicated because they frequently
1. are extremely nephrotoxic.
2. cause psychogenic symptoms.
3. produce dependency reactions.
4. induce anaphylactoid reactions.
5. interfere with indigenous biota.
Good Work
The correct choice is choice 5. interfere with indigenous biota. Broad spectrum antibiotics should be used with extreme caution. Their utilization leads to the eradication of the normal flora colonizing many sites throughout the body. This is dangerous because the normal flora serves an important function. It holds the growth of organisms resinant to antibiotics in check. When the normal flora is cleared, these resistant organisms can replicate freely as competition for nutrients decreases. Many of these resistant organisms, particularly Pseudomonas, E. coli and Klebsiella species, cause sepsis which can only be treated with newer, more powerful antimicrobial agents. These agents select for resistant strains for which no antibiotic treatment is available. Some antibiotics such as the aminoglycosides are extremely nephrotoxic as stated in choice 1. Signs of nephrotoxicity should be monitored when utilizing these drugs, but fear of inducing nephrotoxicity should not inhibit the physician from using them when indicated. Psychogenic symptoms, choice 2, and dependency reactions, choice 3, are not problematic in antimicrobial therapy. Choice 4, anaphylaxis, does occur, however. It can occur with any drug, but is most common with the penicillins and cephalosporins. As with nephrotoxicity, the spread of anaphylaxis should not deter the physician when antibiotic treatment is indicated unless the patient has had a previous episode of anaphylaxis or allergy to the drug of choice. Once again, the correct choice to question is choice 5.
The likelihood that oral bacteria play an important role in gingival inflammation is evidenced by
1. an increase in salivary hyaluronidase.
2. an increased number of bacteria in saliva.
3. an increase in bacterial antibodies in saliva.
4. appearance of new pathogenic strains in the affected area.
5. a reduction in inflammation by removal of bacteria.
Good Work
The correct answer is choice 5.
Among the basic demonstrations that bacteria are involved in gingival inflammation would be that the bacteria are present when the inflammation occurs, that the inflammation would be greater with greater amounts of bacteria, and that the removal of the bacteria should reduce the inflammation. Therefore, choice 5 is logical and is our correct answer. Choice 1 is incorrect. Hyaluronidases are produced by bacteria to hydrolyze hyaluronic acid ground substance of connective tissue. Finding large amounts in the saliva might indicate increased bacterial numbers in the oral cavity, but not that these bacteria cause gingivitis. Likewise, choice 2 is incorrect for a similar reason. Increased number of bacteria in saliva does not show that these bacteria are rehponsible for gingival inflammation or caries or halitosis or anything else, only that their numbers are higher. Similarly, choice 3 is incorrect as increased antibodies probably show only increased numbers of bacteria which we already rejected as answer choice 2. Choice 4 is close but not good enough. While a change in bacterial types during inflammation might give some indication that the bacteria are involved, it is not as direct evidence as that shown in choice 5, where removing the bacteria reduces the inflammation. For example, the change in the bacteria strain might be a result of the gingival inflammation and not a cause of it. So the correct answer to question is choice 5.
The likelihood that oral bacteria play an important role in gingival inflammation is evidenced by which of the following?
A. An increase in salivary hyaluronidase
B. An increased number of bacteria in saliva
C. An increase of neutralizing antibodies in saliva
D. A reduction of inflammation with reduction of plaque
Good Work
The correct answer is choice D. Evidence that a certain condition is associated with a certain microorganism should include some or all of the conditions of Koch's postulates. For example, the organism should be present when the condition is present, it should be absent when the condition is absent, the condition should be worse when more of the organism is present, and improve when the organism is removed. This is consistent with choice D. Hyaluronidase is not a normal constituent of saliva. Increased numbers of bacteria in saliva may show something about the condition of the saliva, soft tissues, teeth, salivary glands or many other conditions. It would not necessarily show anything about the gingiva specifically. A similar argument holds for antibodies in saliva, as they would not be specifically connected to anything occurring within the gingiva. So the correct answer to question is choice D.
The majority of cases of pharyngitis are caused by
A. alpha-hemolytic streptococci.
B. Staphylococcus aureus.
C. a variety of viruses.
D. herpes simplex virus.
E. hemophilus influenzae.
Good Work
The correct answer is choice C. The majority of cases of pharnygitis are caused by a variety of viruses eg. picornaviruses. orthomyxoviruses, Epstein-Barr, etc ... Choice A - is incorrect - alpha-hemolytic streptococci is the most common cause of bacterial pneumonia, and also causes otitis media, sinusitis, bronchitis, and bacteremia. It is the most common cause of meningitis in the elderly. Choice B - is incorrect - Staphylococcus aureus causes skin infections, (eg. cellulitis) deep infections (e-g. bone), systemic infections secondary to deep infections, and toxin mediated disease (eg toxic shock syndrome). Choice D - is incorrect - Herpes simplex virus causes oral and genital lesions by infecting epithelial cells. Upon resolution of acute illness, latent infections are commonly found in neurons. Choice E - is incorrect - Hemophilus influenzae causes meningitis, otitis media, and epiglottis. Primarily children under the age of five are affected. Therefore the correct answer to question is C.
The most characteristic finding in intestinal malabsorption syndrome is
1. pain.
2. melena.
3. anorexia.
4. diarrhea.
5. steatorrhea.
Good Work
The correct choice is choice 5, steatorrhea. Intestinal malabsorption can result from a wide variety of causes. The intestinal mucosa may be defective, such as in intestinal lymphoma or Crohn's disease, or it may be covered by bacteria such as in bacterial overgrowth syndrome. Alternatively, organs participating in digestion may be defective. The pancreas does not secrete normal amounts of digestive enzymes in pancreatitis or cystic fibrosis. In choledocholithiasis, the gall bladder cannot secrete bile salts into the intestine due to obstruction of the common bile duct. In hepatitis, synthesis of bile salts is impaired. Whatever the cause of malabsorption, steatorrhea often results. Steatorrhea literally means "fatty stool." Fat which is not absorbed gets passed in the feces. The stool floats and has a greasy appearance. Pain, choice 1, may be associated with malabsorption when the disease causing malabsorption also causes pain. Crohn's disease, for example, is especially painful. However, many malabsorption syndromes are not painful. Melena, choice 2, is the passage of black stool. The stool is black due to the presence of digested blood. Melena is the marker of an upper GI bleed. It is not associated with malabsorption. Anorexia, choice 3, is not commonly seen in malabsorption. In fact, individuals with malabsorption may exhibit hyperphagia as they are nutritionally deprived secondary to a poorly functioning gut. Diarrhea, choice 4, occurs when the malabsorption syndrome involves the large bowel or when malabsorption is secondary to increased transport of food through the gastrointestinal tract. However, many malabsorption syndromes are not characterized by diarrhea. In addition to steatorrhea, other common symptoms in malabsorption include wasting, weakness, tetany and easy bruisability. Once again, the correct choice to question is choice 5.
The most common cause of a massive hematemesis in alcoholics is
1. peptic ulcer.
2. acute gastritis.
3. esophageal varices.
4. Mallory-Weiss syndrome.
5. acute hemorrhagic pancreatitis.
Good Work
The correct answer is choice 3. Esophageal varices are commonly found in alcoholics. They are large, swollen veins in the esophagus which are easily and often ruptured, leading to massive bleeding or hematemesis. The varices are due to poor portal circulation due to cirrhotic liver damage, followed by drainage of the portal blood through coronary veins of the stomach and the esophageal veins. Choice 1, peptic ulcer, may cause bleeding, but not usually of a massive type and it is not strongly associated with alcoholic patients. Likewise, acute gastritis, choice 2, may be related to alcohol, but has many other etiologies including aspirin, smoking and stress. It is also less likely to lead to massive bleeding. Choice 4 is close to correct. Mallory-Weiss syndrome refers to small tears in the gstroesophageal junction secondary to recurrent forceful vomiting. Although it is often found in alcoholics and can lead to hematemesie, it is not as common among alcoholics as are esophageal varices, the correct choice. Choice 5, acute hemorrhagic pancreatitis, refers to release of enzymes within the pancreas and is common in alcoholics. It often results in severe pain, vomiting and fever, but not usually in hematemesis. So the correct answer to question is choice 3.
The most common clinical consequence of benign prostatic hyperplasia is
1. pain.
2. jaundice.
3. urinary tract obstruction.
4. red blood cell casts in urine.
5. progression to prostatic carcinoma.
Good Work
The correct answer is choice 3. Benign prostatic hyperplasia involves nodular growth within the prostate which usually leads to urinary tract obstruction, which is choice 3. It is currently believed that benign hyperplasia does not predispose the individual to prostatic carcinoma, so choice 5 is incorrect. Choice 1 is incorrect. Many cases of prostatic hyperplasia are asymptomatic. Choice 2 is incorrect as there is no connection between the prostatic hyperplasia and jaundice, which is a general skin discoloration usually associated with liver dysfunction and excess bilirubin. Choice 4 is incorrect as red cell casts in the urine are not associated with prostatic hypertrophy. So the correct answer to question is choice 3.
The rickettsial disease that may have oral manifestations is
NBDE Part I released questions - Microbiology-Pathology
The rickettsial disease that may have oral manifestations is
1. rickettsialpox
2. Brill's disease.
3. epidemic typhus.
4. none of these.
Good Work
The correct answer is choice 1. Rickettsia are gram negative coccobacilli and are most famous for causing rocky mountain spotted fever and typhus. Our correct answer, choice 1, rickettsial pox is spread by mites on house mice. Clinically, it involves a vesicular papular rash which may spread to the lips and buccal mucosa. Lymphadenopathy is another common clinical sign. Choice 2, Brill's disease, is a disease found in patients who formerly had epidemic typhus. It is milder without skin rash and does not have any oral symptoms. Choice 3. epidemic typhus, is spread by the human body louse and signs and symptoms include fever, chills, myalgia and headache followed hy cutaneous erythema but it does not spread orally. So the correct answer to question is choice 1.
The renal lesion most commonly associated with benign hypertension is
1. renal atresia.
2. acute pyelonephritis.
3. chronic pyelonephritis.
4. arteriolonephrosclerosis
Good Work
The correct answer is choice 4. Arterionephrosclerosis refers to thickening or hyalinization of renal arterioles. It is caused by mild hypertension and does not normally impair renal function. Renal atresia, choice 1, refers to a congenital absence or closure of arteries to a kidney. Choice 2 and 3, pyelonephritis, refers to infection of the renal pelvis, tubules or interstitium often from gram- negative bacteria from the patient's fecal flora. In chronic pyelonephritis, some scarring may occur and may be associated with hypertension. However, the association of arterionephrosclerosis and benign hypertension is much closer, so the correct answer to question is choice 4.
The principal antibacterial action of the tetracyclines is inhibition of
1. DNA synthesis.
2. protein synthesis.
3. histone formation.
4. cell wall synthesis.
5. cell membrane function.
Good Work
The correct answer is choice 2.
Tetracyclines act by inhibiting binding of the aminoacyl-tRNA to the 30S subunit of the bacterial ribosome. Thus, protein synthesis is prevented. Tetracycline has no other effects listed in choices 1, 3, 4 and 5. However, as typical examples, penicillins prevent cell wall synthesis, choice 4, as do cephalosporins. Erythromycins interfere with protein synthesis as do clindamycin and lincomycin. Polymyxins coat bacterial cell membranes and destroy active transport, choice 5. Some antiviral drugs, such as acyclovir, can interfere with DNA replication, choice 1. Anyway, the correct answer to question is choice 2.
The primary sources of pathogenic microorganisms in the dental operatory are
1. light handles.
2. dust particles.
3. patients' charts.
4. the mouths of patients.
5. the hands of clinicians.
Good Work
The correct answer is choice 4. In this question, there are a number of possible sources of pathogenic organisms. However, one is by far the greatest source, the human mouth. A large variety of bacteria, viruses, fungi and other microorganisms exist in the human mouth within the saliva, on teeth surfaces or within the blood or mucosa. They could include Staphylococci, spirochetes, hepatitis-B virus and many others. Choice 1 is incorrect. While some pathogens from a patient might survive temporarily on a light handle, it would not be a primary source of pathogens. The same argument holds for Choice 3, patient's charts. However, it is even less likely that many pathogens will survive for very long on a chart. Choice 2 is incorrect. Few if any human pathogens are found on dust particles. Choice 5 is incorrect. Some pathogens could be found on a clinician's hands if they are not washed and gloved between patients. However, the source of the pathogens is likely to be a patient's mouth. So we are back again to choice 4, the correct answer to question.
The oral vesicular lesions of herpangina and hand-foot-and-mouth disease are caused by
1. coxsackievirus.
2. Epstein-Barr virus.
3. herpes simplex type I.
4. varicella-zoster virus.
5. respiratory syncytial virus.
Good Work
The correct answer is choice 1. Herpangina is a disease caused by coxsackievirus . It is a self limited disease and involves abrupt onset of fever, sore throat and anorexia. Hand, foot and mouth disease is caused by coxsackievirus as well and involves ulcerations of the mouth and pharynx and rash of the hands, arms, feet and legs. Epstein Barr virus, choice 2, is a causative agent of infectious mononucleosis and is also associated with development of Burkitt's lymphoma. Herpes simplex type 1 is a causative agent for herpes labialis as well as acute herpetic gingivostomatitis, keratoconjunctivitis and a form of encephalitis. Varicella zoster virus, choice 4, is a causative agent of varicella, or chicken pox and also zoster or shingles. Respiratory syncytial virus, choice 5, is the causative agent of respiratory infections, especially of the upper respiratory tract. So the correct answer to question is choice 1.
The most likely source of bacteria found in diseased periodontal tissue is
1. serum.
2. saliva.
3. subgingival plaque.
4. supragingival plaque.
Good Work
The correct answer is choice 3. It is widely accepted that the major cause of periodontal disease and tissue destruction is the bacteria found in subgingival plaque and calculus. The bacteria in subgingival plaque can become incorporated in the calcified plaque known as calculus and cause destruction of the periodontal ligament and surrounding bone. Choice 1 , serum, is incorrect. Serum refers to blood plasma minus fibrinogen-clotting protein. Serum would not normally contain any bacteria as it is generally sterile except during cases of a bacteremia where bacteria may enter the body and temporarily travel through the bloodstream. Choice 2 is incorrect. While some bacteria exist in saliva, they are usually derived from the bacteria present on teeth and mucosal surfaces andlor bacteria ingested with food. Large numbers of bacteria and saliva do not normally enter the periodontia. Choice 4 is incorrect. Supragingival plaque, plaque formed above the gumline, is generally of a different composition than subgingival and does not lead to subgingival calculus formation and periodontal destruction. It is often involved in caries formation however. So the correct answer to question is choice 3.
The most likely diagnosis for a patient with dysphonia, dysphagia, weight loss and a history of heavy cigarette smoking is
1. laryngitis
2. tonsillitis.
3. laryngeal polyps.
4. carcinoma of the lungs.
5. carcinoma of the larynx.
Good Work
The correct answer is choice 5. Dysphonia refers to difficulty in producing sounds and indicates involvement of the vocal cords or larynx. Dysphagia refers to difficulty in swallowing. The accompanying weight loss and history of cigarette smoking are a both suggestive of the presence of a cancer of the respiratory tract. However, the dysphonia suggests cancer of the larynx and not the lungs. Choices 1 and 2, laryngitis and tonsillitis are both inflammatory processes. However, they are not necessarily associated with weight loss and history of heavy cigarette smoking. While they might be part of a differential diagnosis, a carcinoma should be suspected first. Tonsillitis would also be likely to be accompanied by fever. Choice 3, laryngeal polyps, are inflammatory overgrowths due to chronic irritation, but would not be associated with dysphagia or weight loss, and do not progress to malignancy. So the correct answer to question is choice 5.
The MOST important viral cause of gastroenteritis in children less than 2 years old is
A. rotavirus
B. echovirus
C. rhinovirus
D. coxsackievirus
E. cytomegalovirus
Good Work
The correct answer is choice A. Rotavirus is the major viral cause of gastroenteritis in children less than 2 years old. The disease is seasonal and classically occurs in the fall and winter months. Choice B - Echovirus presents clinically as aseptic meningitis, fever, rash, enteritis, common colds and/or acute hemorrhagic conjunctivitis. Less common symptoms include paralysis, pleurodynia, encephalitis, myocarditis, and respirtory illness. Choice C - Rhinovirus is most commonly associated with the common cold. Clinical manifestations include upper respiaratory tract irritation, headache, nasal discharge, cough, malaise, chills, and myalgia. Choice D - Coxsackievirus Type A may cause herpangina, with headache, sore throat, dysphasia, stiff neck, fever, anorexia, and abdominal pain. Discrete vesicles are seen in the oropharynx. It is also the etiologic agent of hand-foot-and-mouth disease. Type B may cause myocarditis, pericarditis, and pleurodynia. Both types may cause meningitis in humans. Choice E - Cytomegalovirus elicits a mononucleosis illness including fatigue, malaise, tender lymphadenopathy, pharyngitis, fever, headache, and splenomegaly. The majority of these infections are subclinical (with no overt symptoms) but may lead to life-long latent infection. In immunosuppressed patients infection is more severe and marked by fever, adenopathy, leukopenia, hepatosplenomegaly, and myalgias. Therefore the correct answer to question is A.
The MOST important characteristic of malignant neoplasms, which distinguishes them from benign neoplasms, is their
A. nonencapsulation.
B. rapid growth rate.
C. ability to metastasize.
D. lack of differentiation.
E. excessive mitotic activity.
Good Work
The correct answer is choice C. All of the factors listed are related to tumor growth. Benign tumors tend to be defined in shape and capsulated. By contrast, malignant growths are more formless without distinct boundaries. Both benign and malignant tumors may have a fast rate of growth, although benign tumors usually grow slower. Malignant growths, in general, show a lack of differentiation. The cells in the neoplasm do not form well-specialized tissues. Excessive mitotic activity is associated with neoplasms in general. The rapid growth of undifferentiated tissue requires cell mitosis. However, mitotic rate is much greater in the malignant growth as compared to that in the benign neoplasm. The major difference between benign and malignant tumors and the most important to the prognosis for the patient is the ability of the malignant tumor to spread (metastasize) to other parts of the body through blood or lymph channels. As a result, secondary tumors will grow, which makes the prognosis for the patient inevitably worse. So the correct answer to question is choice C.
The most frequent complication following a single transfusion of blood that was inadequately cross matched is
1. leukopenia.
2. infectious hepatitis.
3. altered clotting time.
4. agglutination of the recipient's erythrocytes.
5. agglutination of the donor's erythrocytes.
Good Work
The correct choice is choice 5, agglutination of the donor's erythrocytes. Fatal transfusion reactions most commonly occur when a clerical error is made and blood is given to the wrong patient. On rare occasions, the blood is inadequately cross-matched and the recipient possesses undetected antibodies directed against antigens on the donor's red blood cells which are capable of causing agglutination of donor cells. Agglutination activates complement and hemolysis results. The patient may then progress to acute renal failure which is often irreversible. Leukopenia, choice 1, does not occur in transfusion reactions. Infectious hepatitis, choice 2, is a very common complication of transfusion. Non-A/non-B hepatitis is transmitted through transfusion and 5-10% of transfused individuals will become infected. Transfusion hepatitis, however, cannot be prevented by cross-matching procedures. Improperly and properly cross-matched blood carry the same risk of hepatitis transmission. Altered clotting time, choice 3, is a late manifestation of some severe transfusion reactions. It results when the patient develops disseminated intravascular coagulopathy and is a pre-terminal event. An altered clotting time may also accompany transfusion hepatitis, but as mentioned, hepatitis does not result from improper cross-matching. Agglutination of recipient's erythrocytes, choice 4, does not occur to a significant extent when packed red blood cells are given. Packed red blood cells contain little or no donor antibody. Agglutination of recipient cells may occur, however, when inadequately cross-matched plasma is given and donor antibodies against recipient red blood cells are introduced. Once again, the correct choice to question is choice 5.
The most common type of epithelial metaplasia involves
1. regeneration of epithelium in an area of cutaneous ulceration.
2. replacement of squamous cells by cuboidal cells.
3. replacement of cuboidal cells by columnar cells.
4. replacement of columnar cells by stratified squamous epithelium.
Good Work
The correct answer is choice 4. Metaplasia involves the abnormal replacement of one type of normal cell or tissue by a different type of normal cell or tissue. In this case, choice 1 is incorrect as it does not involve replacement of one normal tissue by a different one. However, choices 2, 3 and 4 all look possible. Choice 4 is correct because it occurs fairly commonly. Ciliated columnar epithelial cells in the trachea can become damaged by smoke or other chemicals and often, the extended chronic exposure to the irritant will cause the tracheal lining to become squamous rather than ciliated columnar. So the correct answer to question is choice 4.
The most common primary bronchogenic carcinoma is the:
A. small cell carcinoma
B. large cell carcinoma
C. squamous cell carcinoma
D. adenocarcinoma
E. oat cell carcinoma
Good Work
The correct answer is choice C. The squamous cell carcinoma composes approximately 35-50% of primary bronchogenic carcinomas. It is highly correlated with smoking, metastasizes later than other types, but grows larger in its primary area. Second and third in frequency (depending on report) are the small cell carcinoma (choice A) and adenocarcinoma (choice D). Note that the small cell is also known as oat cell (choice E) due to the small cell appearance with prominent nucleus and little cytoplasm. These are aggressive tumors and very malignant. They often secrete hormones and cause paraneoplastic syndromes. Large cell carcinomas (choice B) are the rarest (approximately 10%) but have a very poor prognosis.
The most common malignant neoplasm of the stomach is:
A. leiomyoma
B. gastric polyps
C. sarcoma
D. carcinoma
E. Krukenberg tumor
Good Work
The correct answer is choice D. Benign neoplasms of the stomach include leiomyoma (choice A), a benign muscle tumor, and gastric polyps (choice B), which are outgrowths of gastric mucosa. Adenomatous type polyps have high malignant potential, however. Carcinoma is the most common malignant tumor of the stomach. Histologically, they are composed of gastric and intestinal mucus-secreting cells. Prognosis for the condition is poor due to the late occurrence of symptoms. Carcinoma of the stomach spreads through the lymphatics. Sarcoma of the stomach (choice C) is rare. Carcinoma of the stomach can metastasize to the ovary and form the Krukenberg tumor (choice E).
Which chemical substance is usually secreted by pheochromocytomas?
A. Catecholamine
B. Aldosterone
C. Cortisone
D. Insulin
E. Renin
Good Work
The correct answer is choice A. A pheochromocytoma is a tumor which produces catecholamines (epinephrine and norepinephrine). This is due to the fact that the tumor is found in the adrenal medulla, which normally produces these compounds. None of the other hormones listed are from the adrenal medulla. Note that the oversecretion of catecholamines results in severe hypertension. So the correct answer to question is choice A.
Which acidogenic bacteria often causes significant dental decay in deep established dental caries?
A. Bacteroides
B. Strep. mitis
C. Lactobacillus
D. Actinomyces
E. Strep. sanguis
Good Work
The correct answer is choice C. Lactobacillus is an aciduric bacteria which produces lactic acid as a metabolic waste product. It lowers the pH in the immediate environment to the extent that most other bacterial species cannot survive. It is considered to be a late colonizer of carious lesions which are initiated and also continued by aciduric Strep. mutans. In particular, Lactobacillus is found deep within the lesion. Bacteroides is an anaerobe found in the gingival sulcus and asssociated with chronic adult periodontitis (especially B. melaninogenicus). Strep. mitis, Strep salivanus and Strep. sanguis are all Viridans streptococci, and are common oral flora. Actinomyces are also common oral flora. They are gram-positive rods which can cause infections of oral tissue. A characteristic lesion will form in the mouth and drain to the face. Material which resembles "sulfur granules" (but is not!) is found in the exudate from these fistulas.
When one cell type or tissue is replaced by another type that is not normally located at that place, this process is known as:
A. carcinoma in situ
B. anaplasia
C. metaplasia
D. dysplasia
E. hyperplasia
Good Work
The correct answer is choice C. In metaplasia, changes in conditions or local environment cause the change of one tissue type to another. In smokers, a common metaplasia occurs in bronchi and trachea, where the normal pseudostratified ciliated columnar epithelium is replaced by nonciliated stratified squamous epithelium. This is sometimes viewed as a precursor to cancer development. Anaplasia (choice B) refers to loss of cell differentiation. Dysplasia (choice D) refers to abnormal cell proliferation. Hyperplasia (choice E) refers to growth of a large number of cells; however, the cells are generally normal. Carcinoma in situ (choice A) is an epithelial cancer that has not yet invaded connective tissue and resides completely above the basal lamina.
Viral species can be partially identified through the use of any of the following characteristics EXCEPT:
A. ability to pass through filter pores
B. susceptibility to ether
C. type of genome material
D. microscopic appearance
E. ability to grow on various media
Good Work
The correct answer is choice E. As obligate intracellular parasites, viruses must be grown in cell culture and cannot be grown on media. They can be partially identified by filtration (gives size estimate), appearance under electron microscope, whether they contain RNA or DNA (nucleic acid identification), and sensitivity to solvents and chemicals (especially ether). Naked viruses often are more resistant to ether, whereas enveloped viruses are more susceptible to ether damage.
Vancomycin is an antiinfective agent indicated for the treatment of potentially life threatening infections caused by gram positive bacteria. Vancomycin would most likely be used to treat an infection caused by
A. Candida albicans
B. Enterococcus faecium
C. Escherichia coli
D. Klebsiella spp.
E. Pseudomonas aeruginosa
Good Work
The correct answer is B. Vancomycin is an antiinfective agent indicated for the treatment of potentially life threatening infections caused by gram positive bacteria, especially gram positive cocci. This agent is primarily used to treat infections caused by Enterococci (Enterococcus faecium) (choice B), streptococci, and staphylococci, including methiciilin resistant staphylococci. Candida albicans (choice A) is a fungus and is frequently the causative organism in vaginal yeast infections and oral thrush. Since Escherichia coli (choice C), klebsiella (choice D) and Pseudomonas aeruginosa (choice E) are gram-negative organisms, this agent would be ineffective in the treatment of infections caused by these bacteria.
Use of vaccines for preventing clinical symptoms after introduction of the virus is most likely to be effective against
1. rabies.
2. influenza.
3. poliomyelitis.
4. herpes zoster.
Good Work
The correct answer is choice 1.
Rabies is an acute viral infection of the central nervous system. Rabies virus remains latent in tissues for some time after the virus is introduced from a bite. Therefore, a vaccine can be administered and antibodies will form before the virus invades the CNS or central nervous system. In most cases, passively administered antibodies are also introduced. These antibodies, produced from, human or equine blood, provide additional time for the vaccine to stimulate active antibody production before the CNS is invaded. This type of post-infection vaccination is not possible against influenza, polio or herpes zoster because the viruses do not have the long latent period mentioned previously for rabies virus. So the correct answer to question is choice 1.
Two important factors for initiation of caries by oral streptococci are
1. production of protease and production of acid.
2. production of dextranase and production of soluble dextran.
3. production of collagenase and production of hyaluronidase.
4. fermentation of mannitol and sorbitol and production of protease.
5. synthesis of insoluble dextran and production of glucosyltransferase.
Good Work
The correct choice is 5, synthesis of insoluble dextran and production of glycosyl transferase. Of the Streptococci which colonize the oral mucosa, Strep. mutans has been most consistently correlated with the development of dental caries. This organism is an effective pathogen due to its unique capacity to bind to teeth. This is accomplished with the use of glycosyl transferase, an enzyme projecting from the outer membrane of the bacteria. Glycosyl transferase polymerizes the glucose moiety from sucrose to form dextran. Dextran is an insoluble adherent substance which promotes the accumulation of bacteria on the tooth enamel. This substance also prevents the diffusion of organic acids produced by the bacteria. As a consequence, protons accumulate in the dextran bacteria mass, also called plaque, and the pH of the environment decreases. The acid erodes the tooth enamel and caries result. Choice 1 is incorrect because proteases are not involved in this process. Proteases are enzymes which degrade proteins. Tooth enamel is primarily mineral with little protein content. Choice 2 is the wrong answer because, as mentioned, dextran is insoluble. Dextranase, an enzyme which degrades dextran, would impede the development of caries, not promote it, as dextran plays a fundamental role in the pathogenesis of caries. Collagenase and hyaluronidase, enzymes referred to choice 3, degrade collagen and hyaluronic acid, respectively. These substances are components of connective tissue, not tooth enamel. Consequently, these enzymes bear no relevance to this discussion. Choice 4 is incorrect for the same reason choice 1 is incorrect; namely, proteases play no role in the development of caries. Streptococci derive their energy from the fermentation of sugars to produce lactic acid and ATP. They may possess a capacity to ferment mannitol and sorbitol as stated in choice 4, but this capacity is important only in the identification of these organisms, not in understanding their pathogenicity. Once again, the correct answer for question is choice 5.
The term "biofilm" as it applies in dentistry is most often used to describe:
A. A safer type of x-ray film
B. A detector that protects tissue from x-ray damage
C. The surface of dentin inside a cavity preparation
D. The surface of dental waterlines
E. None of the above
Good Work
The correct answer is choice D. The problem of bacteria in dental waterlines is probably going to be the next intensive infection control issue faced by dentistry. The narrow bore of most dental waterlines encourages a thin film colony of bacterial slime (biofilm) to form on the inner circumference of the waterline. Water entering the dental unit, although chlorinated at the source, is clean, but not sterile. The microorganisms it contains can form extensive colonies inside the tubes. While no specific disease cases have been linked to these microorganisms, it is clear that they should be minimized. Several systems of filtration or chemical disinfection are now available. Other alternatives include bottled irrigation water and flushing the lines for several minutes several times each day.
Which of the following describes the function of adjuvants?
A. Enhance secretion of IgA
B. Enhance antibody response
C. Stimulate complement synthesis
D. Desensitize to a given antigen
E. Activate mast cell degranulation
Good Work
The correct answer is choice B. Adjuvants are chemicals, usually proteins, which enhance immune response. For example, if you wish to induce delayed hypersensitivity in an experimental animal, a protein antigen is mixed with an adjuvant to obtain a greater response. The adjuvant is sometimes dead tubercle bacilli in oil, or another organism, such as Nocardia. Research is taking place to identify safe adjuvants for use in humans, for example, to increase immune response against tumors. So the correct answer to question is choice B.
Which of the following conditions results in the development of a normocytic, normochromic anemia?
A. Acute blood loss
B. Folic acid deficiency
C. Iron deficiency
D. Sickle cell disease
E. Thalassemia
Good Work
Anemias are often classified according to the morphology of the red blood cell as well as the etiology of the anemia. With respect to the morphologic classification of red blood cells, "cytic" refers to the size of the red blood cell and chromic refers to the color. For example, acute blood loss (choice A) will typically result in a normocytic, normochromic anemia. In other words, the red blood cells are all "normal"; however, there are not enough of them to meet the needs of the body. Folic acid (choice B) and vitamin B12 deficiency will lead to the development of a macrocytic, normochromic anemia. In a microcytic, hypochromic anemia, the red blood cell is smaller than normal and is lacking color. This type of anemia typically results from an iron deficiency (choice C). As mentioned before anemias can also be classified by their etiology. The major causes of these types of anemias are increased blood cell loss and decreased or defective red blood cell production. For example, sickle cell disease (choice D) causes hemoglobinopathies with an inherited abnormal hemoglobin. Thalassemia (choice E) is caused by an impaired globin synthesis.
Which of the following conditions predisposes to lung cancer by causing squamous metaplasia of bronchial epithelium?
A. Bronchiectasis
B. Bronchial asthma
C. Chronic bronchitis
D. Bronchial carcinoid
E. Pulmonary emphysema
Good Work
The correct answer is choice C. In chronic bronchitis, especially among smokers, normal bronchial epithelium of the pseudostratified ciliated columnar type may be replaced by stratified squamous. In addition to the loss of protective function of the mucus and cilia, this change is also often precancerous. Bronchiectasis is not precancerous and refers to the abnormal dilatation of the bronchi following chronic infection. Bronchial asthma is not precancerous and refers to narrowing of the bronchi and excess mucus production causing difficulty in breathing. Bronchial carcinoids are benign lesions of neuroendocrine argentaffin cells of bronchial mucosa. Pulmonary emphysema is not precancerous and involves distention of air spaces with destruction of alveoli. So the correct answer to question is choice C.
Which of the following conditions increase the risk of developing osteosarcoma?
A. Osteomalacia
B. Osteoporosis
C. Osteoblastoma
D. Osteitis deformans
E. Osteogenesis imperfecta
Good Work
The correct answer is choice D. Osteitis deformans, or Paget's disease of bone is a condition of unknown origin in which bone is replaced by disorganized soft matrix. Multiple areas of soft bone, changed size and shape of bone and bone pain result. Deformity and fracture are common. About 1% transform to malignant neoplasm(osteosarcoma). All four other choices are not associated with malignant transformation. Osteomalacia is a condition of soft bones subsequent to vitamin D deficiency. There is impaired mineralization of osteoid. Osteoporosis is weakening of bones due to loss of bone mass. It is especially common in older women, and may be related to estrogen deficiency, malnutrition or immobilization. Osteoblastoma is a usually benign tumor of bone that occurs at the diaphysis of long bones. It is related to osteoid osteoma. In cases where it is malignant, it does not develop into osteosarcoma. Osteogenesis imperfecta is an inherited condition of collagen synthesis. It involves thin, fragile bones, small discolored teeth and blue sclera. So the correct answer to question is choice D.
Which of the following characterizes victims of fatal, acute carbon monoxide poisoning?
A. Cherry red blood
B. Acute renal failure
C. Massive liver necrosis
D. A hypercoagulability state
E. Anemia and generalized white cell depletion
Good Work
The correct answer is choice A. Carbon monoxide binds strongly to hemoglobin, even more strongly than oxygen does. It also bonds in a very slowly reversible way, so that hemoglobin bound to carbon monoxide stays bound for long periods of time. This hemoglobin has a distinct cherry red color. Carbon monoxide poisoning results in oxygen starvation to all tissues, but starting with such oxygen-sensitive organs as the brain. Findings include CNS hyperemia, edema, focal hemorrhages and degeneration of basal ganglia. None of the other choices listed are characteristic of acute CO poisoning. So the correct answer to question is choice A.
Which of the following cells is responsible for secreting antibodies during an infectious process?
A. Lymphocytes
B. Megakaryocytes
C. Plasma cells
D. Polymorphonuclear neutrophils
E. Prostaglandins
Good Work
The correct answer is C. Lymphocytes are the immature cells that become plasma cells (choice A). Lymphocytes, known as B cells, are responsible for humoral immunity. These activated B cells differentiate into plasma cells (choice C), which are tissue cells that secrete antibodies to attack alien cells. Megakaryocytes (choice B) are the immature cells that become platelets, which clump together and stick to vessel walls following a traumatic event. Neutrophils (polymorphonuclear neutrophils) (choice D) and macrophages are cells of the immune system that "ingest" foreign particles by flowing their cytoplasm around the foreign object. The object is then internalized in the phagosome, which is an internal envelope of cell membrane. Finally, the digestive enzymes of the lysosomes are released into the phagosome to destroyldigest the foreign object. Prostaglandins (choice E) are produced in most tissues in the body and are responsible for coordinating local cellular activities.
Which of the following cells are increased in number in the bloodstream in allergy or parasitic infection?
1. Basophils
2. Eosinophils
3. Plasma cells
4. Atypical lymphocytes
5. Nucleated erythrocytes
Good Work
The correct choice is choice 2, eosinophils. Eosinophils are elevated in the peripheral bloodstream in drug reactions and in five categories of diseases: neoplasm, allergy, atopy, collagen vascular disorders and parasitic infection. Their function is to limit the cascade of events initiated by the degranulation of mast cells and basophils. They also can secrete compounds which are toxic to cell membranes of parasites. Basophils, choice 1, are increased in polycythemia vera, myelofibrosis, and chronic myelogenous leukemia. Plasma cells, choice 3, are not found in the peripheral blood. Their numbers are increased in the bone marrow, however, in multiple myeloma. Atypical lymphocytes, choice 4, are present in high numbers in the peripheral blood in leukemias and in mononucleosis. Nucleated erythrocytes, choice 5. are present peripherally when erythropoiesis is accelerated. This occurs in acute hemolysis or hemorrhage. Once again, the correct choice to question is choice 2.
Which of the following bacteria has the highest lipid content in the cell wall?
1. Escherichia coli
2. Lactobacillus casei
3. Leptotrichia buccalis
4. Staphylococcus aureus
5. Mycobacterium tuberculosis
Good Work
The correct answer is choice 5. The Mycobacteria are rod-shaped, non-spore forming, aerobic bacteria. They are difficult to stain, but once stained, they resist decolorizing and so are known as the acid-fast bacilli. They include Mycobacterium tuberculosis, which causes tuberculosis. The cell walls of Mycobacteria are especially rich in lipids, including fatty acids and waxes. Choice 1 is incorrect. Escherichia coli is a member of the normal flora of the intestinal tract and is a gram-negative rod. Choice 2 is incorrect. Lactobacillus casei is a gram-positive rod, also a member of the normal intestinal flora. Choice 3 is incorrect. Leptotrichia buccalis is a spirochete. Choice 4 is incorrect. Staphylococcus aureus is a gram-positive coccus and a member of the skin flora. So the correct answer to question is choice 5.
Which of the following are most characteristic of cervicofacial actinomycosis?
1. Multiple ulcers
2. Enlarged cervical lymph nodes
3. Multiple cutaneous abscesses with sinuses
4. Indurated nodules in the skin of the neck
Good Work
The correct answer is choice 3. Cervicofacial actinomycosis is caused primarily by Actinomyces israeli, an actinomycete. These are non spore forming, non acid fast facultative anaerobes. Actinomycosis often develops following dental caries and extractions. Multiple pyogenic abscesses develop with characteristic sinus tracts filled with yellow appearing material known clinically as "sulfur granules." Osteomyelitis can also be associated with Actinomycosis. So choice 3 is correct. Actinomycosis can spread to the thorax by extension or aspiration. Choice 1 is incorrect. Multiple abscesses, not ulcers, are common. Choice 2 is incorrect. While cervical lymph nodes may be swollen due to Actinomycosis, swollen lymph nodes would not be characteristic of the condition. Cervical lymph nodes may be swollen due to a large number of bacterial or viral conditions. Choice 4 is incorrect as indurated nodules in the neck are not common in Actinomycosis. So the correct answer to question is choice 3.
Which of the following are MOST antigenic?
A. Lipids
B. Haptens
C. Proteins
D. Nucleic acids
E. Carbohydrates
Good Work
The correct answer is choice C. Antigenic substances will produce an immune response. Proteins are, by far, the most antigenic substances. They include proteins, glycoproteins, lipoproteins and nucleoproteins. However, let's review the other choices. Lipids are not usually antigenic. Haptens are small molecules which contain antibody binding sites, but do not cause an immune response unless they are linked to a protein. In that case, two types of antibodies are formed. One binds to the protein, and the other to the hapten. Nucleic acids, like lipids, are not usually antigenic unless bound to a protein. Some large polysacharides(carbohydrates) can activate B cells, and so may be antigenic in some cases. However, proteins are the most antigenic of all molecule classes. So the correct answer to question is choice C.
Which of the following are clinical signs/symptoms of hepatitis?
A. Weight loss
B. Jaundice
C. Hepatomegaly
D. Myalgia
E. All of the above
Good Work
The correct answer is choice E. Hepatitis is a general term, meaning inflammation of the liver. It is often of viral origin (Hep A, B, C, D, etc.), but can have other origins such as chemical (CC14, etc.). Due to the general damaging effects to the liver, signs and symptoms are many and varied. Many can be logically predicted. They include hepatomegaly (enlarged liver), jaundice (yellow color due to back- up of bile pigments into the blood), fever, malaise, chills, nausea, myalgia (muscle pain), lymphadenopathy, darkened urine (due to increased bilirubin), weight loss, joint pain etc. Many symptoms are non-specific, and a careful history, combined with lab tests, is necessary for diagnosis.
Which form of viral hepatitis does NOT have a carrier state?
A. Hepatitis A
B. Hepatitis B
C. Hepatitis C
D. All viral hepatitis types have carrier states
Good Work
The correct answer is choice A. Hepatitis A (short incubation, infectious hepatitis) does not have a carrier state. Infection by the virus induces a recovery followed by lifelong immunity. Hepatitis B (long incubation, serum hepatitis) has a carrier state and asymptomatic carriers can transmit the disease. The estimated incidence of transformation to the carrier state for hepatitis B is 5-10%. Hepatitis C (NANB) also has a carrier state, with the highest incidence of transformation to carrier estimated at 30-40%.
Which of the following factors has the greatest impact on prognosis of a skin melanoma?
NBDE Part I released questions - Microbiology-Pathology
Which of the following factors has the greatest impact on prognosis of a skin melanoma?
1. Site of origin
2. Depth of invasion
3. Degree of pigmentation
4. Existence of ulceration
5. Size of surface area involved
Good Work
The correct answer is choice 2. Skin melanoma is a tumor of melanocytes. The most important factor in prognosis of this tumor is depth which is staged according to a Clark level 1-5, ranging from level 1, confined to the epidermis, to level 5, invading the subcutaneous fat. Site, degree of pigmentation, ulceration and surface area are all less important considerations. Melanoma is most common in the age group 40-60 and is usually treated by excision. The correct answer to question is choice 2.
Which of the following distinguishes a viral infection from other microbial assaults?
NBDE Part I released questions - Microbiology-Pathology
Which of the following distinguishes a viral infection from other microbial assaults?
1. Necrosis
2. Chronicity
3. Interferon production
4. Intracellular infection
5. Need for an external vector
Good Work
The correct choice is choice 3, interferon production. Interferons are glycoproteins produced by cells when invaded by viruses. These glycoproteins are secreted into the medium by infected cells and they induce the synthesis of antiviral proteins in uninfected cells. The uninfected cells thereby become resistant to viral invasion and the infection caused by the virus becomes attenuated. Interferons also affect the immune system. They enhance the cytotoxicity of natural killer cells which attack virus-infected cells and they augment T-cell immunity. Interferons have also been administered therapeutically for viremias and for cancers, but have shown only limited success in treating these disorders. Necrosis, choice 1, is caused by viruses in a process called lysis. When viral particles replicate within a cell, they often cause cell death. The dead cells disintegrate and the viruses are able to spread to uninfected cells. Necrosis is not specific to virus infection, however. Many bacteria, especially the pyogenic bacteria, cause necrosis of infected tissues. Viral infections can be chronic as stated in choice 2, but so can many bacterial and fungal infections. Cryptococcal meningitis, a fungal meningitis which affects AIDS patients, is an example of a chronic fungal infection. Patients with cryptococcal meningitis may have daily headaches for months before the correct diagnosis is made. Intracellular infection, choice 4, is common among bacteria and parasites. Gonorrhea, for example, is caused by a gram-negative, intracellular diplococcus. Viruses, unlike many parasites, are not usually transmitted by vectors, and choice 5 is therefore incorrect. Viruses tend to be spread through the fecal-oral route and through aerosolization and inhalation. They are also spread by sexual contact as the AIDS epidemic clearly demonstrates. Once again, the correct choice to question is choice 3.
Which of the following diseases is NOT associated with a species of Clostridium?
NBDE Part I released questions - Microbiology-Pathology
Which of the following diseases is NOT associated with a species of Clostridium?
A. Pseudomembranous colitis
B. Traveler's diarrhea
C. Antibiotic-associated colitis (AAC)
D. Botulism
E. Tetanus
Good Work
The correct answer is choice B. Traveler's diarrhea is usually associated with enterotoxigenic forms of Escherichia coli, which produce a peptide exotoxin. Pseudomembranous colitis (choice A) and antibiotic-associated colitis (AAC) (choice B) are caused by Clostridium dificile. AAC is a more modern term and refers to a Clostridium infection of the gut, set up by the killing of the normal gut flora by antibiotic use. The antibiotic most commonly associated with this condition is clindamycin. The most common treatment is vancomycin. Botulism (choice D) is caused by C. botulinum, often found in soil and animal feces. Damage occurs through a protein exotoxin. C. tetani causes tetanus (choice E). These organisms are also found in soil and animal feces. Damage is also caused by a protein neurotoxic exotoxin.
Which of the following diseases is characterized by painful vesicles that occur on the skin or a mucosal surface along the distribution of a sensory nerve?
1. Smallpox
2. Psoriasis
3. Cat-scratch fever
4. Recurrent varicella
4. Infectious mononucleosis
Good Work
The correct answer is choice 4. Recurrent varicella refers to recurrent infection by herpes virus associated with both varicella, or chicken pox, and zoster. Latent virus survive in ganglia of sensory nerves and cause mucosal outbreaks along the distribution of the nerve, often the ophthalmic division of the trigeminal nerve. Choice 1, smallpox or variola, a viral disease, will cause primarily skin lesions rather than mucosal and they are not associated with a sensory nerve distribution. Likewise, psoriasis, choice 2, usually involves skin but not mucosal membranes. The etiology of psoriasis is unknown. Choice 3, cat scratch fever, has an unknown etiology, possibly viral or bacterial, and causes fever and regional lymphadenitis, but not mucosal vesicles along a nerve distribution. Choice 5, infectious mononucleosis, is caused by Epstein-Barr virus and causes fever, enlarged lymph nodes and spleen. It is also not associated with mucosal vesicles along a nerve distribution. So the correct answer to question is choice 4.
Which of the following diseases is associated with the "Dane Particle"?
A. Hepatitis A
B. Hepatitis B
C. Hepatitis C
D. Hepatitis D
E. Hepatitis E
Good Work
The correct answer is choice B. A Dane particle is the name for the entire Hepatitis B virion (viral particle). It is one of three entities that will test positive as Hepatitis B surface antigens. The other two are pieces of viral coats known as spheres or filaments. Although they are not complete viral particles, they do contain the surface antigen (HBsAg). Other hepatitis B antigens include the core antigen (HBcAg) and an "e-antigen" (HBeAg). Presence of the "e-antigen" is associated with increased viral replication and infectivity of the patient.
Which of the following forms of oral ulcerations involves oral, ocular, and genital lesions?
1. Behçet's
2. Herpetiform
3. Recurrent aphthous
4. None of these
1. Behçet's

The correct answer is choice 1. Behcet's disease or syndrome, is a form of vasculitis or disease of the blood vessels caused by immunologic or inflammatory etiology. In particular, it affects venules and causes ulcers of the oral cavity, eyes, genitals and sometimes the skin. Choice 2, herpetiform ulcerations are caused by forms of the herpes virus and are usually found either in the oral cavity especially when caused by herpes simplex 1 or in the genital area often caused by herpes simplex 2. There may be a conjunctivitis or inflammation of the outer covering of the eye but generally no ocular ulcerations. So choice 2 is close but incorrect. Choice 3 is incorrect. Recurrent aphthous lesions have an unknown etiology and often occur in the oral cavity. They are painful, spherical ulcers with a possible autoimmune or psychosomatic connection. They generally do not occur on either the genitals or in the eye. So the correct answer to question 66 is choice 1.
Which of the following fungal species is NOT a dermatophyte?
A. Trichophyton
B. Epidermophyton
C. Coccidioides
D. Microsporum
C. Coccidioides

The correct answer is choice C. Coccidioides is a free-living soil organism that can occasionally cause disease, especially in the immunocompromised. Other soil fungi include Aspergillus, Blastomyces, and Histoplasma. They usually cause systemic infections. The dermatophytes generally grow on skin surfaces, causing athlete's foot, ringworm, jock itch, etc. Ringworm is due to infection by Trichophyton (choice A) or Microsporum (choice D), whereas athlete's foot is usually caused by Trichophyton or Epidermophyton (choice E). The infection by these organisms is usually limited to superficial keratinized tissues.
Which of the following genera is MOST likely involved in bacillary dysentery?
A. Vibrio
B. Shigella
C. Entamoeba
D. Salmonella
E. Escherichieae
B. Shigella

The correct answer is choice B. Shigella is the most likely involved in bacillary dysentery characterized by abdominal cramps and diarrhea. The feces contain blood, polymorphonuclear leukocytes, and mucus. These symptoms are important in differentiating dysentery from diarrhea which refers to watery feces, most commonly associated with increased secretion of fluid across the mucosal surfaces of the small intestine in response to a toxin or viral infection. Choice A - Vibrio typically presents with watery ("rice water") diarrhea (20 liters/day) with the loss of sodium, chloride, potassium, and bicarbonate. Other important clinical manifestations include nausea, vomiting, abdominal cramps, metabolic acidosis and hypovolemic shock. Choice C - Entamoeba is a protozoa and thus cannot specifically cause bacillary dystenery. Clinically, the disease may be mild with diarrhea, abdominal cramps nausea, vomiting, and flatulence. Note because this also causes diarrhea, it is a good trick distractor choice. Remember though the questions asks which is the most likely involved in bacillary dystenery and clearly Shigella is the most likely. Choice D - Salmonella is also a good trick distractor choice. Salmonella infections cause inflammatory diarrhea with fever and variable septicemia. In contrast to Shigella, a large inoculum (> 1 million cells) is needed to survive gastric acid and cause disease. Thus it is less likely a cause than Shigella. Choice E - Escherichieae is the most common cause of "traveler's diarrhea". There are more than 100 serotypes of Escherichieae that cause this non-inflammatry, secretory diarrhea that is similar to Vibrio, but less severe. Therefore the correct answer to question is B.
Which of the following genera most frequently develops resistance to penicillin?
1. Neisseria
2. Treponema
3. Actinomyces
4. Streptococcus
5. Staphylococcus
5. Staphylococcus

The correct answer is choice 5. Choice 1, Neisseria, include Neisseria meningitis and Neisseria gonorrhea. Although both strains are susceptible to penicillin, some strains of N. gonorrhea, which causes gonorrhea are becoming resistant to penicillin. Choice 2. treponema, are spirochete, including Treponema pallidum which causes syphilis. Penicillin is still the drug of choice for treatment. Choice 3, Actinomyces includes Actinomyces israeli which causes actinomycosis. Penicillin is still the drug of choice for treatment. Choice 4, Streptococcus includes a wide variety of organisms including those involved in tooth decay, endocardids and pneumonia. Most Streptococci are still sensitive to penicillin. Choice 5, Staphylococci, include a large variety of organisms and can cause diseases ranging from skin infections to osteomyelitis, pneumonia and toxic shock syndrome. Many strains are resistant to penicillin and penicillinase resistant penicillins such as dicloxacillin are the drugs of choice. So the correct answer to question is choice 5.
Which of the following histopathologic findings during necropsy suggests that the person died of, or at least had, rheumatic fever?
1. Russell bodies
2. Periarteritis nodosa
3. Aschoff bodies in the heart muscle
4. Monckeberg's sclerosis of the aorta
3. Aschoff bodies in the heart muscle

The correct choice is choice 3, Aschoff bodies in the heart muscle. Rheumatic fever occurs when an individual produces antibodies to group A streptococci following Strep. pharyngitis. These antibodies cross-react with antigens present in a variety of tissues, including the myocardium. In the heart muscle, this immunologic damages causes a sequelae of changes. First, an exudate forms which causes separation of muscle cells. This is followed by the formation of Aschoff bodies which are collections of histiocytes surrounding a central focus of fibrinoid material. Finally, fibroblasts proliferate and a scar is formed. Russell bodies, choice 1, are eosinophilic inclusions seen in plasma cells. They are dense collections of immunoglobulins located in the endoplasmic reticulum. Periarteritis nodosa, choice 2, is not merely a finding, but a disease process. This disorder is characterized by necrosis and thrombosis of small and medium sized arteries throughout the body. Monckeberg's sclerosis, choice 4, is a disease process in which the media of large arteries becomes calcified circumferentially. This differs from atherosclerosis in which calcium is deposited in the intima. The aorta may be involved in Monckeberg's sclerosis as indicated in choice 4, but the disease more commonly affects the large arteries of the extremities. These patients are asymptomatic unless the vessels have concomitant atherosclerotic disease. Once again, the correct choice to question is choice 3.
Which of the following is a benign lesion, most commonly found on the buccal mucosa and the dorsal surface of the tongue, and has a surface, appearance of numerous small closely packed nodular structures filled with fluid and may resemble a blister?
A. Hemangioma
B. Lymphangioma
C. Melanoma
D. Nevi
E. Pleomorphic adenoma
B. Lymphangioma

The correct answer is B. A lymphangioma (choice B) is a benign lesion that is most commonly found on the buccal mucosa and the dorsal surface of the tongue. The surface appearance is one of numerous small closely packed nodular structures filled with fluid and may resemble a blister. Oral hemangiomas (choice A) are benign lesions that occur most frequently on the buccal and labial mucosa, vermilion of the lip and the tongue. They typically present as a red, smooth, protruding lesion at the tip of the tongue. Malignant melanomas can occur in any area of the oral cavity; however, they are most frequently found on the buccal gingiva and palatal tissues. Initially the lesions are slightly raised, purplish, dark brown patches several millimeters in diameter. Due to their constant rapid growth pattern, the tumor mass tends to enlarge in all directions. A prominent, ill-defined, firmly anchored lesion eventually results. The characteristic feature of this metastatic lesion is its purplish-black color. This central lesion is surrounded by an area of erythema. Nevi (choice D) are also benign lesions that may occur in any area of the oral cavity; they typically present as discrete, localized, brown-black pigmented lesions. Pleomorphic adenomas (choice E) are commonly found in the salivary glands, palate, lip and buccal glands; this type of tumor causes a firm, slow growing mass. Surgical excision of these lesions is often indicated; however, aggressive chemotherapy and/or radiation therapy is rarely needed.
Which of the following is a characteristic of benign, as opposed to malignant, neoplasms?
A. Encapsulated lesion
B. Many mitoses
C. Rapid growth
D. Poorly differentiated
E. Frequent metastases
A. Encapsulated lesion

The correct answer is choice A. Let's review the characteristics of neoplasm growth. In each case, the benign neoplasm characteristic will be mentioned first, and the corresponding malignant characteristic will be in parentheses. Slow growth (fast or erratic growth) (choice C), encapsulated (no encapsulation, invasive), differentiated, like tissue of origin (undifferentiated, nonspecialized) (choice D), few mitoses (many mitoses) (choice B), normal nuclear/cytoplasm ratio (high nuclear/cytoplasm ratio), expands, compresses tissue (invades tissue), and nonmetastatic (often metastatic) (choice E).
Which of the following is a congenital condition that alters the texture of the oral and/or vaginal mucosa causing painless white lesions?
A. Black hairy tongue
B. Fordyce's disease
C. Lichen planus
D. Nicotinic stomatitis
E. White spongy nevus
E. White spongy nevus

The correct answer is E. White sponge nevus (choice E) is a congenital condition that alters the texture of the oral, vaginal, and/or anal mucosa. It causes painless lesions that are typically "pearly" white in appearance. Hairy tongue (choice A) is a disorder that results in the elongation of filiform papillae of the tongue leading to the characteristic brown-black color of the dorsum of the tongue. Fordyce's disease (choice B) causes either buccal and/or labial mucosal lesions. This condition causes an aggregation of numerous small yellowish spots beneath the mucosal surface. Lichen planus (choice C) affects both the skin and oral mucus membranes. In addition to the characteristic violaceous lesions which are usually etched with fine grayish-white lines (Wickham's striae), one may see white scaly areas over a light pink base similar to psoriasis. Furthermore, lichen planus usually causes lesions to appear on the buccal mucosa, tongue, gingiva and/or lips. Nicotinic stomatitis (choice D) is typically found on the hard palate of pipe smokers and is characterized by several white elevations with a central red area.
Which of the following is a demyelinating disease of the central nervous system?
1. Multiple sclerosis
2. Alzheimer's disease
3. Parkinson's disease
4. Creutzfeldt-Jakob disease
1. Multiple sclerosis

The correct answer is choice 1. Let's briefly review the diseases or conditions listed. Choice 1, multiple sclerosis, or MS is a demyelinating disease of the central nervous system. It is characterized by a changing disease course of exacerbations and remissions with hyperreflexia. weakness and spasticity. In addition, dysfunction of the cerebellum may occur. The etiology is possibly autoimmune with influence by a viral infection. Alzheimer's disease, choice 2, is incorrect. Alzheimer's is a form of dementia with unknown etiology. Atrophy of the cerebral cortex occurs with loss of neurons and tangled areas known as neurofibrillary tangles. Demyelination is not involved. Parkinson's disease, choice 3, involves degeneration of neurons in the nucleus basalis, and dopamine depletion. It is often treated with L-dopa, a dopamine precursor. However, demyelination is not involved. Choice 4, Creutzfeldt - Jakob's disease, is a disease of the cortex and basal ganglia caused hy slow acting virus-like agents. Personality changes, incoordination, dementia and death eventually occur. However, demyelination is not involved. So the correct answer to question is choice 1.
Which of the following is a DNA virus?
A. Hepatitis A
B. Hepatitis B
C. Hepatitis C
D. Hepatitis D
E. None of the above
B. Hepatitis B

The correct answer is choice B. Hepatitis B is the only hepatitis virus that is a double stranded DNA virus. It is a member of the Hepadnavirus family, and is associated with sexual and bloodborne transmission. It is known as long incubation "serum hepatitis". Hepatitis A is an RNA enterovirus. Thus it is a single strand virus. You should also know it as a short incubation hepatitis transmitted by the fecal/oral route. Hepatitis C has been identified as a single strand RNA virus. It used to be more commonly transmitted by transfusions, but a test for the virus now has reduced that type of spread. Hepatitis D is caused by the Delta particle, a single stranded RNA virus. Only persons infected with Hepatitis B can be infected with Hepatitis D. Hepatitis E is another RNA enterovirus, although less is known about it than Hepatitis A.
Which of the following is a fungus that is known to produce small ulcerative lesion in the mouth, primarily on the tongue and gingiva?
A. Bacteroides fragilis
B. Candida albicans
C. Clostridium perfringens
D. Histoplasma capsulatum
E. Streptococcus mutans
D. Histoplasma capsulatum

The correct answer is D. Histoplasma capsulatum (choice D) is a fungus that is associated with the development of small ulcerative lesions in the mouth, particularly on the tongue, gingiva or palate. Furthermore, hoarseness and dyspahgia may occur, secondary to the lesions in the larynx. Fever and malaise are also commonly seen. Bacteroides fragilis (choice A) is an anaerobic organism commonly associated with the development of adult peridontitis and acute necrotizing ulcerative gingivitis. Acute necrotizing ulcerative gingivitis is associated with inflammation of the gingiva with necrosis, tissue loss, pain, bleeding and halitosis. Candida (choice B), a fungus, is the most common cause of "oral thrush", which is typically associated with the appearance of white patches in the mouth that can be easily wiped off to reveal a red, bleeding, sore surface. Clostridium perfringens (choice C) is the bacteria most commonly associated with the development of clostridial myonecrosis (gas-gangrene) and food poisoning. Streptococcus mutans (choice E) is a gram positive aerobic bacteria that is commonly associated with the development of dental caries, a destructive disorder of the hard tissues of the teeth.
Which of the following is a Group A betahemolytic Streptococcus?
A. S. mitis
B. S. pyogenes
C. S. sanguis
D. S. salivarius
E. S.mutans
B. S. pyogenes

The correct answer is choice B. Srreptococcuis pyogenes is a pathogenic Strep of the Group A beta hemolytic group. It is involved in a wide range of diseases, including pharyngitis, scarlet fever, rheumatic fever and glomerulonephritis. All of the other Streptococci listed are Viridans Streptococci, which are non-hemolytic. They are normal oral flora, and are usually non-pathogenic. They can cause subacute endocarditis in susceptible individuals, however. This is the basis of antibiotic prophylaxis for dental procedures. In otherwise healthy individuals, the only other damage caused by these organisms is dental caries (especially S. mutans).
Which of the following is a significant effect of pheochromocytoma?
1. Myxedema
2. Acromegaly
3. Glycosuria
4. Hypertension
4. Hypertension

The correct choice is choice 4, hypertension. Pheochromocytoma is a tumor of chromaffin cells. Most of these tumors are located in the abdomen and 80% are found in the adrenal medulla. Another common site is a the aortic bifurcation. These neoplasms secrete catecholamines, especially norepinephrine, which causes paroxysmal hypertension. The hypertension is relieved when the tumor is resected. Myxedema, choice 1, is a term for the clinical syndrome resulting from hypothyroidism. This syndrome is characterized by generalized lethargy and slowing of mental and motor functions, coarse skin, hoarse voice, peripheral and periorbital edema, and in extreme cases, coma. Acromegaly, choice 2, results from hypersecretion of growth hormone. It is characterized by enlargement of the jaw, hands and feet, and coarsening of the facial features. Glycosuria, choice 3, literally means sugar in the urine. It results whenever glucose filtration exceeds glucose resorption. It commonly occurs in diabetes mellitus, but may also be found in renal tubular disease. So the correct choice to question is choice 4.
Which of the following is an example of naturally-acquired passive immunity in humans?
1. Tetanus immunization
2. Injection of gamma globulin
3. Transfusion of immune blood
4. Placental transfer of antibody
5. Antibody produced during recovery from measles
4. Placental transfer of antibody

The correct choice is choice 4, placental transfer of antibody. Passive immunity occurs when a recipient receives antigen-specific immunoglobulins from a donor. Placental transfer of antibody is an example of passive immunity which occurs naturally and is fundamental to life. Infants prior to six months of age are incapable of generating adequate antibody to fight infections. During this period of immune maturation, they are dependent on maternal antibodies which are acquired prenatally across the placenta and postnatally in breast milk. The only immunoglobulins which can cross the placenta are those of the IgG class, but this is adequate to maintain health. Tetanus immunization, choice 1, is an example of active immunity. The individual is injected with tetanus toxoid, a molecule which is antigenically similar to tetanus toxin, but lacks its biologic effects. An immune response is induced and the individual synthesizes immunoglobulins specific for tetanus toxoid and toxin. Injection of gammaglobulin, choice 2, is an example of m passive immunity, but it does not occur naturally. Gammaglobulin is administered by a healthcare practitioner to an individual who has recently been exposed to an infectious agent such as hepatitis B. The gammaglobulin neutralizes the virus before infection can occur. Transfusion of immune blood, choice 3, is similar to choice 2. It is passive immunity, but is not a natural process. Immune blood is rich in various antibodies and it is given to individuals who cannot synthesize immunoglobulins. Antibody produced during recovery from measles, choice 5, is a classic example of active immunity. The virus induces an immune response in the host. The process is a natural one, but no antibody donor is involved as in placental transfer of IgG. Once again, the correct choice to question is choice 4.
Which of the following is an RNA virus?
A. Herpes simplex
B. Varicella zoster virus
C. Hepatitis A
D. Cytomegalovirus
E. Epstein-Barr virus
C. Hepatitis A

The correct answer is choice C. This is not just a memory question but a grouping question. The herpes viruses are all DNA viruses and include Herpes simplex I and II (choice A), cytomegalovirus (CMV) (choice D), Epstein-Barr virus (EBV) (choice E), and varicella zoster virus (VZV) (choice B). Hepatitis A is an RNA virus, as is Hepatitis C and D. Only Hepatitis B is a DNA virus among the hepatitis viruses.
Which of the following is characteristic of a prokaryotic cell?
1. Mitochondrion
2. Nuclear membrane
3. Single chromosome
4. Endoplasmic reticulum
3. Single chromosome

The correct choice is choice 3, single chromosome. Two fundamental cell types exist. Prokaryotic cells are very simple cells. The bacteria and the blue algae are essentially modified prokaryotic cells. These cells lack specialized organelles such as a nucleus with its nuclear membrane (choice 2), mitochondria (choice 1), endoplasmic reticulum (choice 4), Golgi bodies, peroxisomes and lysosomes. They lack a cytoskeleton and they are incapable of the cytoskeleton-dependent processes of endocytosis and exocytosis. These cells are quite small, measuring only one to ten micrometers in length and they rarely aggregate to form multicellular units. Many prokaryotes lack enzymes of the Krebs' cycle and are therefore dependent on glycolysis for their ATP production. DNA is found in the cytoplasm and it exists, as stated in choice 3, as a single chromosome. This chromosome is organized in a large circular strand rn which does not condense when the organisms divide by binary fission. Eukaryotic cells, by contrast, localize their DNA in the nucleus. There. the genetic material is organized into discreet bundles which undergo condensation when the cell divides by mitosis. Eukaryotic cells have a host of organelles lacking in the prokaryote. The eukaryote utilizes the Krebs' cycle and can thus generate more ATP per molecule of glucose metabolized. These cells can be quite large. Some measure 100 micrometers in length. They a possess a complex cytoskeleton and are capable of endo- and exocytosis, and most eukaryotic cells are organized in multicellular systems wherein each cell is specialized for a different function. All fungi, plants and animals are composed of eukaryotic cells. Once again, the correct choice to question is choice 3.
Which of the following is characterized by a collapse of alveoli?
1. Empyema
2. Pneumonia
3. Emphysema
4. Atelectasis
5. Bronchiectasis
4. Atelectasis

Atelectasis refers to collapse of alveoli and is often found in cases of asthma of the allergic type. Choice 1 is incorrect. In empyema, the pleura are filled with purulent exudate; no alveolar collapse occurs. Choice 2 is incorrect. In bacterial pneumonias, alveoli are filled with purulent exudate. In viral or mycoplasmal pneumonia, alveoli are often covered by hyaline membrane. Choice 3 is incorrect. In emphysema, alveolar septa are destroyed, sometimes leading to confluent distended air spaces. Choice 5 is incorrect. In bronchiectasis, bronchi and bronchioles dilate abnormally. So the correct answer to question is choice 4.
Which of the following is characterized by a cumulative antimicrobial effect?
1. Iodine
2. Alcohol
3. Cationic detergent
4. Acid glutaraldehyde
5. Chlorhexidine gluconate
5. Chlorhexidine gluconate

The correct choice is answer #5, chlorhexidine gluconate. Chlorhexidine is an antiseptic which kills gram-positive bacteria and other organisms by disrupting their plasma membranes. When chlorhexidine is used repeatedly on the skin, the chemical accumulates to produce an enhanced antimicrobial effect. By contrast, the agents listed in choices 1 through 4 do not show this property. Iodine, choice 1, kills microorganisms by an unknown mechanism. It is effective against live bacteria and spores, and it is the most widely used antiseptic in hospitals. Alcohol, choice 2, works as an antiseptic by precipitating protein. It is not effective against spores, however. Cationic detergents, choice 3, include silver and mercury compounds. Silver ions are effective agents for precipitating proteins and interfering with bacterial metabolism. Mercury ions also precipitate proteins, but they have an added effect of self-hydro-enzyme inhibition. Acid glutaraldehyde, choice 4, also kills microorganisms by precipitating proteins. It is effective against spores only when applied for several hours. Once again, the correct choice to question is choice 5.
Which of the following is CORRECT for obligate anaerobic microorganisms in the oral cavity?
A. They do not exist in this area.
B. Only gram-positive organisms are found.
C. They are normal flora and opportunistic.
D. They are seldom isolated in the laboratory.
E. They can be completely controlled by using antibiotics.
C. They are normal flora and opportunistic.

The correct answer is choice C. Many gingival sulcus and periodontal pathogens are anaerobic. They are part of the normal flora and become pathogenic under certain host conditions, including lowered resistance and inadequate plaque and calculus control. They can be either gram positive or gram negative. For example, Actinobacillus is gram negative, while Eubacterium is gram positive. Some of these anaerobes can be isolated in the laboratory, although spirochetes are difficult or impossible. Bacteria can be controlled to some extent through antibiotic use (penicillin for odontogenic infections, streptomycin for some periodontal infections, etc.). However, complete control is clearly impossible. The mouth is full of microorganisms, and the mouth cannot be made sterile through antibiotic use. So the correct answer to question is choice C.
Which of the following is diagnosed by karyotyping?
1. Phenylketonuria
2. Neurofibromatosis
3. Tay-Sachs disease
4. Turner's syndrome
5. Sickle cell anemia
4. Turner's syndrome

The correct answer is choice 4.
Karyotyping involves the counting and typing of chromosomes; in particular, disorders in number of chromosomes such as Down's syndrome, or number and type of sex chromosomes such as Turner's or Klinefelter's syndrome, can be detected by karyotyping. Turner's syndrome refers to a condition of 45 total chromosomes with one X and no Y chromosomes, also known as XO. Choice 1, phenylketonuria, is a disorder of phenylalanine metabolism. This is detected by a bacterial inhibition assay. Choice 2. neurofibromatosis, is a tumor of Schwann cells. It is hereditary and is associated with pigmented cafe au lait skin lesions, neural tumors and pigmented tumors of the iris. Choice 3, Tay-Sachs, is a disorder of sphingolipids and is detected by enzyme assay. Choice 5, sickle cell anemia, is a hereditary protein formation disorder leading to defective hemoglobin and red blood cells. It is detected through microscopic exam of blood smears. So the only choice which is detected by karyotyping is choice 4, Turner's syndrome, and the correct answer to question is choice 4.
Which of the following is elevated in the serum of patients with prostate cancer?
1. Acid phosphatase
2. Alpha-fetoprotein
3. Alkaline phosphatase
4. Carcinoembryonic antigen
1. Acid phosphatase

The correct choice is choice 1, acid phosphatase. Various serum enzymes have been helpful in following patients with cancer. In general, these enzymes cannot be utilized to screen patients for the presence of cancer because elevations of serum levels of these enzymes can be due to a variety of causes. However, once a primary tumor has been discovered, measurement of these enzymes correlates with the progression of the disease. Consider the following example: A 67-year-old black male has a hard nodule measuring 1 cm on his prostate discovered on routine exam. His serum acid phosphatase at that time was twice normal. Following prostatectomy, the acid phosphatase level was normal. He was followed with acid phosphatase measurements every six months. Two years post-op, his acid phosphatase was six times normal. The patient also complained of back pain. A bone scan revealed diffuse bony metastases. This scenario is quite typical of patients with prostatic cancer. Alpha-fetoprotein, choice 2, is generally elevated in patients with hepatocellular carcinoma or yolk sac tumors. Alkaline phosphatase, choice 3, is the least specific of the enzymes given in this question. It is an enzyme found primarily in liver and bone and is often elevated in both benign and malignant diseases involving these tissues. Of note, patients with prostatic cancer may have elevated alkaline phosphatase levels, but this results from bone or liver metastases. It is not secreted by the tumor directly. Carcinoembryonic antigen, choice 4, is elevated primarily in patients with adenocarcinomas of the colon or lung and in various benign conditions. Once again, the correct choice to question is choice 1.
Which of the following is MOST closely related to Mycobacterium tuberculosis?
A. M. bovis
B. M. kansasii
C. M. avium
D. M. intracellulare
A. M. bovis

The correct answer is choice A. All of the Mycobacteria listed are related to M. tuberculosis; however. M. bovis is most closely related. It causes bovine tuberculosis and can infect people who drink unpasteurized milk. The other species listed are known as atypical mycobacteria. They are soil-based organisms, but they take on special significance in immunocomprnmised patients who have difficulty defending against these organisms. Atypical mycobacterial infections are common in AIDS patients and are AIDS-defining.
Which of the following is most commonly associated with development of gastrointestinal cancer?
1. Diverticulosis
2. Villous adenoma
3. Pedunculated adenoma
4. Meckel's diverticulum
5. Duodenal peptic ulcer
2. Villous adenoma

The correct choice is choice 2, villus adenoma. A villus adenoma most commonly occurs in the rectum or sigmoid colon. Villus adenomas are usually not pedunculated, but have a broad short base. Histologically, these lesions show cellular atypia with mitoses. Carcinoma in situ or invasive carcinoma is found in roughly 40% of all lesions. Therefore, when the diagnosis of villus adenoma is made, colonic resection must be performed. Diverticulosis, choice 1, is a common disorder of the colon characterized by multiple herniations of the colonic mucosa. These herniations arise at points of muscular weakness under conditions of increased intraluminal pressure. Diverticula often bleed or become infected, but they do not undergo malignant degeneration. Pedunculated adenoma, choice 3, is also called tubular adenoma. This lesion has a slender stalk and a broad head. Histologically, the cells show atypia, but mitotic figures are rare. When tubular adenomas are small, the risk that they will contain a malignant focus is less than 1%. As the lesions enlarge, however, the risk of cancer increases. Meckel's diverticulum, choice 4, is a remnant of the omphalomesenteric duct. It can usually be found in proximity to the ileocecal valve. These diverticula may bleed or perforate, but they are not associated with cancer. Duodenal peptic ulcer, choice 5, arises in individuals who have increased gastric acid secretion. Duodenal ulcers may bleed, perforate or obstruct the gastrointestinal tract, but they never undergo malignant change. Gastric ulcers, by contrast, are malignant in about 1% of cases. Once again, the correct choice to question is choice 2.
Which of the following is MOST likely to cause a sudden arrest of heart function?
A. Mitral stenosis
B. Angina pectoris
C. Constrictive pericarditis
D. Cardiac tamponade
E. Subacute bacterial endocarditis
D. Cardiac tamponade

The correct answer is choice D. Cardiac tamponade refers to a condition in which fluid fills the pericardial space, produces pressure on the ventricles and prevents filling of the ventricles during diastole. This is an acute life threatening condition. Mitral stenosis involves limited motion of the mitral valve and causes chronic heart dysfunction. Angina pectoris is heart pain caused by poor blood flow in the coronary arteries. It will not usually lead to sudden cardiac arrest. Constrictive pericarditis involves the growth of fibrous tissue in the pericardium, causing decreased ventricular filling, but the condition is not acute. Subacute bacterial endocarditis involves infection of heart valves by bacteria. usually streptococci. Results may include valve damage, myocardial abscess, and septic emboli. However, it does not involve sudden cardiac arrest. So the correct answer to question is choice D.
Which of the following is NOT a common soil fungus?
A. Candida
B. Histoplasma
C. Coccidioides
D. Blastomyces
A. Candida

The correct answer is choice A. As stated before, free-living soil fungi may be taken into the body, especially the lungs, and cause systemic disease. Histoplasma (choice B), Coccidioides (choice C), and Blastomyces (choice D) all fall into this category of fungal organism. Candida, on the other hand, is a member of the normal flora of mouth and skin and becomes an opportunistic pathogen. Oral candidiasis is common, both in the immunocompromised and in patients receiving long-term antibiotics. Oral candidiasis is common in HIV+ individuals, and pharyngeal or esophageal candidiasis is AIDS-defining.
Which of the following is NOT linked to fulminant hepatitis?
A. Hepatitis A
B. Hepatitis B
C. Hepatitis C
D. Isoniazid
E. Carbon tetrachloride
A. Hepatitis A

The correct answer is choice A. Most forms of hepatitis can lead to fulminant hepatitis (massive liver necrosis). In fulminant hepatitis, there is progressive loss of liver function and progressive shrinking of liver size due to hepatocyte death. Mortality is high. Both hepatitis B and C (choices B and C) can cause this state. Hepatitis A is always self-limiting with complete recovery and no chronic carrier stage. Less well known is the ability of chemicals to cause fulminant hepatitis. INH (isoniazid) (choice D), used in treating tuberculosis, can have this side effect. A group of related organic solvents (carbon tetrachloride [choice E], chloroform) are also dangerous in this way, and occupational exposure must be strictly monitored.
Which of the following is NOT true concerning benign prostatic hypertrophy (BPH)?
A. It commonly leads to carcinoma of the prostate
B. Incidence increases with age
C. Tissue may be glandular or fibrous
D. It often causes urinary obstruction
E. It is not considered a neoplasm
A. It commonly leads to carcinoma of the prostate

The correct answer is choice A. BPH (benign prostatic hypertrophy) is a non-neoplastic condition (choice E) in which the prostate gland enlarges and compresses surrounding tissue. This hypertrophy is composed of normal prostate cells, either glandular or fibromuscular (choice C). Signs and symptoms often include difficulty in urination (choice D), and incidence increases with age (choice B), beginning at around age 45. Ninety-five percent of men greater than 70 years of age have some form of this condition. There may be a hormonal etiology, although it is not yet clear. It is not believed to predispose to cancer of the prostate.
Which of the following is NOT true concerning Paget disease?
A. It is also known as osteitis deformans
B. It may be discovered due to change in hat size
C. Affected areas may be warm to the touch
D. It is associated with an extremely low alkaline phosphatase
E. It often results in deafness
D. It is associated with an extremely low alkaline phosphatase

The correct answer is choice D. Paget disease is also known as osteitis deformans (choice A) or Paget disease of bone. It is usually polyostotic (affecting many bones), but effects on the skull may be most noticeable including increasing skull size (choice B) and loss of hearing (choice E) due to impingement of CN VIII. There is increased vascularity of bone and accompanying warmth of the skin over these areas (choice C). Bone processes may be osteolytic or osteoblastic, as bone is dissolved, replaced by fibrous tissue, remineralized, etc. Radiographic appearance may vary due to these changes. The consistent lab finding is extremely elevated alkaline phosphatase.
Which of the following is the most common initial sign or symptom in patients with malignant lymphoma?
1. Pallor
2. Weight loss
3. Lymphadenopathy
4. Chronic infection
5. Unexplained fever
3. Lymphadenopathy

The correct answer is choice 3.
Lymphomas are lymphatic neoplasms characterized by lymphadenopathy and hepatosplenomegaly. Lymphomas are detected first in the lymph nodes, often the cervical chain. The other choices 2 and 5, weight loss and unexpected fever, are possible symptoms associated with lymphomas, but are not usually noticed first as they follow the lymphadenopathy. Choices 1 and 4, pallor and chronic infection, are not common symptoms of patients with lymphomas. So the correct answer to question is choice 3.
Which of the following is the most common type of lung cancer?
1. Adenocarcinoma
2. Bronchiolar carcinoma
3. Alveolar cell carcinoma
4. Squamous cell carcinoma
5. Anaplastic (oat cell) carcinoma
4. Squamous cell carcinoma

The correct choice is choice 4, squamous cell carcinoma. Lung cancer is the most common cancer is men and may have surpassed breast cancer as the most common cancer in women. It is a devastating disease with a 5-year mortality rate of 90%. Roughly one-half of all lung cancers are squamous cell carcinomas. The next most common cancer of the lung is adenocarcinoma, choice 1, followed by oat cell carcinoma, choice 5. Large cell carcinoma, which is not listed in this question, is the fourth most cancer. Bronchiolar and alveolar cell carcinomas, choices 2 and 3 respectively, are subclassifications of adenocarcinomas. Alveolar cell carcinoma is rare. It represents approximate 3% of all malignant lung tumors. Once again, the correct choice to question is choice 4.
Which of the following is the single MOST numerous group of microorganisms in the oral cavity?
A. Enterococci
B. Staphylococci
C. Anaerobic streptococci
D. Facultative streptococci
E. Beta-hemolytic streptococci
D. Facultative streptococci

The correct answer is choice D. This is an especially common NDB point. By far, most bacteria in the mouth are facultative streptococci. These organisms will survive in either aerobic or anaerobic conditions. Chief among these organisms are the non-betahemolytic Streptococci, such as S. mutans, S.salivarius and S. mitor. So the correct answer to question is choice D.
Which of the following is true concerning carcinoma of the cervix?
A. It is usually an adenocarcinoma
B. Carcinoma in situ is rarely curable
C. It is associated with specific viruses
D. It is most common in the sixth to seventh decades
E. It is the most common malignant tumor in women
C. It is associated with specific viruses

The correct answer is choice C. Cervical cancer is the sixth most common cause of cancer death in women (choice E). It is most common in the fourth to fifth decades (choice D), and it is believed that certain factors related to the disease can be passed from male to female, in particular, infection with either HSV II or HPV (human papilloma virus). Cervical cancer is typically a squamous cell carcinoma, not an adenocarcinoma (choice A). When confined to the outer epithelial tissue, without invasion of the connective tissue (carcinoma in situ), it is highly curable (choice B). Late disease has a much lower cure rate. Detection is, on average, much earlier in the last decade due to widespread use of the Papanicolaou test (Pap smear) as a yearly screening tool.
Which of the following is typically elevated in the serum of patients with prostate cancer?
1. Acid phosphatase
2. Alpha-fetoprotein
3. Alkaline phosphatase
4. Carcinoembryonic antigen
1. Acid phosphatase

The correct answer is choice 1. Prostatic acid phosphatase is known as a chemical tumor marker. When elevated, it indicates a prostate tumor which has extended outside the capsule of the prostate. Choice 2 is incorrect. Alphafetoprotein is another chemical tumor marker. Its presence indicates the presence of a hepatoma, an embryonal cell tumor of the testis, or a malignant teratoma. Choice 4 is incorrect. Carcinoembryonic antigen is another tumor marker which indicates the presence of any tumor derived from gut epithelium. It may also be elevated in smokers without tumors. Choice 3 is incorrect. Elevated alkaline phosphatase can be found in patients with Paget's disease of bone. So the correct answer to question is choice 1.
Which of the following laboratory results is diagnostic of acute pancreatitis?
1. Decreased albumin
2. Elevated serum lipase
3. Decreased serum amylase
4. Elevated alkaline phosphatase
2. Elevated serum lipase

The correct answer is choice 2. Acute pancreatitis involves diffuse necrosis of the pancreas caused by release of activated pancreatic enzymes. Laboratory results reveal elevated amylase and lipase enzymes in the blood, and sometimes a hypocalcemia. Labs for chronic pancreatitis also reveal elevated amylase and alkaline phosphatase, but not lipase. So elevated lipase is more diagnostic of acute pancreatitis. So choice 2 is correct. Choice 3 is incorrect also because it says decreased serum amylase. Choice 4, elevated alkaline phosphatase, is characteristic of chronic pancreatitis, not acute, as well as other conditions, including Paget's disease of bone. So the correct answer to question is choice 2.
Which of the following medications is effective in treating oral candidiasis?
A. Griseofulvin
B. Protease inhibitor
C. Tetracycline
D. Metronidazole
E. Nystatin
E. Nystatin

The correct answer is choice E. Oral candidiasis can be treated topically or systemically. The most common topical treatment is nystatin (rinse or troches), amphotericin B or clotrimazole. Systemic treatments include fluconazole and ketoconazole. Typically, early cases are treated with topical medications first. Griseofulvin is a systemic antifungal used for treating tinia, ringworm and athlete's foot. Tetracycline is an anti-bacterial used often in treating gingival infections. It should not be given to young children or pregnant women. It is bacteriostatic. Metronidazole (flagyl) is an agent effective against both anaerobic bacteria and many protozoans.
Which of the following microorganisms is most likely to be cultured from chronic, bilateral ulcerations at the corners of the mouth?
1. Candida
2. Brucella
3. Treponema
4. Aspergillus
5. Histoplasma
6. Trichophyton
1. Candida

The correct answer is choice 1. Candida albicans is a fungal organism, part of the normal oral flora. It can cause problems under certain conditions including chronic ulcerations at the comers of the mouth. In edentulous patients, the loss of vertical dimension may cause drooping at the comers of the mouth and Candida infection may cause ulcerations at these points. Candida is also found infecting the tongue and buccal mucosa in immunocompromised patients such as AIDS patients. Choice 2 is incorrect. Brucella is a gram negative bacillus which can cause infections in man but not specifically at the corners of the mouth. Choice 3 is incorrect. Treponema are spirochetes, the most familiar of which is Treponema pallidum which causes syphilis. Choice 4 is incorrect. Aspergillus is a fungus which may invade the hand or foot of a farmer or rural worker usually from the soil. Choice 5 is incorrect. Histoplasma is a fungus which causes histoplasmosis, a fungal infection of in the lungs. It can be spread by bird excrement. Choice 6 is incorrect. Trichophyton is a fungus often involved in fingemail and toenail infections known as tinea unguium. So the correct answer to question is choice 1.
Which of the following most accurately describes the antibacterial mechanism of penicillin?
1. Inhibits DNA synthesis
2. Inhibits the terminal step in peptidoglycan synthesis
3. Inhibits protein synthesis in growing bacterial cells
4. Disrupts the integrity of the cell membrane
2. Inhibits the terminal step in peptidoglycan synthesis

The correct choice is choice 2, inhibits the terminal step in peptidoglycan synthesis. Both gram-positive and gram-negative bacteria have cell walls composed of peptidoglycans which encase their plasma membrane. Peptidoglycans are composed of polymers 4 of N-acetylglucosamine, N-acetylneuraminic acid and amino acids. These polymers are cross-linked to form a stable barrier in the terminal step of peptidoglycan synthesis. Penicillin interferes with this cross-linking step and thereby prevents the formation of a durable cell coat. Affected organisms become highly susceptible to rupture and fragmentation as a result. Other antibiotics which interfere with cell well synthesis include bacitracin, the cephalosporins and vancomycin. Some antibiotics exert their effect by disrupting cell membranes of pathogenic organisms as mentioned in choice 4. Antibiotics in this class include the antifungal agents, amphotericin-B and nystatin. These agents form channels in the fungal wall and disrupt ion gradients required for normal metabolism. Other antibiotics act as mentioned in choice 3, through protein synthesis inhibition. Antibiotics in this class include the aminoglycosides, chloramphenicol. erythromycin and tetracycline. Finally, drugs such as rifampin, sulfonamides and trimethoprim are effective antimicrobial agents through the inhibition of bacterial DNA synthesis which is mentioned in choice 1. Once again, the correct choice for question is choice 2.
Which of the following neoplasms appears the MOST often in children?
B. Chondrosarcoma
C. Adenocarcinoma
D. Multiple myeloma
E. Basal cell carcinoma
A. Neuroblastoma

The correct answer is choice A. Neuroblastoma is the most common extracranial solid tumor in children. It is often found in the adrenal medulla, causes elevated catecholamines and metastasizes readily. Chondrosarcoma is a malignant tumor of chondroblasts, found mostly in adults 30-60, male more often than female. The most common adenocarcinoma is found in glandular elements of the respiratory tract, especially in male adult smokers between 40-70 years old. Multiple myeloma is a neoplasm of plasma cells resulting in excessive abnormal immunoglobulins, including Bence-Jones proteins. It is most common in adults 50-60 years old. Basal cell carcinomas are malignant skin neoplasms most common in middle aged and older adults. So the correct answer to question is choice A.
Which of the following organisms causes pneumonia exclusively in immunocompromised individuals?
A. Diplococcus
B. Pneumocystis carinii
C. Mycoplasma pneumoniae
D. Legionella pneumophila
E. Influenza virus
B. Pneumocystis carinii

The correct answer is choice B. Pneumocystis carinii pneumonia (PCP)is found exclusively in immunocompromised patients. It is common in AIDS patients, but other immunocompromised individuals may have the disease as well. It is one of many forms of pneumonia, including bacterial, chemical, and viral. Ali of the other organisms listed in the question can cause pneumonia; however, only PCP is exclusively found in individuals with damaged immune systems. Note that Legionnaires disease, caused by L. pneumophila, is more common in the elderly and immunocompromised, but it is not exclusive to them. Note also that PCP is currently classified as a fungus and had been formerly classified as a protozoan. PCP pneumonia is usually treated with sulfamethoxazole-trimethoprim.
Which of the following organisms is the most common cause of scalded-skin syndrome?
A. Escherichia coli
B. Haemophilus influenzae
C. Pseudomonas aeruginosa
D. Staphylococcus aureus
E. Streptococcus pneumoniae
D. Staphylococcus aureus

The correct answer is D. Scalded-skin syndrome (SSS) is a condition characterized by a redness and tenderness of the central face, neck, trunk, and intertriginous zones. This initial presentation is followed by the appearance of flaccid bullae and a slough or exfoliation of the superficial epidermis. The majority of the cases of SSS are caused by Staphylococcus aureus. Escherichia coli (choice A) is a gram negative organism commonly seen in urinary tract infections. Haemophilus influenzae (choice B) is gram-negative organism commonly seen in individuals with pneumonia, meningitis, respiratory infections, sinusitis and otitis infections. Pseudomonas aeruginosa (choice C) is a gram-negative organism associated with serious respiratory infections. These bacteria are most often seen in individuals receiving respiratory support. Streptococcus pneumoniae (choice E), as the name suggests, is the causative organism of pneumococcal pneumonia.
Which of the following organisms, described as being an encapsulated yeast, is the most common cause of fungal meningitis?
A. Aspergillus
B. Candida
C. Coccidioides
D. Cryptococcus
E. Pneumocystis
D. Cryptococcus

The correct answer is D. Cryptococcosus is the most common cause of fungal meningitis. This type of meningitis is caused by the organism Cryptococcus neoformans, which is classified as an encapsulated budding yeast. This infection is acquired by inhalation and is most commonly seen in HIV-infected individuals. The organism Aspergillus fumigates (choice A) is associated with the development of Aspergillosis, which is an allergic bronchopulmonary disease. Candida albicans (choice B) primarily affects the gastrointestinal mucosa and is also known to cause candidal endocarditis. Coccidioides immitis (choice C) is a mold that grows in the soil of areas with arid climates. This organism typically causes respiratory tract infections, which can lead to arthralgia and periarticular swelling when this infection spreads to the bones. Pneumocystis carinii (choice E) is a fungus which causes bilateral diffuse interstitial pneumonia. This type of pneumonia is most often seen in HIV-infected individuals.
Which of the following organs is the most uncommon site for infarcts?
1. Brain
2. Heart
3. Liver
4. Kidney
5. Adrenals
3. Liver

The correct answer is choice 3. Infarctions are regions of tissue necrosis resulting from acute decrease in blood supply. They occur primarily when arterial flow is disrupted. In this question, we will find the most uncommon site for infarcts by selecting the liver as the organ best supplied with blood. The blood supply to the liver is so extensive and overlapping that decreased flow in one artery or arteriole does not necessarily compromise the supply to the area as an overlapping supply exists from a nearby artery or arteriole. In addition, the liver has what is known as a dual blood supply, with each area receiving blood not only from the hepatic artery, but also from the hepatic portal vein. Choice 1, the brain, is a common site for infarcts. These infarcts are commonly known as strokes. Choice 2, the heart, is incorrect. Myocardial infarction, commonly known as a heart attack, is also a common condition. Choices 4 and 5, while not common places for infarcts, do have infarcts occasionally and do not have a dual blood supply. So the correct answer to question is choice 3.
Which of the following particles does NOT contain HbsAg?
A. Dane particle
B. Spheres
C. Tubules (filaments)
D. Delta particle
D. Delta particle

The correct answer is choice D. The Hepatitis B surface antigen( HbsAg) is an antigenic component found on a variety of particles of Hepatitis B. The actual hepatitis B virus (HBV) is a DNA virus and the particle is known as the Dane particle (choice A). It contains both DNA and a protein coat. Excess production of the protein coat results in the presence of spheres (choice B) and tubules (filaments) (choice C) of protein coat, which also contain the surface antigen. The Dane particle also contains the two other Hepatitis B antigens, known as the core antigen and the e-antigen. Delta antigen is a component of Hepatitis D or delta hepatitis. It can be transmitted only concurrently with hepatitis B but is a distinct disease entity with an RNA genome.
Which of the following pneumoconioses is most often associated with bronchogenic carcinoma and mesothelioma in man?
1. Silicosis
2. Asbestosis
3. Anthracosis
4. Berylliosis
2. Asbestosis

The correct choice is choice 2. asbestosis. Asbestos exposure has been linked to the development of mesothelioma and bronchogenic carcinoma. Mesothelioma is often a rapidly fatal tumor arising from the pleura or peritoneum. It is a very rare disease among the general population, but as many as 10% of heavily exposed individuals will develop mesothelioma. This cancer frequently shows a very long latency period. Often it does not occur for up to 50 years following asbestos exposure. The incidence of bronchogenic carcinoma is slightly increased among those exposed to asbestos. However, among those exposed who also smoke heavily, the incidence of bronchogenic carcinoma is increased up to ninety-fold as compared to controls. Silicosis, choice 1, is associated with both acute and chronic lung disease. In the acute form, an exudative pneumonitis occurs and death ensues within one year. In the chronic form, the patient's pulmonary function gradually deteriorates over many years. During these years, the lung is gradually replaced by fibrous tissue. Pulmonary tuberculosis complicates chronic silicosis as does rheumatoid arthritis. Anthracosis, choice 3, is a common lung finding among smokers and city dwellers. It refers to the accumulation of carbon in macrophages in the lung. Anthracosis by itself is of no clinical significance, but may be present in severe diseases including coal worker's pneumoconiosis. Berylliosis, choice 4, is a granulomatous disease of the lung resulting from heavy beryllium exposure. Exposure to beryllium has been associated with the development of bronchogenic carcinoma, but it has not been shown to play an etiologic role in the development of mesothelioma. Once again, the correct choice to question is choice 2.
Which of the following represent(s) the MOST common source of pulmonary embolism?
A. Esophageal varices
B. Endarteritis
C. Lymphangitis
D. Buerger's disease
E. Thrombophlebitis
E. Thrombophlebitis

The correct answer is choice E. Pulmonary emboli are formed in the systemic venous system, travel through the right atrium, right ventricle, pulmonary artery, and then lodge in the lung. They are clots (thrombi) formed within veins, especially deep leg veins. Esophageal varices usually cause severe bleeding and are discussed in other questions. Endarteritis is inflammation of the lining of arteries, and lymphangitis is inflammation of the lining of lymph vessels. Neither causes pulmonary emboli. Buergers disease involves immune complexes causing damage to arteries and other vessels. It often causes thrombosis in legs. So the correct answer to question is choice E.
Which of the following represents the chief complication of mumps in the adult male?
A. Orchitis
B. Prostatitis
C. Glomerulonephritis
D. Chronic nonspecific sialadenitis
A. Orchitis

The correct answer is choice A. Orchitis is the swelling of the testes commonly found in male mumps patients. It often develops about 1 week after the parotid gland swelling. It is more common in older male patients (over l0), and less common in those less than 10. Prostatitis and glomerulonephritis are not generally associated with mumps. Sialadenitis (inflammation of the salivary gland) is associated with mumps, especially that of the parotid gland. However, it would be acute, not chronic, and is caused by the mumps virus, a paramyxovirus, and therefore is not nonspecific. So the correct answer to question is choice A.
Which of the following represents the MOST frequent cause of a clinically palpable breast mass in an adult woman?
A. Sarcoma
B. Fibroadenoma
C. Adenocarcinoma
D. Fibrocystic disease
E. lntraductal papilloma
D. Fibrocystic disease

The correct answer is choice D. Fibrocystic disease is the most common breast disorder(more common than all tumors). It includes both fibrosis of areas of the breast and formation of serous fluid-filled cysts. Fibroadenoma is the most common tumor, and is benign, and usually single. Most malignant tumors of the breast are varieties of carcinomas, and not sarcomas. Intraductal papilloma is generally a solitary lesion within a duct or cyst. When single, it is usually benign. So the correct answer to question is choice D.
Which of the following represents the MOST potent carcinogen?
A. Estrogen
B. Benzpyrene
C. Folic acid
D. Cholic acid
B. Benzpyrene

The correct answer is choice B. Of the compounds listed, benzpyrene is by far the most potent carcinogen. Common proven chemical carcinogens include benzpyrene, benzene, aniline dyes, and asbestos. So the correct answer to question is choice B.
Which of the following represents the MOST reliable postmortem indicator of left ventricular cardiac failure?
A. Ascites
B. Venous congestion
C. Enlargement of the spleen
D. Peripheral edema of the ankles
E. Chronic passive congestion of the lungs
E. Chronic passive congestion of the lungs

The correct answer is choice E. In left ventricular failure, the lungs become congested with tissue fluid. If you review the path of blood in the circulatory system, you will note that blood in the left ventricle comes from the left atrium. This blood in the atrium has just arrived in the pulmonary vein from the lungs. Ineffective pumping by the left ventricle therefore causes higher than normal pressure in the pulmonary vein and lungs, and congestion results. Other signs of left sided failure include hypoperfusion of the kidney and brain. Edema of the ankles, general systemic venous congestion, and ascites are generally signs of right sided failure, with an analogous backup of fluid into the systemic venous system, rather than the pulmonary. Ascites refers to edematous swelling of the abdomen. So the correct answer to question is choice E.
Which of the following serum values is likely to be elevated in a patient with hyperparathyroidism?
1. Calcium
2. Chloride
3. Potassium
4. Phosphorus
5. Acid phosphatase
1. Calcium

The correct answer is choice 1.
In hyperparathyroidism, especially the primary type, serum calcium is elevated. Choice 2, chloride, and choice 3, potassium, are unaffected. Choice 4, phosphorus, will be elevated in cases of hypoparathyroidism. Choice 5, acid phosphatase, is elevated in cases of prostatic carcinoma as discussed earlier. Alkaline phosphatase levels are elevated in Paget's disease of bone, although this is not a choice here. Anyway, the correct answer to question is choice 1.
Which of the following species of streptococci is usually NOT found in human dental plaque?
1. S. mutans
2. S. sanguis
3. S. pyogenes
4. S. salivarius
5. S. mitior S. (mitis)
3. S. pyogenes

The correct choice is choice 3, S. pyogenes. Dental plaque is complex mixture of bacteria and dextran which adheres to teeth and causes dental caries. Many microorganisms may be cultured from dental plaque, the most common of which are Streptococcal species. Step. mutans, mitor and sanguis are present in high amounts, but some Strep. salivarius is deposited from saliva. In addition to these organisms, the following organisms are among those found in dental plaque: Staph. epidermitis, Neisseria, Actinomycin and Haemophilus. Strep. pyogenes is a pathogen of the pharynx where it causes Streptococcal pharyngitis. If not adequately treated, Streptococcal pharyngitis may induce glomerular nephritis or rheumatic fever. Strep. pyogenes also causes cellulitis, lymphangitis, impetigo, and post-partum infections of the uterus. Once again, the correct choice for question is choice 3.
Which of the following viral-associated enzymes is unique to RNA tumor viruses?
1. Neuramidase
2. Capping enzyme
3. Reverse transcriptase
4. DNA-dependent RNA polymerase
3. Reverse transcriptase

The correct choice is answer #3. The fundamental axiom of modern biology states that protein synthesis proceeds from DNA to RNA to protein. The RNA viruses consist of merely a protein coat surrounding an RNA nucleic acid core. These viruses are able to replicate and synthesize protein only by first generating DNA. This is accomplished with the use of reverse transcriptase which synthesizes DNA from an RNA template. Once DNA is formed, the DNA acts as a template for RNA synthesis. Some of this RNA will form the core of the new virus particles. However, much of this RNA is messenger RNA utilized for the production of viral proteins including reverse transcriptase. Neuraminidase is an enzyme present in the outer membrane of para-myxoviruses and orthomyxoviruses which are RNA viruses, but is also produced by the bacteria Vibrio cholerae. Consequently, choice 1 is incorrect. This enzyme cleaves N-acetylneuraminic acid, also called NANA, from glycoproteins on the surface of red blood cells. NANA is essential for hemoagglutination. Thus, neuraminidase inhibits this process. Capping enzymes, choice 2, are present in all cells. They are not present in viruses. These proteins attach to the ends of cytoskeletal proteins and determine the rate at which these ends elongate or depolymerize. They play a vital role in determining cell shape. DNA-dependent RNA-polymerase is present in all animal cells. Choice 4 is therefore incorrect. This enzyme or group of enzymes is responsible for transcription, the process whereby RNA is formed from a DNA template. So the correct choice to question is answer 3.
Which of the following viruses causes two distinct diseases in different age groups?
1. Influenza
2. Measles
3. Smallpox
4. Varicella
5. Newcastle disease
4. Varicella

The correct choice is choice 4, Varicella. The Varicella zoster virus, a herpes virus, causes chicken pox in young children. Following infection, the virus enters a dormant phase in the dorsal ganglia. Reactivation of the virus occurs under conditions of fir decreased immunocompetence. Elderly individuals have a less vigorous immune system and reactivation commonly occurs in this population. Reactivation rarely causes disseminated disease. It commonly presents as vesicular lesions confined within a single dermatome in a condition known as shingles. The lesions are quite painful and they are fully infectious. Influenza, choice 1, destroys the respiratory mucosa of infected individuals. Elderly individuals may succumb more readily to complications of this disease, but the disease that they acquire is identical to that of their younger counterparts. Measles, choice 2, can manifest itself in a variety of ways. It most cc>mmonly causes a diffuse maculopapolar rash with severe constitutional symptoms. Less often, it causes pneumonia and encephalitis. Rarely, it causes subacute sclerosirlg panencephalitis (SSPE) which is a degenerative neurologic disease characterized by myoclonic jerks and motor deterioration. Pneumonia, encephalitis and SSPE are considered complications of measles infection. They can affect any individual infected with the measles virus and are not age-specific. Smallpox, choice 4, causes a diffuse vesicular skin rash similar to chicken pox. This virus has been eradicated worldwide due to successful vaccination programs. It no longer causes disease. Newcastle disease, choice 5, is a respiratory tract disease of chickens. The Newcastle disease virus, however, can cause disease in humans exposed. This disease is common among poultry workers and among researchers who work with the Newcastle virus. When humans are infected, they usually do not manifest respiratory symptoms as do chickens. Rather, they acquire a mild conjunctivitis. Once again, the correct choice to question is choice 4.
Which of the following viruses is strongly associated with Burkitt's lymphoma and nasopharyngeal carcinoma?
1. Adenovirus
2. Cytomegalovirus
3. Epstein-Barr virus
4. Herpes simplex virus type I
5. Herpes simplex virus type II
3. Epstein-Barr virus

The correct answer is choice 3. Epstein Barr virus is the causative agent of infectious mononucleosis. It is in the herpes family that includes Herpes-simplex 1, choice 4, which causes herpes labialis and herpetic gingivostomatitis. Also in the family is herpes-simplex type 2, causative agent of genital herpes and neonatal herpes. Also in the same family is choice 2, CMV , or cytomegalovirus, causative agent of cytomegalic inclusion disease, a generalized infection of infants. Adenoviruses , choice 1, are not in the herpes family and cause a variety of respiratory, eye and gastrointestinal infections. Our answer, Epstein Barr virus, besides causing infection, is believed to be associated with the B-cell lymphomas, Burkitt's lymphoma and nasopharyngeal carcinoma, all neoplasms. The virus is not implicated in actually causing the neoplasm but is associated in some way with its development. So the correct answer to question is choice 3.
Which organism is LEAST likely to be involved in lesions of chronic adult periodontitis?
A. Spirochetes
B. Bacteroides melaninogenicus
C. Capnocytophaga
D. Streptococcus mitis
E. Fusobacteria
D. Streptococcus mitis

The correct answer is choice D. Periodontal disease pathogens are usually anaerobic or capnophilic. The term capnophilic means "carbon dioxide-loving". In the anaerobic environment deep in the sulcus, the following genera are often said to contribute to the tissue destruction of periodontal disease: Bacteroides, Porphyromonas gingivalis, Borelia. Spirochetes, Capnocytophaga, Fusobacteria and Eichenella. Streptococci (mutans, mitis, sanguis, salivarius) are all gram positive facultative anaerobes that do not require anaerobic conditions to survive. They are Viridans streptococci, generally nonpathogenic, and not linked to periodontal disease, although S. mutans is clearly linked to caries.
Which pair of bacterial genera produce spores?
A. Clostridium and Staphylococci
B. Bacillus and Streptococci
C. Staphylococci and Streptococci
D. Clostridium and Bacillus
E. Bacillus and Staphylococci
D. Clostridium and Bacillus

The correct answer is choice D. The two genera of spore forming bacteria are Clostridium and Bacillus. Clostridium is most known for causing gas gangrene, (C. perfringens), botulism, (C. botuiinum) and tetanus (C. tetani). Bacillus is known for anthrax (Bacillus anthracis) and food poisoning (B. cereus). Bacillus stearothermophilus is used for the autoclave spore test (biological monitor), due to the extreme conditions needed to kill its spores.